Insurance Case Digests (1)

Published on February 2017 | Categories: Documents | Downloads: 93 | Comments: 0 | Views: 864
of 74
Download PDF   Embed   Report

Comments

Content

Insurance Case Digests
Atty. Bathan-Basuel

CASES: INSURANCE
1.

Secs 1-9
Travellers v. CA

[G.R. No. 82036. May 22, 1997]
Travellers Insurance & Surety Corporation vs. Hon. Court of Appeals &
Vicente Mendoza
Facts:
Vicente Mendoza, Jr. as heir of his mother (Feliza Vineza de Mendoza) who was
killed in a vehicular accident, filed an action for damages against the erring taxicab
driver (Rodrigo Dumlao), the owner (Armando Abellon) of the taxicab (Lady Love Taxi
with Plate No. 438-HA Pilipinas Taxi 1980) and the alleged insurer of the vehicle which
featured in the vehicular accident. The erring taxicab was allegedly covered by a thirdparty liability insurance policy issued by petitioner Travellers Insurance & Surety
Corporation. Petitioner was included in the complaint as the compulsory insurer of the
said taxicab under Certificate of Cover No. 1447785-3.
The trial court rendered judgment in favor of private respondent and ordered
Rodrigo Dumlao, Armando Abellon and petitioner to pay private respondent death
indemnity, moral damages, exemplary damages, attorney’s fees and other litigation
expenses, jointly and severally.
The decision was affirmed by the CA and the subsequent MR was denied.
Hence this petition.
ISSUE: Whether petitioner is liable to private respondent?
HELD: NO.
I. The right of the person injured to sue the insurer of the party at fault (insured),
depends on whether the contract of insurance is intended to benefit third persons also
or on the insured. And the test applied has been this: Where the contract provides
for indemnity against liability to third persons, then third persons to whom the insured
is liable can sue the insurer. Where the contract is for indemnity against actual loss or
payment, then third persons cannot proceed against the insurer, the contract being
solely to reimburse the insured for liability actually discharged by him thru payment to
third persons, said third persons’ recourse being thus limited to the insured alone.”
The trial court did not distinguish between the private respondent’s cause of
action against the owner and the driver of the Lady Love taxicab and his cause of
action against petitioner. The former is based on torts and quasi-delicts while the
latter is based on contract. Confusing these two sources of obligations as they arise
from the same act of the taxicab fatally hitting private respondent’s mother, and in the
face of overwhelming evidence of the reckless imprudence of the driver of the Lady
Love taxicab, the trial court brushed aside its ignorance of the terms and conditions of
2-S, 2013-2014

the insurance contract and forthwith found all three - the driver of the taxicab, the
owner of the taxicab, and the alleged insurer of the taxicab - jointly and severally liable
for actual, moral and exemplary damages as well as attorney’s fees and litigation
expenses. This is clearly a misapplication of the law by the trial court, and respondent
appellate court grievously erred in not having reversed the trial court on this ground.
“While it is true that where the insurance contract provides for indemnity against
liability to third persons, such third persons can directly sue the insurer, however, the
direct liability of the insurer under indemnity contracts against third-party liability does
not mean that the insurer can be held solidarily liable with the insured and/or the other
parties found at fault. The liability of the insurer is based on contract; that of the
insured is based on tort.”
II. At the time of the vehicular incident which resulted in the death of private
respondent’s mother, during which time the Insurance Code had not yet been amended
by Batas Pambansa (B.P.) Blg. 874, Section 384 provided as follows:
“Any person having any claim upon the policy issued pursuant to this chapter shall,
without any unnecessary delay, present to the insurance company concerned a written
notice of claim setting forth the amount of his loss, and/or the nature, extent and
duration of the injuries sustained as certified by a duly licensed physician. Notice of
claim must be filed within six months from date of the accident, otherwise, the claim
shall be deemed waived. Action or suit for recovery of damage due to loss or injury
must be brought in proper cases, with the Commission or the Courts within one year
from date of accident, otherwise the claimant’s right of action shall prescribe”
[emphasis and underscoring supplied].
It is significant to note that the aforecited Section 384 was amended by B.P. Blg.
874 to categorically provide that “action or suit for recovery of damage due to loss or
injury must be brought in proper cases, with the Commissioner or the Courts within one
year from denial of the claim, otherwise the claimant’s right of action shall
prescribe” [emphasis ours].
We have certainly ruled with consistency that the prescriptive period to bring suit
in court under an insurance policy, begins to run from the date of the insurer’s rejection
of the claim filed by the insured, the beneficiary or any person claiming under an
insurance contract. This ruling is premised upon the compliance by the persons suing
under an insurance contract, with the indispensable requirement of having filed the
written claim mandated by Section 384 of the Insurance Code before and after its
amendment. Absent such written claim filed by the person suing under an insurance
contract, no cause of action accrues under such insurance contract, considering that it
is the rejection of that claim that triggers the running of the one-year prescriptive
period to bring suit in court, and there can be no opportunity for the insurer to even
reject a claim if none has been filed in the first place, as in the instant case.
WHEREFORE, the instant petition is HEREBY GRANTED.
1

Insurance Case Digests
Atty. Bathan-Basuel

Gulf Resorts v. Phil Charter
GULF RESORTS, INC. VS. PHILIPPINE CHARTER INSURANCE CORPORATION
G.R. No. 156167 May 16, 2005
Puno, J.;
FACTS:
Plaintiff Gulf Resorts is the owner of the Plaza Resort situated at Agoo, La
Union and had its properties in said resort insured originally with the American Home
Assurance Company or AHAC-AIU. In the first four insurance policies issued by AHACAIU the risk of loss from earthquake shock was extended to only the plaintiff’s two
swimming pools, thus, “earthquake shock endt.” Subsequently, AHAC-AIU issued in the
plaintiff’s favor Policy number 206-4182383-0 covering the period March 14, 1988 to
March 14, 1989 and in the said policy, the earthquake endorsement clause was deleted
and the entry under Endorsements/Warranties at the time of issue read that plaintiff
renewed its policy with AHAC-AIU for the period of March 14, 1989 to March 14, 1990
under
policy
number
206-4568061-9
which
carried
the
entry
under
“Endorsement/Warranties at Time of Issue” which read “Endorsement to Include
Earthquake Shock” in the amount of P10,700 and paid P42,658.14 as premium thereof.
The plaintiff agreed to insure with defendant the properties covered by AHACAIU Policy number 206-4568061-9 provided that the policy wording and rates in the
said policy be copied in the policy to be issued by defendant. The defendant then issued
Policy number 31944 to plaintiff covering the period of March 14, 1990 to March 14,
1991 for P10,700,600 for a total premium of P45,159.92. In Policy number 31944
issued by defendant, the shock endorsement provide:
“In consideration of the payment by the insured to the
company of the sum included additional premium the Company agrees,
notwithstanding what is stated in the printed conditions of this policy due to
the contrary, that this insurance covers loss or damage to shock to any of
the property insured by this Policy occasioned by or through or in
consequence of earthquake.”
That in the said policy, the word “included” was deleted. On July 16, 1990 an
earthquake struck Central Luzon and Northern Luzon and the plaintiff’s properties
covered by Policy No. 31944 issued by defendant, including the two swimming pools in
its Agoo Playa Resort were damaged.
After the earthquake, petitioner advised respondent that it would be making a
claim under its Insurance Policy No. 31944 for damages on its properties. Respondent
instructed petitioner to file a formal claim, then assigned the investigation of the claim
to an independent claims adjuster. On July 30, 1990, respondent through its adjuster,
requested petitioner to submit various documents in support of its claim. Subsequently,
the adjuster rendered a decision saying “except for the swimming pools, all affected
items have no coverage for earthquake shocks.” Petitioner then filed its formal demand
for settlement for the damage to all its properties in the Agoo Playa Resort.
2-S, 2013-2014

The regional trial court ruled in favor of respondent stating that only the 2
swimming pools had earthquake shock coverage. Upon appeal, the appellate court
affirmed the decision of the trial court.
ISSUE:
Whether or not the other properties in the Agoo Playa Resort are included in
the earthquake shock coverage of the said insurance policy
HELD:
It is basic that all the provisions of the insurance policy should be examined
and interpreted in consonance with each other. All its parts are reflective of the true
intent of the parties. The policy cannot be construed piecemeal.
Petitioner cannot focus on the earthquake shock endorsement to the exclusion
of the other provisions. All the provisions and riders, taken and interpreted together,
indubitably show the intention of the parties to extend earthquake shock coverage to
the two swimming pools only.
A careful examination of the premium recapitulation will show that it is the
clear intent of the parties to extend earthquake coverage shock only to the 2 swimming
pools. Section 2(1) of the Insurance Code defines a contract of insurance as an
agreement whereby one undertakes for a consideration to indemnify another against
loss, damage or liability arising from an unknown or contingent event. Thus, an
insurance contract exists where the following elements concur:
1. the insured has an insurable interest
2. the insured is subject to a risk of loss by the happening of the designated
peril
3. the insurer assumes the risk
4. such assumption of risk is part of a general scheme to distribute actual
losses among a large group of persons bearing similar risk
5. in consideration of the insurer’s promise, the insured pays a premium
an insurance premium is the consideration paid an insurer for undertaking to
indemnify the insured against a specified peril. In the subject policy, no
premium payments were made with regard to earthquake shock coverage,
except on the 2 swimming pools. There is no mention of any premium payable
for the other resort properties with regard to earthquake shock. This is
consistent with the history of petitioner’s previous insurance policies from
AHAC-AIU. Hence, the judgment of the Court of Appeals is affirmed.
New World v. NYK Fil-Japan
FACTS: Petitioner New World International Development (Phils.), Inc. (New
World) bought from DMT Corporation (DMT) through its agent, Advatech Industries, Inc.
(Advatech) three emergency generator sets worth US$721,500.00.

2

Insurance Case Digests
Atty. Bathan-Basuel

DMT shipped the generator sets by truck from Wisconsin, United States, to
LEP Profit International, Inc. (LEP Profit) in Chicago, Illinois. From there, the shipment
went by train to Oakland, California, where it was loaded on S/S California Luna
V59, owned and operated by NYK Fil-Japan Shipping Corporation (NYK) for
delivery to petitioner New World in Manila. NYK issued a bill of lading, declaring
that it received the goods in good condition.
NYK unloaded the shipment in Hong Kong and transshipped it to S/S
ACX Ruby V/72 that it also owned and operated.
On its journey to Manila, however, ACX Ruby encountered typhoon
Kadiang whose captain filed a sea protest on arrival at the Manila South Harbor on
October 5, 1993 respecting the loss and damage that the goods on board his vessel
suffered.
Marina Port Services, Inc. (Marina), the Manila South Harbor arrastre or
cargo-handling operator, received the shipment on October 7, 1993. Upon inspection
of the three container vans separately carrying the generator sets, two vans bore
signs of external damage while the third van appeared unscathed.
An examination of the three generator sets in the presence of petitioner
New World’s representatives, Federal Builders (the project contractor) and surveyors of
petitioner New World’s insurer, Seaboard–Eastern Insurance Company
(Seaboard), revealed that all three sets suffered extensive damage and could
no longer be repaired.
New World demanded recompense for its loss from respondents
NYK, DMT, Advatech, LEP Profit, LEP International Philippines, Inc. (LEP), Marina, and
Serbros. While LEP and NYK acknowledged receipt of the demand, both
denied liability for the loss.
Since Seaboard covered the goods with a marine insurance policy, New
World sent it a formal claim dated November 16, 1993. Replying on February 14,
1994, Seaboard required petitioner New World to submit to it an itemized list of the
damaged units, parts, and accessories, with corresponding values, for the processing of
the claim. But petitioner New World did not submit what was required of it,
insisting that the insurance policy did not include the submission of such a
list in connection with an insurance claim. Reacting to this, Seaboard refused
to process the claim.
On October 11, 1994 New World filed an action for specific performance and
damages against all the respondents before the Regional Trial Court (RTC)
of Makati City.
ISSUE: Whether CA erred in ruling that Seaboard’s request from petitioner New World
for an itemized list is a reasonable imposition and did not violate the insurance
contract between them; and
2-S, 2013-2014

HELD: YES. Itemized listing is not substantially necessary.
The record shows that petitioner New World complied with the documentary
requirements evidencing damage to its generator sets.
The marine open policy that Seaboard issued to New World was an all-risk
policy. Such a policy insured against all causes of conceivable loss or damage
except when otherwise excluded or when the loss or damage was due to
fraud or intentional misconduct committed by the insured. The policy
covered all losses during the voyage whether or not arising from a marine peril.
The policy enumerated certain exceptions like unsuitable packaging, inherent
vice, delay in voyage, or vessels unseaworthiness, among others. But Seaboard had
been unable to show that petitioner New World’s loss or damage fell
within some or one of the enumerated exceptions.
Moreover, New World has submitted various documents, namely:
(1) copy of the Supplier’s Invoice;
(2) copy of the Packing List;
(3) copy of the Bill of Lading;
(4) the Delivery of Waybill Receipts 1135, 1222, and 1224;
(5) original copy of Marine Insurance Policy MA-HO-000266;
(6) copies of Damage Report from Supplier and Insurance Adjusters;
(7) Consumption Report from the Customs Examiner; and
(8) Copies of Received Formal Claim from the following:
a) LEP International Philippines, Inc.;
b) Marina Port Services, Inc.; and
c) Serbros Carrier Corporation.
Seaboard cannot pretend that the above documents are inadequate since they
were precisely the documents listed in its insurance policy. Being a contract of
adhesion, an insurance policy is construed strongly against the insurer who prepared
it. The Court cannot read a requirement in the policy that was not there.
White Gold v. Pioneer
WHITE GOLD MARINE SERVICES, INC., petitioner vs. PIONEER INSURANCE
AND SURETY CORPORATION and THE STEAMSHIP MUTUAL UNDERWRITING
ASSOCIATION (BERMUDA) LTD., respondents
Ponente: QUISUMBING
FACTS:
White Gold procured a protection and indemnity coverage for its vessels from The
Steamship Mutual through Pioneer.
It was issued a Certificate of Entry and
Acceptance. Pioneer also issued receipts evidencing payments. When White Gold
failed to fully pay its accounts, Steamship Mutual refused to renew the coverage.
3

Insurance Case Digests
Atty. Bathan-Basuel

Steamship Mutual filed a case against White Gold for collection of sum of
money. White Gold filed a complaint before the Insurance Commission claiming that
Steamship Mutual violated Sections 186 and 187 of the Insurance Code, while Pioneer
violated Sections 299-301 in relation to Sections 302-303. It was dismissed, ruling
that there was no need for Steamship Mutual to secure a license because it was not
engaged in the insurance business, as a Protection and Indemnity Club (P & I
Club). Pioneer also need not obtain another license as insurance agent and/or a broker
for Steamship Mutual. Moreover, Pioneer was already licensed. CA affirmed. Hence, the
petition.
A P & I Club is “an association composed of ship owners in general who band together
for the specific purpose of providing insurance cover on a mutual basis against liabilities
incidental to ship owning that the members incur in favor of third parties.” As a P & I
Club, Steamship Mutual’s primary purpose is to solicit and provide protection and
indemnity coverage and for this purpose, it has engaged the services of Pioneer to act
as its agent.
ISSUE:
Whether Steamship Mutual, a P & I Club, is engaged in the insurance business in the
Philippines. YES
Whether Pioneer needs a license as an insurance agent/broker for Steamship Mutual.
YES
HELD:
Section 2 (2) of the Insurance Code enumerates what constitutes “doing an insurance
business” or “transacting an insurance business”. The test to determine if a contract is
an insurance contract or not, depends on the nature of the promise, the act required to
be performed, and the exact nature of the agreement in the light of the occurrence,
contingency, or circumstances under which the performance becomes requisite. It is
not by what it is called. Basically, an insurance contract is a contract of indemnity. In
it, one undertakes for a consideration to indemnify another against loss, damage or
liability arising from an unknown or contingent event.
In particular, a marine insurance undertakes to indemnify the assured against marine
losses, such as the losses incident to a marine adventure. A mutual insurance company
is a cooperative enterprise where the members are both the insurer and
insured. Additionally, mutual insurance associations, or clubs, provide three types of
coverage, namely, protection and indemnity, war risks, and defense costs.
A P & I Club is “a form of insurance against third party liability, where the third party is
anyone other than the P & I Club and the members.” By definition then, Steamship
Mutual as a P & I Club is a mutual insurance association engaged in the marine insurance
business.

involves public interest, regulation by the State is necessary. Thus, no insurer or
insurance company is allowed to engage in the insurance business without a license or a
certificate of authority from the Insurance Commission.
Although Pioneer is already licensed as an insurance company, it needs a separate
license to act as insurance agent for Steamship Mutual. CA decision REVERSED AND
SET ASIDE. Steamship Mutual and Pioneer are ORDERED to obtain licenses.
RP v. Del Monte
Republic of the Philippines, (represented by Eduardo Malinis in his capacity
as Insurance Commisioner) vs. Del Monte Motors Inc.
G.R. No, . 156956 October 9, 2006
C.J. Panganiban
Facts:
On January 15, 2002, the RTC rendered a decision in a civil case finding Vilfran
Liner, Hilaria Villegas and Maura Villegas jointly and severally liable to pay Del Monte
Motors lost P11,835,375.50 for service contracts with Del Monte. The Trial Court
further ordered the execution of the decision against the counterbond posted by Vilfran
and issued by Capital Insurance and Surety Co. Inc. (CISCO).
CISCO opposed the Motion for Execution claiming that they had no record or
document regarding the alleged issuance of the counterbond, theus the bond was not
valid and enforceable. However, the RTC released a motion for execution commanding
the sheriff to levy the amount on the property of CISCO. To completely satisfy the
amount, the Insurance Commissioner was also commanded to withdraw the security
deposit filed by CISCO with the Commission according to Sec 203 of the Insurance
Code.
Insurance Commissioner Malinis was ordered by the RTC to withdraw the
security bond of CISCO for the payment of the insurance indemnity won by Del Monte
Motor against Vilfran Liner, the insured. Malinis didn’t obey the order, so the respondent
moved to cite him in contempt of Court. The RTC ruled against Malinis. It explained that
the commissioner had no legal justification for his refusal to allow the withdrawal of
CISCO’s security deposit. Hence, this petition.
Issues:
1. Whether or not the security deposit held by the Insurance Commissioner pursuant to
Section 203 of the Insurance Code may be levied or garnished in favor of only one
insured.
2. Whether or not the Insurance Commissioner has power to withhold the release of the
security deposit.
Held:
1. NO.

Thus, to continue doing business here, Steamship Mutual or through its agent Pioneer,
must secure a license from the Insurance Commission. Since a contract of insurance
2-S, 2013-2014

Sec 203 of the Insurance Code provides, “no judgment creditor or other
claimant shall have the right to levy upon any of the securities of the insurer held on
4

Insurance Case Digests
Atty. Bathan-Basuel

deposit pursuant to the requirement of the Commissioner.” The court also claimed that
the security deposit shall be (1) answerable for all the obligations of the depositing
insurer under its insurance contracts; (2) at all times free from any liens or
encumbrance; and (3) exempt from levy by any claimant.
“To allow the garnishment of that deposit would impair the fund by decreasing
it to less than the percentage of paid-up capital that the law requires to be maintained.
Further, this move would create, in favor of respondent, a preference of credit over the
other policy holders and beneficiaries.”
“Also, the securities are held as a contingency fund to answer for the claims
against the insurance company by all its policy holders and their beneficiaries. This step
is taken in the event that the company becomes insolvent or otherwise unable to
satisfy the claims against it. Thus, a single claimant may not lay stake on the securities
to the exclusion of all others. The other parties may have their own claims against the
insurance company under other insurance contracts it has entered into.”

COMMISSIONER OF INTERNAL REVENUE, Respondent.

2. YES.

CIR appealed the CTA decision to CA insofar as it cancelled the DST
assessment claiming that petitioner’s health care agreement was a contract of
insurance subject to DST under Section 185 of the 1997 Tax Code.

The Insurance Code has vested the Office of the Insurance Commission with
both regulatory and adjudicatory authority over insurance matters. Under Sec 414 of
the Insurance Code, "The Commissioner may issue such rulings, instructions, circulars,
orders and decisions as he may deem necessary to secure the enforcement of the
provisions of this Code.”
“The commissioner is authorized to (1) issue (or to refuse to issue)
certificates of authority to persons or entities desiring to engage in insurance business
in the Philippines;16 (2) revoke or suspend these certificates of authority upon finding
grounds for the revocation or suspension; (3) impose upon insurance companies, their
directors and/or officers and/or agents appropriate penalties -- fines, suspension or
removal from office -- for failing to comply with the Code or with any of the
commissioner's orders, instructions, regulations or rulings, or for otherwise conducting
business in an unsafe or unsound manner.”
Included here is the duty to hold security deposits under Secs 191 and 202 of
the Code for the benefit of policy holders. Sec 192, on the other hand, states: “the
securities deposited as aforesaid shall be returned upon the company's making
application therefor and proving to the satisfaction of the Commissioner that it has no
further liability under any of its policies in the Philippines.”
He has been given great discretion to regulate the business to protect the
public. Also “An implied trust is created by the law for the benefit of all claimants under
subsisting insurance contracts issued by the insurance company.” He believed that the
security deposit was exempt from execution to protect the policy holders.
Philhealth v. CIR
PHILHEALTH CARE PROVIDERS vs. CIR
G.R. No. 167330
September 18, 2009

PHILIPPINE HEALTH CARE PROVIDERS, INC., Petitioner,
2-S, 2013-2014

CORONA, J.:
FACTS:
On January 27, 2000, respondent Commissioner of Internal Revenue [CIR] sent
Philhealth a formal demand letter and the corresponding assessment notices demanding
the payment of deficiency taxes for the taxable years 1996 and 1997 in the total
amount of P224,702,641.18. Philhealth protested the assessment and, as CIR did not
act on the protest, Philhealth filed a petition for review in the Court of Tax Appeals
(CTA) seeking the cancellation of the deficiency VAT and DST (documentary stamp tax)
assessments. CTA rendered a decision ordering the Philhealth to pay the deficiency VAT
for 1996 and 1997 and canceling the 1996 and 1997 deficiency DST assessment
against it.

CA reversed the decision of CTA and upheld that the petitioner’s health care
agreement was in the nature of non-life insurance contract subject to DST.
ISSUES:
1. WON the respondent is a Health Maintenance Organization (HMO)
2. WON petitioner, as an HMO, engaged in the business of insurance during the
pertinent taxable years
3. WON a health care agreement is an insurance contract.
HELD:
1. Philhealth is admittedly an HMO. Under RA 7875 (or "The National Health
Insurance Act of 1995"), an HMO is “an entity that provides, offers or arranges for
coverage of designated health services needed by plan members for a fixed prepaid
premium.”
2. NO, Philhealth is not engaged in the business of insurance. Pursuant to the
“principal object and purpose test” adopted by the Supreme Court, the primary purpose
of a medical service corporation, such as Philhealth, is NOT the assumption of risk and
indemnification, but rather to provide physicians who will render services to subscribers
on a prepaid basis. In fact, a substantial portion of petitioner’s services covers
preventive and diagnostic medical services intended to keep members from developing
medical conditions or diseases. As an HMO, it is its obligation to maintain the good
health of its members. Accordingly, its health care programs are designed to prevent or
to minimize the possibility of any assumption of risk on its part. Thus, its undertaking
under its agreements is not to indemnify its members against any loss or damage
arising from a medical condition but, on the contrary, to provide the health and medical
services needed to prevent such loss or damage. Petitioner, as an HMO, is not part of
5

Insurance Case Digests
Atty. Bathan-Basuel

the insurance industry. This is evident from the fact that it is not supervised by the
Insurance Commission but by the Department of Health.
3. NO, a healthcare agreement is not an insurance contract. The agreements
between the petitioner and its members do not possess all the elements of an insurance
contract provided in Sec. 2(1) of the Insurance Code because the healthcare's primary
purpose is to render service and not to assume risk and indemnify another. Also, there
is nothing in petitioner's agreements that gives rise to a monetary liability on the part
of the member to any third party-provider of medical services which might in turn
necessitate indemnification from petitioner. The terms "indemnify" or "indemnity"
presuppose that a liability or claim has already been incurred. There is no indemnity
precisely because the member merely avails of medical services to be paid or already
paid in advance at a pre-agreed price under the agreements.
NOTE:


“Principal object and purpose test” of United State jurisprudence, to wit:
whether the assumption of risk and indemnification of loss (which are elements
of an insurance business) are the principal object and purpose of the
organization or whether they are merely incidental to its business. If these are
the principal objectives, the business is that of insurance. But if they are
merely incidental and service is the principal purpose, then the business is not
insurance.

Philamcare v. CA

Held: No. Petition dismissed.
Ratio: Petitioner claimed that it granted benefits only when the insured is alive during
the one-year duration. It contended that there was no indemnification unlike in
insurance contracts. It supported this claim by saying that it is a health maintenance
organization covered by the DOH and not the Insurance Commission. Lastly, it claimed
that the Incontestability clause didn’t apply because two-year and not one-year
effectivity periods were required.
Section 2 (1) of the Insurance Code defines a contract of insurance as “an agreement
whereby one undertakes for a consideration to indemnify another against loss, damage
or liability arising from an unknown or contingent event.”
Section 3 states: every person has an insurable interest in the life and health:
(1) of himself, of his spouse and of his children.
In this case, the husband’s health was the insurable interest. The health care agreement
was in the nature of non-life insurance, which is primarily a contract of indemnity. The
provider must pay for the medical expenses resulting from sickness or injury.
While petitioner contended that the husband concealed material fact of his sickness,
the contract stated that: “that any physician is, by these presents, expressly authorized
to disclose or give testimony at anytime relative to any information acquired by him in
his professional capacity upon any question affecting the eligibility for health care
coverage of the Proposed Members.”
This meant that the petitioners required him to sign authorization to furnish reports
about his medical condition. The contract also authorized Philam to inquire directly to
his medical history.

Philamcare v CA G.R. No. 125678. March 18, 2002

Hence, the contention of concealment isn’t valid.

J. Ynares-Santiago

They can’t also invoke the “Invalidation of agreement” clause where failure of the
insured to disclose information was a grounds for revocation simply because the answer
assailed by the company was the heart condition question based on the insured’s
opinion. He wasn’t a medical doctor, so he can’t accurately gauge his condition.

Facts:
Ernani Trinos applied for a health care coverage with Philam. He answered no to a
question asking if he or his family members were treated to heart trouble, asthma,
diabetes, etc.
The application was approved for 1 year. He was also given hospitalization benefits and
out-patient benefits. After the period expired, he was given an expanded coverage for
Php 75,000. During the period, he suffered from heart attack and was confined at MMC.
The wife tried to claim the benefits but the petitioner denied it saying that he
concealed his medical history by answering no to the aforementioned question. She had
to pay for the hospital bills amounting to 76,000. Her husband subsequently passed
away. She filed a case in the trial court for the collection of the amount plus damages.
She was awarded 76,000 for the bills and 40,000 for damages. The CA affirmed but
deleted awards for damages. Hence, this appeal.
Issue: WON a health care agreement is not an insurance contract; hence the
“incontestability clause” under the Insurance Code does not apply.
2-S, 2013-2014

Henrick v Fire - “in such case the insurer is not justified in relying upon such statement,
but is obligated to make further inquiry.”
Fraudulent intent must be proven to rescind the contract. This was incumbent upon the
provider.
“Having assumed a responsibility under the agreement, petitioner is bound to answer
the same to the extent agreed upon. In the end, the liability of the health care
provider attaches once the member is hospitalized for the disease or injury covered by
the agreement or whenever he avails of the covered benefits which he has prepaid.”
Section 27 of the Insurance Code- “a concealment entitles the injured party to rescind a
contract of insurance.”
As to cancellation procedure- Cancellation requires certain conditions:
1. Prior notice of cancellation to insured;
6

Insurance Case Digests
Atty. Bathan-Basuel

2.
3.
4.

Notice must be based on the occurrence after effective date of the policy
of one or more of the grounds mentioned;
Must be in writing, mailed or delivered to the insured at the address
shown in the policy;
Must state the grounds relied upon provided in Section 64 of the
Insurance Code and upon request of insured, to furnish facts on which
cancellation is based

None were fulfilled by the provider.
As to incontestability- The trial court said that “under the title Claim procedures of
expenses, the defendant Philamcare Health Systems Inc. had twelve months from the
date of issuance of the Agreement within which to contest the membership of the
patient if he had previous ailment of asthma, and six months from the issuance of the
agreement if the patient was sick of diabetes or hypertension. The periods having
expired, the defense of concealment or misrepresentation no longer lie.”
Eternal v. Philamlife
ETERNAL VS. PHILAMLIFE
G.R. No. 166245
April 09, 2008
FACTS: Respondent Philamlife entered into an agreement denominated as Creditor
Group Life Policy with petitioner Eternal Gardens Memorial Park Corporation (Eternal).
Under the policy, the clients of Eternal who purchased burial lots from it on installment
basis would be insured by Philamlife. The amount of insurance coverage depended upon
the existing balance of the purchased burial lots.

purchasers, together with a copy of the application of each purchaser, and the amounts
of the respective unpaid balances of all insured lot purchasers. Eternal complied by
submitting a letter dated December 29, 1982, containing a list of insurable balances of
its lot buyers for October 1982. One of those included in the list as “new business” was
a certain John Chuang. His balance of payments was 100K. on August 2, 1984, Chuang
died.
Eternal sent a letter dated to Philamlife, which served as an insurance claim for
Chuang’s death. Attached to the claim were certain documents. In reply, Philamlife
wrote Eternal a letter requiring Eternal to submit the additional documents relative to
its insurance claim for Chuang’s death. Eternal transmitted the required documents
through a letter which was received by Philamlife.
After more than a year, Philamlife had not furnished Eternal with any reply to the
latter’s insurance claim. This prompted Eternal to demand from Philamlife the payment
of the claim for PhP 100,000.
In response to Eternal’s demand, Philamlife denied
Eternal’s insurance claim in a letter a portion of which reads:
The deceased was 59 years old when he entered into Contract #9558 and 9529 with
Eternal Gardens Memorial Park in October 1982 for the total maximum insurable amount
of P100,000.00 each. No application for Group Insurance was submitted in our
office prior to his death on August 2, 1984
Eternal filed a case with the RTC for a sum of money against Philamlife, which decided
in favor of Eternal, ordering Philamlife to pay the former 100K representing the
proceeds of the policy.
CA reversed. Hence this petition.
ISSUE: WON Philamlife should pay the 100K insurance proceeds

The relevant provisions of the policy are:
HELD: Petition granted.
ELIGIBILITY.
xx
EVIDENCE OF INSURABILITY.
xx
LIFE INSURANCE BENEFIT.
xx
EFFECTIVE DATE OF BENEFIT.
The insurance of any eligible Lot Purchaser shall be effective on the date
he contracts a loan with the Assured. However, there shall be no insurance
if the application of the Lot Purchaser is not approved by the Company.
xx

YES. An examination of the provision of the POLICY under effective date of benefit,
would show ambiguity between its two sentences. The first sentence appears to state
that the insurance coverage of the clients of Eternal already became effective upon
contracting a loan with Eternal while the second sentence appears to require Philamlife
to approve the insurance contract before the same can become effective.
It must be remembered that an insurance contract is a contract of adhesion which must
be construed liberally in favor of the insured and strictly against the insurer in order to
safeguard the latter’s interest


Eternal was required under the policy to submit to Philamlife a list of all new lot

2-S, 2013-2014

7

Insurance Case Digests
Atty. Bathan-Basuel

On the other hand, the seemingly conflicting provisions must be harmonized to mean
that upon a party’s purchase of a memorial lot on installment from Eternal, an insurance
contract covering the lot purchaser is created and the same is effective, valid, and
binding until terminated by Philamlife by disapproving the insurance application. The
second sentence of the Creditor Group Life Policy on the Effective Date of Benefit is in
the nature of a resolutory condition which would lead to the cessation of the insurance
contract. Moreover, the mere inaction of the insurer on the insurance application must
not work to prejudice the insured; it cannot be interpreted as a termination of the
insurance contract. The termination of the insurance contract by the insurer must be
explicit and unambiguous.
Filipinas v. Christern
FILIPINAS
COMPAÑIA
DE
vs.
CHRISTERN, HUENEFELD and CO., INC., respondent.
G.R. No. L-2294

SEGUROS, petitioner,

May 25, 1951

FACTS:
On October 1, 1941, the respondent corporation, Christern Huenefeld, & Co.,
Inc., after payment of corresponding premium, obtained from the petitioner ,Filipinas
Cia. de Seguros, fire policy No. 29333 in the sum of P1000,000, covering merchandise
contained in a building located at No. 711 Roman Street, Binondo Manila. On February
27, 1942, or during the Japanese military occupation, the building and insured
merchandise were burned.
Respondent then claimed under its policy with the petitioner. The salvage
goods were sold at public auction and, after deducting their value, the total loss
suffered by the respondent was fixed at P92,650. The petitioner refused to pay the
claim on the ground that the policy in favor of the respondent had ceased to be in force
on the date the United States declared war against Germany, the respondent
Corporation (though organized under and by virtue of the laws of the Philippines) being
controlled by the German subjects and the petitioner being a company under American
jurisdiction when said policy was issued on October 1, 1941. The petitioner, however, in
pursuance of the order of the Director of Bureau of Financing, Philippine Executive
Commission, dated April 9, 1943, paid to the respondent the sum of P92,650 on April
19, 1943.
After trial, the Court of First Instance of Manila dismissed the action filed by
the petitioner to recover the amount from respondent. Upon appeal to the Court of
Appeals, the judgment of the Court of First Instance of Manila was affirmed. Hence this
petition.
2-S, 2013-2014

ISSUE: Whether or not Filipinas Cia de Seguros can claim the amount it paid against
respondent.
HELD:
1)

In case of war, the control test applies to determine the nationality of
a corporation.

The Court of Appeals overruled the contention of the petitioner that the
respondent corporation became an enemy when the United States declared war against
Germany, relying on English and American cases which held that a corporation is a
citizen of the country or state by and under the laws of which it was created or
organized. It rejected the theory that nationality of private corporation is determine by
the character or citizenship of its controlling stockholders.
There is no question that majority of the stockholders of the respondent corporation
were German subjects. This being so, we have to rule that said respondent became an
enemy corporation upon the outbreak of the war between the United States and
Germany. The English and American cases relied upon by the Court of Appeals have lost
their force in view of the latest decision of the Supreme Court of the United States in
Clark vs. Uebersee Finanz Korporation, decided on December 8, 1947, 92 Law. Ed.
Advance Opinions, No. 4, pp. 148-153, in which the controls test has been adopted.
This pronouncement was based on the situation that arise during the WWI and WWII.
2)

The Philippine Insurance
against a public enemy.

Law

prohibits

the

granting

of

insurance

The Philippine Insurance Law (Act No. 2427, as amended,) in section 8, provides that
"anyone except a public enemy may be insured." It stands to reason that an insurance
policy ceases to be allowable as soon as an insured becomes a public enemy.

Effect of war, generally. — All intercourse between citizens of belligerent
powers which is inconsistent with a state of war is prohibited by the law of
nations. Such prohibition includes all negotiations, commerce, or trading with
the enemy; all acts which will increase, or tend to increase, its income or
resources; all acts of voluntary submission to it; or receiving its protection;
also all acts concerning the transmission of money or goods; and all contracts
relating thereto are thereby nullified. It further prohibits insurance upon trade
with or by the enemy, upon the life or lives of aliens engaged in service with
the enemy; this for the reason that the subjects of one country cannot be
permitted to lend their assistance to protect by insurance the commerce or
property of belligerent, alien subjects, or to do anything detrimental too their
country's interest. The purpose of war is to cripple the power and exhaust the
resources of the enemy, and it is inconsistent that one country should destroy
its enemy's property and repay in insurance the value of what has been so
8

Insurance Case Digests
Atty. Bathan-Basuel

destroyed, or that it should in such manner increase the resources of the
enemy, or render it aid, and the commencement of war determines, for like
reasons, all trading intercourse with the enemy, which prior thereto may have
been lawful. All individuals therefore, who compose the belligerent powers,
exist, as to each other, in a state of utter exclusion, and are public enemies. (6
Couch, Cyc. of Ins. Law, pp. 5352-5353.)
In the case of an ordinary fire policy, which grants insurance only from year, or
for some other specified term it is plain that when the parties become alien
enemies, the contractual tie is broken and the contractual rights of the
parties, so far as not vested. lost. (Vance, the Law on Insurance, Sec. 44, p.
112.)
The respondent having become an enemy corporation on December 10, 1941, the
insurance policy issued in its favor on October 1, 1941, by the petitioner (a Philippine
corporation) had ceased to be valid and enforcible, and since the insured goods were
burned after December 10, 1941, and during the war, the respondent was not entitled
to any indemnity under said policy from the petitioner. However, elementary rules of
justice (in the absence of specific provision in the Insurance Law) require that the
premium paid by the respondent for the period covered by its policy from December
11, 1941, should be returned by the petitioner.

-

-

-

Both policies contained this provision: All premiums are due in advance and
any unpunctuality in making such payment shall cause this policy to lapse
unless and except as kept in force by the grace period condition.
Paz Constantino and Agustina Peralta claim as beneficiaries, that they are
entitled to receive the proceeds of the policies less all sums due for premiums
in arrears. They also allege that non-payment of the premiums were caused
by the closing of ALIC’s offices during the war and the impossible
circumstances by the war, therefore, they should be excused and the policies
should not be forfeited.
Lower court ruled in favor of ALIC.

Issue:
May a beneficiary in a life insurance policy recover the amount thereof although the
insured died after repeatedly failing to pay the stipulated premiums, such failure being
caused by war?
Held:
NO. Due to the express terms of the policy, non-payment of the premium produces its
avoidance. In Glaraga v. Sun Life, it was held that a life policy was avoided because the
premium had not been paid within the time fixed; since by its express terms, nonpayment of any premium when due or within the 31 day grace period ipso fact caused
the policy to lapse.

Constantino v. Asia Life
Facts:
-

-

-

Appeal consolidates two cases.
Asia life insurance Company (ALIC) was incorporated in Delaware.
For the sum of 175.04 as annual premium duly paid to ALIC, it issued Policy
No. 93912 whereby it insured the life of Arcadio Constantino for 20 years for
P3T with Paz Constantino as beneficiary.
First premium covered the period up to Sept. 26, 1942. No further premiums
were paid after the first premium and Arcadio died on Sept. 22, 1944.
Due to Jap occupation, ALIC closed its branch office in Manila from Jan. 2
1942-1945.
On Aug. 1, 1938, ALIC issued Policy no. 78145 covering the lives of Spouses
Tomas Ruiz and Agustina Peralta for the sum of P3T for 20 years. The annual
premium stipulated was regularly paid from Aug. 1, 1938 up to and including
Sept. 30, 1940.
Effective Aug. 1, 1941, the mode of payment was changed from annually to
quarterly and such quarterly premiums were paid until Nov. 18, 1941.
Last payment covered the period until Jan. 31, 1942.
Tomas Ruiz died on Feb. 16, 1945 with Agustina Peralta as his beneficiary.
Due to Jap occupation, it became impossible and illegal for the insured to deal
with ALIC. Aside from this the insured borrowed from the policy P234.00
such that the cash surrender value of the policy was sufficient to maintain the
policy in force only up to Sept. 7, 1942.

2-S, 2013-2014

When the life insurance policy provides that non-payment of premiums will cause its
forfeiture,
war
does
NOT
excuse
non-payment
and
does
not
avoid
forfeiture. Essentially, the reason why punctual payments are important is that the
insurer calculates on the basis of the prompt payments. Otherwise, malulugi sila.
It should be noted that the parties contracted not only as to peace time conditions but
also as to war-time conditions since the policies contained provisions applicable
expressly to wartime days. The logical inference therefore is that the parties
contemplated the uninterrupted operation of the contract even if armed conflict should
ensue.
Great Pacific v. CA
FACTS:
Great Pacific Life Assurance (Grepalife) assailed the decision of the CA, which affirmed
the decision of the RTC (Misamis Oriental) when it ruled that Grepalife should pay
Development Bank of the Philippines (DBP) as creditor of the insured Dr. Wilfredo
Leuterio (Php 86,200.00).
A contract of GROUP LIFE INSURANCE was executed between Grepalife and DBP.
Grepalife agreed to insure the lives of ELIGIBLE HOUSING LOAN MORTGAGORS of DBP.
9

Insurance Case Digests
Atty. Bathan-Basuel

Dr. Wilfredo Leutorio applied for membership and answered the question concerning his
health conditions:

“7. Have you ever had, or consulted, a physician for a heart condition, high
blood pressure, cancer, diabetes, lung, kidney or stomach disorder or any
other physical impairment?

3.

Whether the Court of Appeals erred in holding Grepalife liable in the amount of
eighty six thousand, two hundred (P86, 200.00) pesos without proof of the actual
outstanding mortgage payable by the mortgagor to DBP. NO.

HELD:
Background:

Answer: No. If so give details ___________.
8. Are you now, to the best of your knowledge, in good health?
Answer: [ x ] Yes [

] No.”

Grepalife issued certificate insurance coverage of Dr. Leuterio, to the extent of his DBP
mortgage indebtedness amounting to P86, 200.00 pesos. Dr. Leuterio DIED due to
“massive cerebral hemorrhage.”
DBP submitted a death claim to Grepalife. Grepalife denied the claim alleging that Dr.
Leuterio was not physically healthy when he applied for an insurance coverage on
November 15, 1983. Grepalife insisted that Dr. Leuterio did not disclose he had been
suffering from hypertension, which caused his death. Allegedly, such non-disclosure
constituted concealment that justified the denial of the claim.
The widow of the late Dr. Leuterio, respondent Medarda V. Leuterio, filed a complaint
with the Regional Trial Court of Misamis Oriental, Branch 18, against Grepalife for
“Specific Performance with Damages.”
During the trial, Dr. Hernando Mejia, who issued the death certificate, was called to
testify. Dr. Mejia’s findings, based partly from the information given by the respondent
widow, stated that Dr. Leuterio complained of headaches presumably due to high blood
pressure. The inference was not conclusive because Dr. Leuterio was not autopsied,
hence, other causes were not ruled out.
Trial court rendered a decision in favor of respondent widow and against Grepalife. The
widow appealed to the CA but CA sustained trial court’s decision.
ISSUES:
1.

2.

Whether the Court of Appeals erred in holding petitioner liable to DBP as
beneficiary in a group life insurance contract from a complaint filed by the widow of
the decedent/mortgagor? NO.
Whether the Court of Appeals erred in not finding that Dr. Leuterio concealed that
he had hypertension, which would vitiate the insurance contract? NO.

2-S, 2013-2014

INSURABLE INTEREST IN MORTGAGED PROPERTIES & PARTIES TO THIS TYPE OF
CONTRACT. The rationale of a group insurance policy of mortgagors, otherwise
known as the “mortgage redemption insurance,” is a device for the protection of
both the mortgagee and the mortgagor.
a. On the part of the mortgagee, it has to enter into such form of contract so
that in the event of the unexpected demise of the mortgagor during the
subsistence of the mortgage contract, the proceeds from such insurance will
be applied to the payment of the mortgage debt, thereby relieving the heirs of
the mortgagor from paying the obligation.
b. Ample protection is given to the mortgagor under such a concept so that in
the event of death; the mortgage obligation will be extinguished by the
application of the insurance proceeds to the mortgage indebtedness.
Consequently, where the mortgagor pays the insurance premium under the group
insurance policy, making the loss payable to the mortgagee, the insurance is on the
mortgagor’s interest, and the mortgagor continues to be a party to the contract. In
this type of policy insurance, the mortgagee is simply an appointee of the insurance
fund, such loss-payable clause does not make the mortgagee a party to the contract.
Section 8 of the Insurance Code provides:

“Unless the policy provides, where a mortgagor of property effects insurance
in his own name providing that the loss shall be payable to the mortgagee, or
assigns a policy of insurance to a mortgagee, the insurance is deemed to be
upon the interest of the mortgagor, who does not cease to be a party to the
original contract, and any act of his, prior to the loss, which would otherwise
avoid the insurance, will have the same effect, although the property is in the
hands of the mortgagee, but any act which, under the contract of insurance, is
to be performed by the mortgagor, may be performed by the mortgagee
therein named, with the same effect as if it had been performed by the
mortgagor.”
1. PETITIONER’S CONTENTION: Petitioner alleges that the complaint was
instituted by the widow of Dr. Leuterio, not the real party in interest,
hence the trial court acquired no jurisdiction over the case.
The policy stating that: “In the event of the debtor’s death before his indebtedness
with the Creditor [DBP] shall have been fully paid, an amount to pay the outstanding
indebtedness shall first be paid to the creditor and the balance of sum assured, if there
10

Insurance Case Digests
Atty. Bathan-Basuel

is any, shall then be paid to the beneficiary/ies designated by the debtor.” And since a
policy of insurance upon life or health may pass by transfer, will or succession to any
person, whether he has an insurable interest or not, and such person may recover it
whatever the insured might have recovered, the widow of the decedent Dr. Leuterio
may file the suit against the insurer, Grepalife.

should not unjustly enrich itself at the expense of another (Nemo cum alterius
detrimenio protest). Hence, it cannot collect the insurance proceeds, after it already
foreclosed on the mortgage. The proceeds now rightly belong to Dr. Leuterio’s heirs
represented by his widow, herein private respondent Medarda Leuterio.
PETITION DENIED.

2. PETITIONER’S CONTENTION: Concealment exists where the assured had
knowledge of a fact material to the risk, and honesty, good faith, and fair
dealing requires that he should communicate it to the assured, but he
designedly and intentionally withholds the same.
On the contrary the medical findings were not conclusive because Dr. Mejia did not
conduct an autopsy on the body of the decedent. As the attending physician, Dr. Mejia
stated that he had no knowledge of Dr. Leuterio’s any previous hospital confinement.
Dr. Leuterio’s death certificate stated that hypertension was only “the possible cause of
death.” It was considered as hearsay. The fraudulent intent on the part of the insured
must be established to entitle the insurer to rescind the contract. Misrepresentation as
a defense of the insurer to avoid liability is an affirmative defense and the duty to
establish such defense by satisfactory and convincing evidence rests upon the insurer.
In the case at bar, the petitioner failed to clearly and satisfactorily establish its defense,
and is therefore liable to pay the proceeds of the insurance.
3. PETITIONER’S CONTENTION: Petitioner claims that there was no evidence
as to the amount of Dr. Leuterio’s outstanding indebtedness to DBP at the
time of the mortgagor’s death.
Petitioner’s claim is without merit. A life insurance policy is a valued policy. Unless the
interest of a person insured is susceptible of exact pecuniary measurement, the
measure of indemnity under a policy of insurance upon life or health is the sum fixed in
the policy. The mortgagor paid the premium according to the coverage of his insurance,
which states that:

“The policy states that upon receipt of due proof of the Debtor’s death during the
terms of this insurance, a death benefit in the amount of P86,200.00 shall be paid.
In the event of the debtor’s death before his indebtedness with the creditor shall have
been fully paid, an amount to pay the outstanding indebtedness shall first be paid to
the Creditor and the balance of the Sum Assured, if there is any shall then be paid to
the beneficiary/ies designated by the debtor.”
NOTE: (Supervening event) Court of Appeals’ decision was promulgated on May 17,
1993. In private respondent’s memorandum, she states that DBP foreclosed in 1995
their residential lot, in satisfaction of mortgagor’s outstanding loan. Considering this
supervening event, the insurance proceeds shall inure to the benefit of the
heirs of the deceased person or his beneficiaries. Equity dictates that DBP
2-S, 2013-2014

2.

Section 8, Additional Cases
Geagonia v. CA, G.R. No. 114427 February 6, 1995

FACTS:
The petitioner is the owner of Norman's Mart located in the public market of San
Francisco, Agusan del Sur. On 22 December 1989, he obtained from the private
respondent fire insurance policy for P100,000. The period of the policy was from 22
December 1989 to 22 December 1990 and covered the following: "Stock-in-trade
consisting principally of dry goods such as RTW's for men and women wear and other
usual to assured's business. The petitioner declared in the policy under the subheading
entitled CO-INSURANCE that Mercantile Insurance Co., Inc. was the co-insurer for
P50,000.00. From 1989 to 1990, the petitioner had in his inventory stocks amounting
to P392,130.50.
The policy contained the following condition:

3. The insured shall give notice to the Company of any insurance or insurances already
affected, or which may subsequently be effected, covering any of the property or
properties consisting of stocks in trade, goods in process and/or inventories only
hereby insured, and unless such notice be given and the particulars of such insurance or
insurances be stated therein or endorsed in this policy pursuant to Section 50 of the
Insurance Code, by or on behalf of the Company before the occurrence of any loss or
damage, all benefits under this policy shall be deemed forfeited, provided however, that
this condition shall not apply when the total insurance or insurances in force at the time
of the loss or damage is not more than P200,000.00.
On 27 May 1990, fire of accidental origin broke out at around 7:30 p.m. at the public
market of San Francisco, Agusan del Sur. The petitioner's insured stock-in-trade were
completely destroyed prompting him to file with the private respondent a claim under
the policy. On 28 December 1990, the private respondent denied the claim because it
found that at the time of the loss the petitioner's stocks-in-trade were likewise covered
by fire insurance policies No. GA-28146 and No. GA-28144, for P100,000.00 each,
issued by the Cebu Branch of the Philippines First Insurance Co., Inc. (hereinafter PFIC).
3
These policies indicate that the insured was "Messrs. Discount Mart (Mr. Armando
Geagonia, Prop.)" with a mortgage clause reading:
MORTGAGE: Loss, if any shall be payable to Messrs. Cebu Tesing Textiles, Cebu City as
their interest may appear subject to the terms of this policy. CO-INSURANCE DECLARED:
11

Insurance Case Digests
Atty. Bathan-Basuel

P100,000. — Phils. First CEB/F 24758. 4
The basis of the private respondent's denial was the petitioner's alleged violation of
Condition 3 of the policy.
PETITIONER’S CONTENTION: at the time he obtained the private respondent's fire
insurance policy he knew that the two policies issued by the PFIC were already in
existence; however, he had no knowledge of the provision in the private respondent's
policy requiring him to inform it of the prior policies; this requirement was not
mentioned to him by the private respondent's agent; and had it been mentioned, he
would not have withheld such information.
RESPONDENT’S CONTENTION: denied the allegations in the complaint and set up as its
principal defense the violation of Condition 3 of the policy.
INSURANCE COMMISSION: petitioner did not violate Condition 3 as he had no knowledge
of the existence of the two fire insurance policies obtained from the PFIC.
CA: reversed the decision of I.C.
ISSUES:
(a) whether the petitioner had prior knowledge of the two insurance policies issued by
the PFIC when he obtained the fire insurance policy from the private respondent,
thereby, for not disclosing such fact, violating Condition 3 of the policy, and [YES]
(b) if he had, whether he is precluded from recovering therefrom. [NO]
HELD: We agree with the Court of Appeals that the petitioner knew of the prior policies
issued by the PFIC. His letter of 18 January 1991 to the private respondent
conclusively proves this knowledge. His testimony to the contrary before the Insurance
Commissioner and which the latter relied upon cannot prevail over a written admission
made ante litem motam. It was, indeed, incredible that he did not know about the prior
policies since these policies were not new or original. Policy No. GA-28144 was a
renewal of Policy No. F-24758, while Policy No. GA-28146 had been renewed twice, the
previous policy being F-24792.
Condition 3 of the private respondent's Policy No. F-14622 is a condition which is not
proscribed by law. Its incorporation in the policy is allowed by Section 75 of the
Insurance Code which provides:

"[a] policy may declare that a violation of specified provisions thereof shall avoid it,
otherwise the breach of an immaterial provision does not avoid the policy." Such a
condition is a provision which invariably appears in fire insurance policies and is intended
to prevent an increase in the moral hazard. It is commonly known as the additional or
"other insurance" clause and has been upheld as valid and as a warranty that no other
insurance exists. Its violation would thus avoid the 
policy. However, in order to
constitute a violation, the other insurance must be upon same subject matter, the same
interest therein, and the same risk.”
2-S, 2013-2014

As to a mortgaged property, the mortgagor and the mortgagee have each an
independent insurable interest therein and both interests may be one policy, or each
may take out a separate policy covering his interest, either at the same or at separate
times. A mortgagor may, however, take out insurance for the benefit of the mortgagee,
which is the usual practice. The mortgagee may be made the beneficial payee in several
ways. He may become the assignee of the policy with the consent of the insurer; or the
mere pledgee without such consent; or the original policy may contain a mortgage
clause; or a rider making the policy payable to the mortgagee "as his interest may
appear" may be attached; or a "standard mortgage clause," containing a collateral
independent contract between the mortgagee and insurer, may be attached; or the
policy, though by its terms payable absolutely to the mortgagor, may have been
procured by a mortgagor under a contract duty to insure for the mortgagee's benefit,
in which case the mortgagee acquires an equitable lien upon the proceeds.
It is a cardinal rule on insurance that a policy or insurance contract is to be interpreted
liberally in favor of the insured and strictly against the company, the reason being,
undoubtedly, to afford the greatest protection which the insured was endeavoring to
secure when he applied for insurance. It is also a cardinal principle of law that forfeitures
are not favored and that any construction which would result in the forfeiture of the
policy benefits for the person claiming thereunder, will be avoided, if it is possible to
construe the policy in a manner which would permit recovery, as, for example, by
finding a waiver for such forfeiture.
With these principles in mind, we are of the opinion that Condition 3 of the subject
policy is not totally free from ambiguity and must, perforce, be meticulously analyzed.
Such analysis leads us to conclude that (a) the prohibition applies only to double
insurance, and (b) the nullity of the policy shall only be to the extent exceeding
P200,000.00 of the total policies obtained.
The first conclusion is supported by the portion of the condition referring to other
insurance "covering any of the property or properties consisting of stocks in trade,
goods in process and/or inventories only hereby insured," and the portion regarding the
insured's declaration on the subheading CO-INSURANCE that the co-insurer is Mercantile
Insurance Co., Inc. in the sum of P50,000.00.
A double insurance exists where the same person is insured by several insurers
separately in respect of the same subject and interest. As earlier stated, the insurable
interests of a mortgagor and a mortgagee on the mortgaged property are distinct and
separate. Since the two policies of the PFIC do not cover the same interest as that
covered by the policy of the private respondent, no double insurance exists. The nondisclosure then of the former policies was not fatal to the petitioner's right to recover
on the private respondent's policy.
When a property owner obtains insurance policies from two or more insurers in a total
amount that exceeds the property's value, the insured may have an inducement to
destroy the property for the purpose of collecting the insurance. The public as well as
12

Insurance Case Digests
Atty. Bathan-Basuel

the insurer is interested in preventing a situation in which a fire would be profitable to
the insured.
PETITION GRANTED. CA DECISION SET ASIDE.
Palileo v. Cosio, G.R. No. L-7667, November 28, 1955
Cherie Palileo vs Beatriz Cosio
G.R. No. L-7667
November 28, 1955
J. Bautista Angelo
FACTS: Plaintiff Cherie Palileo obtained from defendant Beatriz Cosio a loan of P12,000
with these conditions: (a) that plaintiff shall pay to defendant an interest in the amount
of P250 a month; (b) that defendant shall deduct from the loan certain obligations of
plaintiff to third persons amounting to P4,550, plus the sum of P250 as interest for the
first month; and (c) that after making the above deductions, defendant shall deliver to
plaintiff only the balance of the loan of P12,000.
Cherie paid Beatriz a sum of P2,250 interest corresponding for 9 months, which is more
than the maximum interest authorized by law. To secure the payment of the loan,
defendant Beatriz required plaintiff to sign a document known as "Conditional Sale of
Residential Building", purporting to convey to defendant, with right to repurchase, a
two-story building of strong materials belonging to plaintiff Cherrie. This document did
not express the true intention of the parties which was merely to place said property as
security for the payment of the loan.

HELD: No. The rule is that "where a mortgagee, independently of the mortgagor,
insures the mortgaged property in his own name and for his own interest, he is entitled
to the insurance proceeds in case of loss, but in such case, he is not allowed to retain
his claim against the mortgagor, but is passed by subrogation to the insurer to the
extent of the money paid." (Vance on Insurance, 2d ed., p. 654) Or, stated in another
way, "the mortgagee may insure his interest in the property independently of the
mortgagor. In that event, upon the destruction of the property the insurance money
paid to the mortgagee will not inure to the benefit of the mortgagor, and the amount
due under the mortgage debt remains unchanged. The mortgagee, however, is not
allowed to retain his claim against the mortgagor, but it passes by subrogation to the
insurer, to the extent of the insurance money paid." "The general rule and the weight of
authority is, that the insurer is thereupon subrogated to the rights of the mortgagee
under the mortgage. This is put upon the analogy of the situation of the insurer to that
of a surety."
Consistent with the foregoing pronouncement, we therefore modify the judgment of
the lower court as follows: (1) the transaction had between the plaintiff and defendant
as shown in Exhibit A is merely an equitable mortgage intended to secure the payment
of the loan of P12,000; (2) that the proceeds of the insurance amounting to
P13,107.00 was properly collected by defendant who is not required to account for it
to the plaintiff; (3) that the collection of said insurance proceeds shall not be deemed
to have compensated the obligation of the plaintiff to the defendant, but bars the latter
from claiming its payment from the former; and (4) defendant shall pay to the plaintiff
the sum of P810.00 representing the overpayment made by plaintiff by way of interest
on the loan. No pronouncement as to costs.
3.

After the execution of the aforesaid document, defendant insured the building against
fire with the Associated Insurance & Surety Co., Inc. for the sum of P15,000, the
insurance policy having been issued in the name of defendant Beatriz. When the building
was partly destroyed by fire, defendant Beatriz collected from the insurance company
an indemnity of P13,107.00. Plaintiff Cherrie asked defendant that she be credited with
the necessary amount to pay her obligation out of the insurance proceeds but
defendant refused to do so.
The lower court ruled in favor of plaintiff Cherrie, declaring the transaction as one of
equitable mortgage to secure the 12,000-peso loan and ordering the defendant to
credit the sum of P13,107 received by the defendant from the Associated Insurance &
Surety Co., Inc. to the payment of plaintiff's obligation in the sum of P12,000.00 and
lastly to return the overpaid interest in the sum of P810 and the balance of 1,107
pesos (difference between the amount of loan and the proceeds of the insurance).
ISSUE: Whether the trial court is correct in in considering the obligation of plaintiff
fully compensated by the insurance amount and in ordering defendant to refund to
plaintiff the sum of P1,107 notwithstanding the fact that it was not proven that the
insurance was taken for the benefit of the mortgagor
2-S, 2013-2014

Sections 10-25
Insular Life Assurance Co., Ltd vs Ebrado, 80 SCRA 181

G.R. No. L-44059 October 28, 1977
THE INSULAR LIFE ASSURANCE COMPANY, LTD., plaintiff-appellee,
vs.
CARPONIA T. EBRADO and PASCUALA VDA. DE EBRADO, defendants-appellants.
MARTIN, J .:
FACTS:
1) On September 1, 1968, Buenaventura Cristor Ebrado was issued by The Life
Assurance Co., Ltd., Policy No. 009929 on a whole-life for P5,882.00 with a, rider
for Accidental Death for the same amount. Buenaventura C. Ebrado designated T.
Ebrado as the revocable beneficiary in his policy. He to her as his wife.
2)

On October 21, 1969, Buenaventura C. Ebrado died as a result of an t when he was
hit by a failing branch of a tree.

13

Insurance Case Digests
Atty. Bathan-Basuel

3)

As the policy was in force, The Insular Life Assurance Co., Ltd. Liable to pay the
coverage in the total amount of P11,745.73, representing the face value of the
policy in the amount of P5,882.00 plus the additional benefits for accidental death
also in the amount of P5,882.00 and the refund of P18.00 paid for the premium
due November, 1969, minus the unpaid premiums and interest thereon due for
January and February, 1969, in the sum of P36.27.

4)

Carponia T. Ebrado filed with the insurer a claim for the proceeds of the Policy as
the designated beneficiary therein, although she admits that she and the insured
Buenaventura C. Ebrado were merely living as husband and wife without the benefit
of marriage.

5)

Pascuala Vda. de Ebrado also filed her claim as the widow of the deceased insured.
She asserts that she is the one entitled to the insurance proceeds, not the
common-law wife, Carponia T. Ebrado.

6)

In doubt as to whom the insurance proceeds shall be paid, the insurer, The Insular
Life Assurance Co., Ltd. commenced an action for Interpleader before the Court of
First Instance of Rizal on April 29, 1970.

7)

After the issues have been joined, a pre-trial conference was held on July 8, 1972.
During this conference, parties Carponia T. Ebrado and Pascuala Ebrado agreed and
stipulated among others that:
a. that the deceased Buenaventura Ebrado was married to Pascuala Ebrado with
whom she has six — (legitimate) namely; Hernando, Cresencio, Elsa, Erlinda,
Felizardo and Helen, all surnamed Ebrado;
b. that during the lifetime of Buenaventura Ebrado, he was living with his
common-wife, Carponia Ebrado, with whom she had 2 children although he was
not legally separated from his legal wife;

"insured" and not to the beneficiary, since a contract of insurance is
personal in character. Otherwise, the prohibitory laws against illicit relationships
especially on property and descent will be rendered nugatory, as the same could easily
be circumvented by modes of insurance. Rather, the general rules of civil law
should be applied to resolve this void in the Insurance Law. Article 2011 of
the New Civil Code states: "The contract of insurance is governed by special laws.
Matters not expressly provided for in such special laws shall be regulated by this Code."
When not otherwise specifically provided for by the Insurance Law, the contract of life
insurance is governed by the general rules of the civil law regulating contracts. And
under Article 2012 of the same Code, "any person who is forbidden from receiving any
donation under Article 739 cannot be named beneficiary of a life insurance policy by the
person who cannot make a donation to him. Common-law spouses are, definitely, barred
from receiving donations from each other. Article 739 of the new Civil Code provides:
The following donations shall be void:
1. Those made between persons who were guilty of adultery or
concubinage at the time of donation;
2. Those made between persons found guilty of the same criminal
offense, in consideration thereof;
3. Those made to a public officer or his wife, descendants or
ascendants by reason of his office.
In the case referred to in No. 1, the action for declaration of nullity may be brought by
the spouse of the donor or donee; and the guilt of the donee may be proved by
preponderance of evidence in the same action.

The trial court rendered judgment declaring among others, Carponia T. Ebrado
disqualified from becoming beneficiary of the insured Buenaventura Cristor Ebrado
and directing the payment of the insurance proceeds to the estate of the deceased
insured. From this judgment, Carponia T. Ebrado appealed to the Court of Appeals,
but the Appellate Court certified the case to the Supreme Court as involving only
questions of law.

In essence, a life insurance policy is no different from a civil donation
insofar as the beneficiary is concerned. Both are founded upon the same
consideration: liberality. A beneficiary is like a donee, because from the
premiums of the policy which the insured pays out of liberality, the
beneficiary will receive the proceeds or profits of said insurance. As a
consequence, the proscription in Article 739 of the new Civil Code should
equally operate in life insurance contracts. The mandate of Article 2012 cannot
be laid aside: any person who cannot receive a donation cannot be named as beneficiary
in the life insurance policy of the person who cannot make the donation. Under
American law, a policy of life insurance is considered as a testament and in construing
it, the courts will, so far as possible treat it as a will and determine the effect of a
clause designating the beneficiary by rules under which wins are interpreted.

ISSUE: Whether a common-law wife named as beneficiary in the life insurance policy of a
legally married man can claim the proceeds thereof in case of death of the latter

Southern Luzon Employee’s Association v. Golpeo, G.R. No. L-6114, October 30,
1954

8)

HELD: No. It is quite unfortunate that the Insurance Act (RA 2327, as amended) or
even the new Insurance Code (PD No. 612, as amended) does not contain any specific
provision grossly resolutory of the prime question at hand. Section 50 of the
Insurance Act which provides that "(t)he insurance shall be applied
exclusively to the proper interest of the person in whose name it is made"
cannot be validly seized upon to hold that the mm includes the beneficiary.
The word "interest" highly suggests that the provision refers only to the
2-S, 2013-2014

Facts: The plaintiff, Southern Luzon Employees' Association is composed of laborers
and employees of Laguna tayabas Bus Co., and Batangas Transportation Company, and
one of its purposes is mutual aid of its members and their defendants in case of death.
Roman A. Concepcion was a mem

14

Insurance Case Digests
Atty. Bathan-Basuel

In the form required by the association to be accomplished by its members, with
reference to the death benefit, Roman A. Concepcion listed as his beneficiaries Aquilina
Maloles, Roman M. Concepcion, Jr., Estela M. Concepcion, Rolando M. Concepcion and
Robin M. Concepcion.
After the death of Roman A. Concepcion, the association was able to collect voluntary
contributions from its members amounting to P2,5055. Three sets of claimants
presented themselves, namely, (1) Juanita Golpeo, legal wife of Roman A. Concepcion,
and her children, Aquilina Maloles, common law wife of Roman, and her children, named
beneficiaries by the deceased; and (3) Elsie Hicban, another common law wife of Roman
A. Concepcion, and her child.

cannot make any donation to him, according to said article." Inasmuch as, according to
article 739 of the new Civil Code, a donation is valid when made "between persons who
are guilty or adultery or concubinage at the time of the donation," it is alleged that the
defendant-appellee Aquilina Maloles, cannot be named a beneficiary, every assuming
that the insurance law is applicable. Without considering the intimation in the brief for
the defendant appellees that appellant Juanita Golpeo, by her silence and actions, had
acquiesced in the illicit relations between her husband and appellee Aquilina Maloles,
appellant argument would certainly not apply to the children of Aquilina likewise named
beneficiaries by the deceased Roman A. Concepcion. As a matter of a fact the new Civil
Code recognized certain successional rights of illegitimate children. (Article 287.)
Gercio v. Sun Life Insurance of Canada, G.R. No. 23703 September 28, 1925

After hearing, the court rendered a decision, declaring the defendants Aquilina Maloles
and her children the sole beneficiaries of the sum of P2,505.00, and ordering the
plaintiff to deliver said amount to them.
Issue: whether the court erred in designating a common law wife of an insured as the
beneficiary instead of the legal wife?
Held: no. judgement was affirmed!
The decision is based mainly on the theory that the contract between the plaintiff and
the deceased Roman A. Concepcion partook of the nature of an insurance and that,
therefore, the amount in question belonged exclusively to the beneficiaries, invoking the
following pronouncements of this Court in the case of Del Val vs. Del Val, 29 Phil., 534:
With the finding of the trial court that the proceeds of the life-insurance policy
belongs exclusively to the defendant as his individual and separate property,
we agree. That the proceeds of an insurance policy belong exclusively to the
beneficiary and not to the estate of the person whose life was insured, and
that such proceeds are the separate and individual property of the beneficiary,
and not of the heirs of the person whose life was insured, is the doctrine in
America. We believe that the same doctrine obtains in these Islands by virtue
of section 428 of the Code of Commerce, which reads:
"The amounts which the underwriter must deliver to the person insured, in
fulfillment of the contract, shall be the property creditors of any kind
whatsoever of the person who effected the insurance in favor of the formers."
Appellant also contend that the stipulation between the plaintiff and the deceased
Roman A. Concepcion regarding the specification of the latter's beneficiaries, and the
resolution of September 17, 1949, are void for the being contrary to law, moral or
public policy. Specifically, the appellants cite article 2012 of the new Civil Code
providing that "Any person who is forbidden from receiving any donation under article
739 cannot be named beneficiary of a life insurance policy and by the person who
2-S, 2013-2014

FACTS: On January 29, 1910, the defendant Sun Life Assurance Co. of Canada issued
insurance policy No. 161481 on the life of Hilario Gercio. By its terms, the insurance
company agreed to insure the life of Hilario Gercio for the sum of P2,000, to be paid
him on February 1, 1930, or if the insured should die before said date, then to his wife,
Mrs. Andrea Zialcita, should she survive him; otherwise to the executors, administrators,
or assigns of the insured. The policy did not include any provision reserving to the
insured the right to change the beneficiary.
On the date the policy was issued, Andrea Zialcita was the lawful wife of Hilario Gercio.
Towards the end of the year 1919, she was convicted of the crime of adultery. On
September 4, 1920, a decree of divorce was issued in civil case no. 17955, which had
the effect of completely dissolving the bonds of matrimony contracted by Hilario Gercio
and Andrea Zialcita.
On March 4, 1922, Hilario Gercio formally notified the Sun Life Assurance Co. of Canada
that he had revoked his donation in favor of Andrea Zialcita, and that he had designated
in her stead his present wife, Adela Garcia de Gercio, as the beneficiary of the policy.
Gercio requested the insurance company to eliminate Andrea Zialcita as beneficiary,
which the insurance company has refused and still refuses to do.
ISSUE: Whether the insured — plaintiff Gercio — has the power/right to change the
beneficiary — his former wife — and to name instead his actual wife, where the insured
and the beneficiary have been divorced and where the policy of insurance does not
expressly reserve to the insured the right to change the beneficiary?
Held: NO, the wife has an insurable interest in the life of her husband. The beneficiary
has an absolute vested interest in the policy from the date of its issuance and delivery.
So when a policy of life insurance is taken out by the husband in which the wife is
named as beneficiary, she has a subsisting interest in the policy. And this applies to a
policy to which there are attached the incidents of a loan value, cash surrender value,
an automatic extension by premiums paid, and to an endowment policy, as well as to an
ordinary life insurance policy. If the husband wishes to retain to himself the control and
15

Insurance Case Digests
Atty. Bathan-Basuel

ownership of the policy he may so provide in the policy. But if the policy contains no
provision authorizing a change of beneficiary without the beneficiary's consent, the
insured cannot make such change. Accordingly, it is held that a life insurance policy of a
husband made payable to the wife as beneficiary, is the separate property of the
beneficiary and beyond the control of the husband.
As to the effect produced by the divorce, the Philippine Divorce Law, Act No. 2710,
merely provides in section 9 that the decree of divorce shall dissolve the community
property as soon as such decree becomes final. Unlike the statutes of a few
jurisdictions, there is no provision in the Philippine Law permitting the beneficiary in a
policy for the benefit of the wife of the husband to be changed after a divorce. It must
follow, therefore, in the absence of a statute to the contrary, that if a policy is taken
out upon a husband's life the wife is named as beneficiary therein, a subsequent divorce
does not destroy her rights under the policy.
Since it is said that our Insurance Act is mostly taken from the statute of California, it
should prove of interest to know the stand taken by the Supreme Court of that State. A
California decision oft cited in the Cyclopedias isYore vs. Booth, in which we find the
following:
. . . It seems to be the settled doctrine, with but slight dissent in the courts of
this country, that a person who procures a policy upon his own life, payable to
a designated beneficiary, although he pays the premiums himself, and keeps
the policy in his exclusive possession, has no power to change the beneficiary,
unless the policy itself, or the charter of the insurance company, so provides.
In policy, although he has parted with nothing, and is simply the object of
another's bounty, has acquired a vested and irrevocable interest in the policy,
which he may keep alive for his own benefit by paying the premiums or
assessments if the person who effected the insurance fails or refuses to do so.
Another controlling decision of the United States Supreme Court is that of the Central
National Bank of Washington City vs. Hume ([1888], 128 U.S., 134). Therein, Mr. Chief
Justice Fuller, as the organ of the court, announced the following doctrines:
We think it cannot be doubted that in the instance of contracts of insurance
with a wife or children, or both, upon their insurable interest in the life of the
husband or father, the latter, while they are living, can exercise no power of
disposition over the same without their consent, nor has he any interest
therein of which he can avail himself; nor upon his death have his personal
representatives or his creditors any interest in the proceeds of such contracts,
which belong to the beneficiaries to whom they are payable.
It is indeed the general rule that a policy, and the money to become due under
it, belong, the moment it is issued, to the person or persons named in it as the
beneficiary or beneficiaries, and that there is no power in the person procuring
2-S, 2013-2014

the insurance, by any act of his, by deed or by will, to transfer to any other
person the interest of the person named.
Vda. de Consuegra v. GSIS 37 SCRA 315 (1971)

VDA. DE CONSUEGRA vs.
G OVERNMENT SERVICE INSURANCE SYSTEM
FACTS:
Jose Consuegra, at the time of his death, was employed as a shop foreman of the office
of the District Engineer in the province of Surigao del Norte. In his lifetime, Consuegra
contracted two marriages, the first with herein respondent Rosario Diaz,; and the
second, which was contracted in good faith while the first marriage was subsisting, with
herein petitioner Basilia Berdin, out of which marriage were born seven children
Being a member of the Government Service Insurance System (GSIS, for short) when
Consuegra died on September 26, 1965, the proceeds of his life insurance were paid by
the GSIS to petitioner Basilia Berdin and her children who were the beneficiaries named
in the policy. Having been in the service of the government for 22.5028 years,
Consuegra was entitled to retirement insurance benefits in the sum of P6,304.47.
Consuegra did not designate any beneficiary who would receive the retirement
insurance benefits due to him.
Respondent Rosario Diaz, the widow by the first marriage, filed a claim with the GSIS
asking that the retirement insurance benefits be paid to her as the only legal heir of
Consuegra, considering that the deceased did not designate any beneficiary with
respect to his retirement insurance benefits. Petitioner Basilia Berdin and her children,
likewise, filed a similar claim with the GSIS, asserting that being the beneficiaries named
in the life insurance policy of Consuegra, they are the only ones entitled to receive the
retirement insurance benefits due the deceased Consuegra. Resolving the conflicting
claims, the GSIS ruled that the legal heirs of the late Jose Consuegra were Rosario Diaz,
his widow by his first marriage who is entitled to one-half, or 8/16, of the retirement
insurance benefits, on the one hand; and Basilia Berdin, his widow by the second
marriage and their seven children, on the other hand, who are entitled to the remaining
one-half, or 8/16, each of them to receive an equal share of 1/16.

It is the submission of appellants that because the deceased Jose Consuegra failed to
designate the beneficiaries in his retirement insurance, the appellants who were the
beneficiaries named in the life insurance should automatically be considered the
beneficiaries to receive the retirement insurance benefits, to the exclusion of
respondent Rosario Diaz.

ISSUE:
16

Insurance Case Digests
Atty. Bathan-Basuel

To whom should the retirement insurance benefits of Jose Consuegra be paid, because
he did not, or failed to, designate the beneficiary of his retirement insurance?
HELD:
When Consuegra designated his beneficiaries in his life insurance he could not have
intended those beneficiaries of his life insurance as also the beneficiaries of his
retirement insurance because the provisions on retirement insurance under the GSIS
came about only when Com. Act 186 was amended by Rep. Act 660 on June 16, 1951.
Hence, it cannot be said that because herein appellants were designated beneficiaries in
Consuegra's life insurance they automatically became the beneficiaries also of his
retirement insurance.
Section 11 of Commonwealth Act 186, as amended by Rep. Act 660, clearly indicate
that there is need for the employee to file an application for retirement insurance
benefits when he becomes a member of the GSIS, and he should state in his application
the beneficiary of his retirement insurance. Hence, the beneficiary named in the life
insurance does not automatically become the beneficiary in the retirement insurance
unless the same beneficiary in the life insurance is so designated in the application for
retirement insurance.
Thus, We see that the GSIS offers two separate and distinct systems of benefits to its
members — one is the life insurance and the other is the retirement insurance. These
two distinct systems of benefits are paid out from two distinct and separate funds that
are maintained by the GSIS.
If the employee failed or overlooked to state the beneficiary of his retirement insurance,
the retirement benefits will accrue to his estate and will be given to his legal heirs in
accordance with law, as in the case of a life insurance if no beneficiary is named in the
insurance policy.
It is Our view, therefore, that the respondent GSIS had correctly acted when it ruled
that the proceeds of the retirement insurance of the late Jose Consuegra should be
divided equally between his first living wife Rosario Diaz, on the one hand, and his
second wife Basilia Berdin and his children by her, on the other.
Nario vs. Phil-Am Life Ins. Co. 20 SCRA 434
G.R. No. L-22796
June 26, 1967
DELFIN
NARIO,
and
ALEJANDRA
SANTOS-NARIO, plaintiffs-appellants,
vs.
THE PHILIPPINE AMERICAN LIFE INSURANCE COMPANY, defendant-appellee.
REYES, J.B.L., J.:

FACTS: Mrs. Alejandra Santos-Nario was, upon application, issued, on June 12,
1959, by the Philippine American Life Insurance Co., a life insurance policy
(No. 503617) under a 20-year endowment plan, with a face value of P5,000.00. She
designated thereon her husband, Delfin Nario, and their unemancipated minor son,
Ernesto Nario, as her irrevocable beneficiaries.
About the middle of June, 1963, Mrs. Nario applied for a loan on the above stated
policy with the Insurance Company, which loan she, as policy-holder, has been entitled
to avail of under one of the provisions of said policy after the same has been in force
for three (3) years, for the purpose of using the proceeds thereof for the school
expenses of her minor son, Ernesto Nario. Said application bore the written signature
and consent of Delfin Nario in two capacities: first, as one of the irrevocable
beneficiaries of the policy; and the other, as the father-guardian of said minor son and
irrevocable beneficiary, Ernesto Nario, and as the legal administrator of the minor's
properties, pursuant to Article 320 of the Civil Code of the Philippines.
The Insurance Company denied said application, manifesting to the policy holder
that the written consent for the minor son must not only be given by his father as legal
guardian but it must also be authorized by the court in a competent guardianship
proceeding.
After the denial of said policy loan application, Mrs. Nario signified her decision to
surrender her policy to the Insurance Company, which she was also entitled to avail of
under one of the provisions of the same policy, and demanded its cash value which then
amounted to P520.00.
The Insurance Company also denied the surrender of the policy, on the same
ground as that given in disapproving the policy loan application; hence, on September
10, 1963, Mrs. Alejandra Santos-Nario and her husband, Delfin Nario, brought suit
against the Philippine American Life Insurance Co. in the above mentioned court
of first instance, seeking to compel Philam to grant their policy loan
application and/or to accept the surrender of said policy in exchange for
its cash value.
Philam answered the complaint, virtually admitting its material allegations, but it set up
the affirmative defense that inasmuch as the policy loan application and the surrender
of the policy involved acts of disposition and alienation of the property rights of the
minor, said acts are not within the powers of the legal administrator, under article 320
in relation to article 326 of the Civil Code; hence, mere written consent given by the
father-guardian, for and in behalf of the minor son, without any court authority
therefor, was not a sufficient compliance of the law, and Philam was, therefore,
justified in refusing to grant and in disapproving the proposed transactions in question.
The lower court found and opined that since the parties expressly stipulated in the
endorsement attached to the policy and which formed part thereof that —

2-S, 2013-2014

17

Insurance Case Digests
Atty. Bathan-Basuel

It is hereby understood and agreed that, notwithstanding the provisions of this
Policy to the contrary, inasmuch as the designation of the beneficiaries have
been made by the Insured without reserving the right to change said
beneficiaries, the Insured may not designate a new beneficiary or
assign, release or surrender this Policy to the Company and exercise
any and all other rights and privileges hereunder or agree with the Company to
any change in or amendment to this Policy, without the consent of the
beneficiaries originally designated;
that under the above quoted provision, the minor son, as one of the designated
irrevocable beneficiaries, "acquired a vested right to all benefits accruing to the policy,
including that of obtaining a policy loan to the extent stated in the schedule of values
attached to the policy; that the proposed transactions in question (policy loan and
surrender of policy) involved acts of disposition or alienation of the minor's properties
for which the consent given by the father-guardian for and in behalf of the minor son,
must be with the requisite court authority; and in the case at bar, such consent was
given by the father-guardian without any judicial authority; said court, agreeing with
defendant's contention, sustained defendant's affirmative defense, and rendered, on
January 28, 1964, its decision dismissing plaintiffs' complaint.
Unable to secure reconsideration of the trial Court's ruling, petitioner appealed directly
to this Court.
ISSUE: Whether PhilAm Life is justified in denying the proposed transaction.
HELD: The appeal is unmeritorious. We agree with the lower court that the vested
interest or right of the beneficiaries in the policy should be measured on
its full face value and not on its cash surrender value, for in case of death of
the insured, said beneficiaries are paid on the basis of its face value and in case the
insured should discontinue paying premiums, the beneficiaries may continue paying it
and are entitled to automatic extended term or paid-up insurance options, etc. and that
said vested right under the policy cannot be divisible at any given time. The proposed
transactions in question (policy loan and surrender of policy) constitute acts of
disposition or alienation of property rights and not merely of management
or administration because they involve the incurring or termination of contractual
obligations.
As above noted, the full face value of the policy is P5,000.00 and the minor's vested
interest therein, as one of the two (2) irrevocable beneficiaries, consists of one-half (½)
of said amount or P2,500.00.
Article 320 of the Civil Code of the Philippines provides —
The father, or in his absence the mother, is the legal administrator of the
property pertaining to the child under parental authority. If the property is
2-S, 2013-2014

worth more than two thousand pesos, the father or mother shall give a bond
subject to the approval of the Court of First Instance.
and article 326 of the same Code reads —
When the property of the child is worth more than two thousand pesos, the
father or mother shall be considered a guardian of the child's property, subject
to the duties and obligations of guardians under the Rules of Court.
It appearing that the minor beneficiary's vested interest or right on the policy exceeds
two thousand pesos (P2,000.00); that plaintiffs did not file any guardianship bond to
be approved by the court; and as later implemented in Section 7, Rule 93 of the
Revised Rules of Court, plaintiffs should have, but, had not, filed a formal application or
petition for guardianship, plaintiffs-parents cannot possibly exercise the powers vested
on them, as legal administrators of their child's property, under articles 320 and 326 of
the Civil Code. As there was no such petition and bond, the consent given by the
father-guardian, for and in behalf of the minor son, without prior court
authorization, to the policy loan application and the surrender of said
policy, was insufficient and ineffective, and defendant-appellee was justified
in disapproving the proposed transactions in question.
The result would be the same even if we regarded the interest of the ward to be worth
less than P2,000.00. While the father or mother would in such event be exempt from
the duty of filing a bond, and securing judicial appointment, still the parent's authority
over the estate of the ward as a legal-guardian would not extend to acts of
encumbrance or disposition, as distinguished from acts of management or
administration. The distinction between one and the other kind of power is too basic in
our law to be ignored. Thus, under Article 1877 of the Civil Code of the Philippines, an
agency in general terms does not include power to encumber or dispose of the property
of the principal; and the Code explicitly requires a special power or authority for the
agent "to loan or borrow money, unless the latter act be urgent or indispensable for
the preservation of the thing under administration" Similarly, special powers are
required to required to effect novations, to waive any obligation gratuitously or
obligate the principal as a guarantor or surety. By analogy, since the law merely
constitutes the parent as legal administrator of the child's property (which is a general
power), the parent requires special authority for the acts above specified, and this
authority can be given only by a court. This restricted interpretation of the parent's
authority becomes all the more necessary where as in the case before us, there is no
bond to guarantee the ward against eventual losses.
Appellants seek to bolster their petition by invoking the parental power (patria
potestas) under the Civil Code of 1889.The appeal profits them nothing. For the new
Civil Code has not effected a restitutio in integrum of the Spanish patria
potestas; the revival has been only in part.

18

Insurance Case Digests
Atty. Bathan-Basuel

Wherefore, the decision appealed
appellants Nario. So ordered.

from

is

affirmed.

Costs

against

Phil-Am Life Ins Co vs. Pineda 175 SCRA 416
FACTS:
On January 15, 1968, private respondent procured an ordinary life insurance policy
from the petitioner company and designated his wife and children as irrevocable
beneficiaries of said policy. Under date February 22, 1980 private respondent filed a
petition which was docketed as Civil Case No. 9210 of the then Court of First Instance
of Rizal to amend the designation of the beneficiaries in his life policy from irrevocable
to revocable. Petitioner then, on March 10, 1980 filed an Urgent Motion to Reset
Hearing. Also on the same date, petitioner filed its Comment and/or Opposition to
Petition. When the petition was called for hearing on March 19, 1980, the respondent
Judge Gregorio G. Pineda, presiding Judge of the then Court of First Instance of Rizal,
Pasig Branch XXI, denied petitioner's Urgent Motion, thus allowing the private
respondent to adduce evidence, the consequence of which was the issuance of the
questioned Order granting the petition. Petitioner promptly filed a Motion for
Reconsideration but the same was denied in an Order June 10, 1980. Hence, this
petition.

the Petition for Review on Certiorari), to wit: It is hereby understood and agreed that,
notwithstanding the provisions of this policy to the contrary, inasmuch as the
designation of the primary/contingent beneficiary/beneficiaries in this Policy has been
made without reserving the right to change said beneficiary/ beneficiaries, such
designation may not be surrendered to the Company, released or assigned; and no right
or privilege under the Policy may be exercised, or agreement made with the Company to
any change in or amendment to the Policy, without the consent of the said
beneficiary/beneficiaries. (Petitioner's Memorandum, p. 72, Rollo)

2. NO. The alleged acquiescence of the six (6) children beneficiaries of the policy (the
beneficiary-wife predeceased the insured) cannot be considered an effective ratification
to the change of the beneficiaries from irrevocable to revocable. Indubitable is the fact
that all the six (6) children named as beneficiaries were minors at the time,** for which
reason, they could not validly give their consent. Neither could they act through their
father insured since their interests are quite divergent from one another. The parentinsured cannot exercise rights and/or privileges pertaining to the insurance contract, for
otherwise, the vested rights of the irrevocable beneficiaries would be rendered
inconsequential.
Sps. Cha vs. Court of Appeals, 277 SCRA 690
Cha vs Court of Appeals

ISSUE:

1. WHETHER THE DESIGNATION OF THE IRREVOCABLE BENEFICIARIES COULD BE
CHANGED OR AMENDED WITHOUT THE CONSENT OF ALL THE IRREVOCABLE
BENEFICIARIES.

2. WHETHER THE IRREVOCABLE BENEFICIARIES HEREIN, ONE OF WHOM IS ALREADY
DECEASED WHILE THE OTHERS ARE ALL MINORS, COULD VALIDLY GIVE CONSENT TO
THE CHANGE OR AMENDMENT IN THE DESIGNATION OF THE IRREVOCABLE
BENEFICIARIES.
HELD:

1. NO. Needless to say, the applicable law in the instant case is the Insurance Act,
otherwise known as Act No. 2427 as amended, the policy having been procured in
1968. Under the said law, the beneficiary designated in a life insurance contract cannot
be changed without the consent of the beneficiary because he has a vested interest in
the policy (Gercio v. Sun Life Ins. Co. of Canada, 48 Phil. 53; Go v. Redfern and the
International Assurance Co., Ltd., 72 Phil. 71). In this regard, it is worth noting that the
Beneficiary Designation Indorsement in the policy which forms part of Policy Number
0794461 in the name of Rodolfo Cailles Dimayuga states that the designation of the
beneficiaries is irrevocable (Annex "A" of Petition in Sp. Proc. No. 9210, Annex "C" of
2-S, 2013-2014

Padilla
On October 5, 1988, petitioner spouses Nilo Cha and Stella Uy-Cha as lessees, entered
into a lease contract with respondent CKS Development Corporation. The lease contract
specifically provides that:
“ the lessee shall not insure against fire the textiles, chattels, merchandise, goods and
effects placed at any stall or store or space in the leased premises without first
obtaining the written consent and approval of the lessor (CKS).”
It also provides that “if the lessee should should obtain the insurance thereof without
the consent of the lessor then the policy is deemed assigned and transferred to the
lessor for its own benefit.”
The Cha spouses insured the merchandise inside the leased premises for P500,000.00
with United Insurance Co., notwithstanding the said provision in the contract.
On the day that the lease was about to expire, fire broke out in the leased premises.
CKS then wrote United that based on the contract of lease, United should pay the
proceeds of the insurance directly to it (CKS). United refused.
CKS filed a complaint in the RTC of Manila which later rendered a decision in favor of
CKS. On appeal, the CA affirmed the decision of the RTC with the modifications.
19

Insurance Case Digests
Atty. Bathan-Basuel

Hence this petition by the Cha spouses.
In this petition, the Cha spouses argued that the stipulation in the lease contract that
providing for the transfer of the proceeds of the insurance to CKS is null and void for
being contrary to law, morals and public policy.
Issue: whether the automatic transfer of the proceeds of the insurance policy entered
into by CKS and the Cha spouses is valid.
Held: No, the said stipulation is invalid for being contrary to law, morals and public
policy. A non-life insurance policy such as the fire insurance policy taken by the Cha
spouses over the merchandise is primarily a contract of indemnityInsurable interest in
the property insured must exist at the time the insurance takes effect and at the time
the loss occurs. The basis of such requirement of insurable interest in property insured
is based on sound public policy: to prevent a person from taking out an insurance policy
on property upon which he has no insurable interest and collecting the proceeds of said
policy in case of loss of the property. In such a case, the contract of insurance is a mere
wager which is void under Section 25 of the Insurance Code . CKS has no insurable
interest in the goods and merchandise inside the leased premises under Section 17 of
the Insurance Code. Therefore, CKS cannot under the Insurance Code – a special law –
be validly be a beneficiary of the fire insurance policy taken by the petitioner spouses.
The liability of the Cha spouses to CKS for violating their lease contract in that the Cha
spouses obtained a fire insurance policy over their own merchandise, without the
consent of CKS is a separate and distinct issue.
Ong Lim Sing vs. Feb Leasing & Finance Corporation G.R. No. 168115, June 8,
2007
Ong Lim Sing vs Feb Leasing
Facts:
- FEB Leasing and Finance Corporation (FEB) entered into a lease of equipment and
motor vehicles with JVL Food Products (JVL). On the same date, Vicente Ong Lim Sing,
Jr. (Lim) executed an Individual Guaranty Agreement with FEB to guarantee the prompt
and faithful performance of the terms and conditions of the aforesaid lease agreement.
Corresponding Lease Schedules with Delivery and Acceptance Certificates over the
equipment and motor vehicles formed part of the agreement. Under the contract, JVL
was obliged to pay FEB an aggregate gross monthly rental of P170,494.00.
- JVL defaulted in the payment of the monthly rentals. the amount in arrears, including
penalty charges and insurance premiums, amounted to P3,414,468.75. Thereafter, FEB
sent a letter to JVL demanding payment of the said amount. However, JVL failed to
pay.

2-S, 2013-2014

-FEB filed a Complaint with the Regional Trial Court of Manila for sum of money,
damages, and replevin against JVL, Lim, and John Doe.
-In the Amended Answer, JVL and Lim admitted the existence of the lease agreement
but asserted that it is in reality a sale of equipment on installment basis, with FEB acting
as the financier. JVL and Lim claimed that this intention was apparent from the fact
that they were made to believe that when full payment was effected, a Deed of Sale will
be executed by FEB as vendor in favor of JVL and Lim as vendees. FEB purportedly
assured them that documenting the transaction as a lease agreement is just an industry
practice and that the proper documentation would be effected as soon as full payment
for every item was made. They also contended that the lease agreement is a contract
of adhesion and should, therefore, be construed against the party who prepared it, FEB.
- TC: The case was a sale on installment there is no chattel mortgage on the thing sold,
but it appears amongst the Complaint’s prayer, that the plaintiff elected to exact
fulfillment of the obligation.
For the vehicles returned, the plaintiff can only recover the unpaid balance of
the price because of the previous payments made by the defendants for the reasonable
use of the units, specially so, as it appears, these returned vehicles were sold at auction
and that the plaintiff can apply the proceeds to the balance. However, with respect to
the unreturned units and machineries still in the possession of the defendants, they are
liable therefore and accordingly are ordered jointly and severally to pay the price
thereof to the plaintiff together with attorney’s fee and the costs of suit in the sum of
Php25,000.00.
- FEB filed its Notice of Appeal.
- CA: declared the transaction between the parties as a financial lease agreement under
Republic Act (R.A.) No. 8556.
Issue: Whether or not the agreement was a contract of adhesion. Whether petitioner
has insurable interest.
Held:
(1) The subject lease agreement is a contract of adhesion, such a contract is not void
per se. It is as binding as any ordinary contract. A party who enters into an adhesion
contract is free to reject the stipulations entirely. If the terms thereof are accepted
without objection, then the contract serves as the law between the parties.
- In Section 23 of the lease contract, it was expressly stated that:
SECTION 23. ENTIRE AGREEMENT; SEVERABILITY CLAUSE
23.1. The LESSOR and the LESSEE agree this instrument constitute the entire
agreement between them, and that no representations have been made other than as
set forth herein. This Agreement shall not be amended or altered in any manner, unless
20

Insurance Case Digests
Atty. Bathan-Basuel

such amendment be made in writing and signed by the parties hereto.
-Petitioner’s claim that the real intention of the parties was a contract of sale of
personal property on installment basis is more likely a mere afterthought in order to
defeat the rights of the respondent.
- FEB leased the subject equipment and motor vehicles to JVL in consideration of a
monthly periodic payment of P170,494.00. The periodic payment by petitioner is
sufficient to amortize at least 70% of the purchase price or acquisition cost of the said
movables in accordance with the Lease Schedules with Delivery and Acceptance
Certificates.
-The allegation of petitioner that the rent for the use of each movable constitutes the
value of the vehicle or equipment leased is of no moment. The law on financial lease
does not prohibit such a circumstance and this alone does not make the transaction
between the parties a sale of personal property on installment. In fact, the value of the
lease, usually constituting the value or amount of the property involved, is a benefit
allowed by law to the lessor for the use of the property by the lessee for the duration
of the lease. It is recognized that the value of these movables depreciates through
wear and tear upon use by the lessee.
- the validity of Lease between FEB and JVL should be upheld. JVL entered into the
lease contract with full knowledge of its terms and conditions. The contract was in
force for more than four years. Since its inception JVL and Lim never questioned its
provisions. They only attacked the validity of the contract after they were judicially
made to answer for their default in the payment of the agreed rentals.
(2) The stipulation in the lease contract, that the equipment shall be insured at the
cost and expense of the lessee against loss, damage, or destruction from fire, theft,
accident, or other insurable risk for the full term of the lease, is a binding and valid
stipulation. Petitioner, as a lessee, has an insurable interest in the equipment and motor
vehicles leased. Section 17 of the Insurance Code provides that the measure of an
insurable interest in property is the extent to which the insured might be damnified by
loss or injury thereof. It cannot be denied that JVL will be directly damnified in case of
loss, damage, or destruction of any of the properties leased.
- In the financial lease agreement, FEB did not assume responsibility as to the quality,
merchantability, or capacity of the equipment. This stipulation provides that, in case of
defect of any kind that will be found by the lessee in any of the equipment, recourse
should be made to the manufacturer. “The financial lessor, being a financing company,
an extender of credit rather than an ordinary equipment rental company, does not
extend a warranty of the fitness of the equipment for any particular use. Thus, the
financial lessee was precisely in a position to enforce such warranty directly against the
supplier of the equipment and not against the financial lessor. The court found nothing
contra legem or contrary to public policy in such a contractual arrangement.

Facts: E.M. Bachrach commenced an action against the defendant British American
Assurance Company to recover the amount due, deducting the salvage, upon the
following fire insurance policy issued by the defendant to the plaintiff:
The defendant alleged certain facts under which it claimed that it was released from all
obligations whatever under said policy. These special facts are as follows:
-That the plaintiff maintained a paint and varnish shop in the said building where the
goods which were insured were stored.
-That the plaintiff transferred his interest in and to the property covered by the policy
to H. W. Peabody & Co. to secure certain indebtedness due and owing to said company,
and also that the plaintiff had transferred his interest in certain of the goods covered by
the said policy to one Macke, to secure certain obligations assumed by the said Macke
for and on behalf of the insured. That the sanction of the said British American
Insurance had not been obtained by the plaintiff, as required by the said policy.
- That the plaintiff, immediately preceding the outbreak of the alleged fire, willfully
placed a gasoline can containing 10 gallons of gasoline in the upper story of said
building in close proximity to a portion of said goods, wares, and merchandise, which
can was so placed by the plaintiff as to permit the gasoline to run on the floor of said
second story, and after so placing said gasoline, he, the plaintiff, placed in close
proximity to said escaping gasoline a lighted lamp containing alcohol, thereby greatly
increasing the risk of fire.
Bachrach, after denying nearly all of the facts set out in the special answer of the
defendant, alleged:
- That he had been acquitted in a criminal action against him, after a trial duly and
regularly had, upon a charge of arson, based upon the same alleged facts set out in the
answer of the defendant.
After hearing the evidence adduced during the trial of the cause, the lower court found
that the defendant was liable to the plaintiff and rendered a judgment against the
defendant for the sum.
Issue: Whether British American may not be held liable because
1. The use of the building as a paint and varnish shop annulled the policy of insurance.
2. The execution of the chattel mortgages without the knowledge and consent of the
insurance company annulled the policy of insurance.

Bachrach v. British American Insurance Company 17 Phil 555 (1910)
2-S, 2013-2014

21

Insurance Case Digests
Atty. Bathan-Basuel

3. E. M. Bachrach, the insured, willfully placed a gasoline can containing about 10
gallons of gasoline in the upper story of said building, in close proximity to a portion of
the goods, wares, and merchandise stored therein, and that said can was so placed by
said Bachrach as to permit the gasoline to run on the floor of said second story.
4. The court erred in refusing to deduct from the loss sustained by Bachrach the value
of the automobile, which was saved without damage.
Ruling: No. British American is ordered to pay Bachrach.
1.) It is claimed that either gasoline or alcohol was kept in violation of the policy in
the bodega containing the insured property. The property insured consisted mainly of
household furniture kept for the purpose of sale. The preservation of the furniture in a
salable condition by retouching or otherwise was incidental to the business. It is well
settled that the keeping of inflammable oils on the premises, though prohibited by the
policy, does not void it if such keeping is incidental to the business. It may be added
that there was no provision in the policy prohibiting the keeping of paints and varnishes
upon the premises where the insured property was stored
2.) There is no provision in said policy prohibiting the plaintiff from placing a mortgage
upon the property insured, but, admitting that such a provision was intended, the
interest in property insured does not pass by the mere execution of a chattel mortgage
and that while a chattel mortgage is a conditional sale, there is no alienation within the
meaning of the insurance law until the mortgage acquires a right to take possession by
default under the terms of the mortgage. No such right is claimed to have accrued in
the case at bar, and the alienation clause is therefore inapplicable.
3.) The record discloses that some time prior to the commencement of this present
action, a criminal action was commenced against the plaintiff, in which he was charged
with willfully and maliciously burning the property covered by the policy in the present
case. The lower court, that the evidence was insufficient to show beyond peradventure
of doubt that the defendant was guilty of the crime. While the evidence shows some
very peculiar and suspicious circumstances concerning the burning of the goods
covered by the said policy, yet, nevertheless, in view of the findings of the lower court
and in view of the apparent conflict in the testimony, we can not find that there is a
preponderance of evidence showing that the plaintiff did actually set fire or cause fire
to be set to the goods in question.
4.) It does not positively appear of record that the automobile in question was not
included in the other policies. It does appear that the automobile was saved and was
considered as a part of the salvaged. It is alleged that the salvage amounted to P4,000,
including the automobile. This amount (P4,000) was distributed among the different
insurers and the amount of their responsibility was proportionately reduced. The
defendant and appellant in the present case made no objection at any time in the lower
court to that distribution of the salvage. The claim is now made for the first time.
2-S, 2013-2014

San Miguel Brewery vs. Law Union & Rock Ins. Co., 41 Phil 674 (1920)
Facts:
-

-

-

-

-

-

On Jan. 12, 1918, Dunn mortgaged a parcel of land to SMB to secure a debt
of 10T.
Mortgage contract stated that Dunn was to have the property insured at his
own expense, authorizing SMB to choose the insurers and to receive the
proceeds thereof and retain so much of the proceeds as would cover the
mortgage debt.
Dunn likewise authorized SMB to take out the insurance policy for him.
Brias, SMB’s general manager, approached Law Union for insurance to the
extent of 15T upon the property. In the application, Brias stated that SMB’s
interest in the property was merely that of a mortgagee.
Law Union, not wanting to issue a policy for the entire amount, issued one for
P7,500 and procured another policy of equal amount from Filipinas Cia de
Seguros. Both policies were issued in the name of SMB only and contained no
reference to any other interests in the propty. Both policies required
assignments to be approved and noted on the policy.
Premiums were paid by SMB and charged to Dunn. A year later, the policies
were renewed.
In 1917, Dunn sold the property to Harding, but no assignment of the policies
was made to the latter.
Property was destroyed by fire. SMB filed an action in court to recover on
the policies. Harding was made a defendant because by virtue of the sale, he
became the owner of the property, although the policies were issued in SMB’s
name.
SMB sought to recover the proceeds to the extent of its mortgage credit with
the balance to go to Harding.
Insurance Companies contended that they were not liable to Harding because
their liability under the policies was limited to the insurable interests of SMB
only.
SMB eventually reached a settlement with the insurance companies and was
paid the balance of it’s mortgage credit. Harding was left to fend for
himself. Trial court ruled against Harding. Hence the appeal.

Issue: Whether or not the insurance companies are liable to Harding for the balance of
the proceeds of the 2 policies.
Held: NO. Under the Insurance Act, the measure of insurable interest in the property is
the extent to which the insured might be daminified by the loss or injury thereof. Also
it is provided in the IA that the insurance shall be applied exclusively to the proper
interest of the person in whose name it is made. Undoubtedly, SMB as the mortgagee
of the property, had an insurable interest therein; but it could NOT, an any event,
recover upon the two policies an amount in excess of its mortgage credit.
By virtue of the Insurance Act, neither Dunn nor Harding could have recovered from the
two policies. With respect to Harding, when he acquired the property, no change or
22

Insurance Case Digests
Atty. Bathan-Basuel

assignment of the policies had been undertaken. The policies might have been worded
differently so as to protect the owner, but this was not done.
If the wording had been: “Payable to SMB, mortgagee, as its interests may appear,
remainder to whomsoever, during the continuance of the risk, may become owner of
the interest insured”, it would have proved an intention to insure the entire interest in
the property, NOT merely SMB’s and would have shown to whom the money, in case of
loss, should be paid. Unfortunately, this was not what was stated in the policies.
If during the negotiation for the policies, the parties had agreed that even the owner’s
interest would be covered by the policies, and the policies had inadvertently been
written in the form in which they were eventually issued, the lower court would have
been able to order that the contract be reformed to give effect to them in the sense
that the parties intended to be bound. However, there is no clear and satisfactory
proof that the policies failed to reflect the real agreement between the parties that
would justify the reformation of these two contracts.
Heirs of Loreto C. Maramag represented by surviving spouse Vicente Pangilinan
Maramag v. Eva Verna De Guzman Maramag, et al, G.R. No. 181132, June 5, 2009
HEIRS OF LORETO C. MARAMAG, represented by
PANGILINAN
vs.
EVA VERNA DE GUZMAN MARAMAG, ODESSA DE
BRIAN DE GUZMAN MARAMAG, TRISHA ANGELIE
LIFE ASSURANCE COMPANY, LTD., and GREAT
CORPORATION, Respondents.

surviving spouse VICENTA
MARAMAG,Petitioners,
GUZMAN MARAMAG, KARL
MARAMAG, THE INSULAR
PACIFIC LIFE ASSURANCE

The petition alleged that: (1) petitioners were the legitimate wife and children of Loreto
Maramag (Loreto), while respondents were Loreto’s illegitimate family; (2) Eva de
Guzman Maramag (Eva) was a concubine of Loreto and a suspect in the killing of the
latter, thus, she is disqualified to receive any proceeds from his insurance policies from
Insular Life Assurance Company, Ltd. (Insular)4 and Great Pacific Life Assurance
Corporation (Grepalife);5 (3) the illegitimate children of Loreto—Odessa, Karl Brian, and
Trisha Angelie—were entitled only to one-half of the legitime of the legitimate children,
thus, the proceeds released to Odessa and those to be released to Karl Brian and Trisha
Angelie were inofficious and should be reduced; and (4) petitioners could not be
deprived of their legitimes, which should be satisfied first.
In answer,6 Insular admitted that Loreto misrepresented Eva as his legitimate wife and
Odessa, Karl Brian, and Trisha Angelie as his legitimate children, and that they filed their
claims for the insurance proceeds of the insurance policies; that when it ascertained
that Eva was not the legal wife of Loreto, it disqualified her as a beneficiary and divided
the proceeds among Odessa, Karl Brian, and Trisha Angelie, as the remaining designated
beneficiaries; and that it released Odessa’s share as she was of age, but withheld the
release of the shares of minors Karl Brian and Trisha Angelie pending submission of
letters of guardianship. Insular alleged that the complaint or petition failed to state a
2-S, 2013-2014

cause of action insofar as it sought to declare as void the designation of Eva as
beneficiary, because Loreto revoked her designation as such in Policy No. A001544070
and it disqualified her in Policy No. A001693029; and insofar as it sought to declare as
inofficious the shares of Odessa, Karl Brian, and Trisha Angelie, considering that no
settlement of Loreto’s estate had been filed nor had the respective shares of the heirs
been determined. Insular further claimed that it was bound to honor the insurance
policies designating the children of Loreto with Eva as beneficiaries pursuant to Section
53 of the Insurance Code.
In its own answer7 with compulsory counterclaim, Grepalife alleged that Eva was not
designated as an insurance policy beneficiary; that the claims filed by Odessa, Karl Brian,
and Trisha Angelie were denied because Loreto was ineligible for insurance due to a
misrepresentation in his application form that he was born on December 10, 1936 and,
thus, not more than 65 years old when he signed it in September 2001; that the case
was premature, there being no claim filed by the legitimate family of Loreto; and that
the law on succession does not apply where the designation of insurance beneficiaries is
clear.
The RTC initially ruled in favour of plaintiff-petitioners but upon a motion for
reconsideration by the respondents, it reversed its own decision and dismissed the
case. CA affirmed
ISSUE: (A)re the members of the legitimate family entitled to the proceeds of the
insurance for the concubine?
HELD: although petitioners are the legitimate heirs of Loreto, they were not named
as beneficiaries in the insurance policies issued by Insular and Grepalife. The basis of
petitioners’ claim is that Eva, being a concubine of Loreto and a suspect in his murder,
is disqualified from being designated as beneficiary of the insurance policies, and that
Eva’s children with Loreto, being illegitimate children, are entitled to a lesser share of
the proceeds of the policies. They also argued that pursuant to Section 12 of the
Insurance Code,19 Eva’s share in the proceeds should be forfeited in their favor, the
former having brought about the death of Loreto. Thus, they prayed that the share of
Eva and portions of the shares of Loreto’s illegitimate children should be awarded to
them, being the legitimate heirs of Loreto entitled to their respective legitimes.
It is evident from the face of the complaint that petitioners are not entitled to a
favorable judgment in light of Article 2011 of the Civil Code which expressly provides
that insurance contracts shall be governed by special laws, i.e., the Insurance Code.
Section 53 of the Insurance Code states—
SECTION 53. The insurance proceeds shall be applied exclusively to the proper interest
of the person in whose name or for whose benefit it is made unless otherwise specified
in the policy.

23

Insurance Case Digests
Atty. Bathan-Basuel

Pursuant thereto, it is obvious that the only persons entitled to claim the insurance
proceeds are either the insured, if still alive; or the beneficiary, if the insured is already
deceased, upon the maturation of the policy.20 The exception to this rule is a situation
where the insurance contract was intended to benefit third persons who are not parties
to the same in the form of favorable stipulations or indemnity. In such a case, third
parties may directly sue and claim from the insurer.21
Petitioners are third parties to the insurance contracts with Insular and Grepalife and,
thus, are not entitled to the proceeds thereof. Accordingly, respondents Insular and
Grepalife have no legal obligation to turn over the insurance proceeds to petitioners.
The revocation of Eva as a beneficiary in one policy and her disqualification as such in
another are of no moment considering that the designation of the illegitimate children
as beneficiaries in Loreto’s insurance policies remains valid. Because no legal
proscription exists in naming as beneficiaries the children of illicit relationships by the
insured,22the shares of Eva in the insurance proceeds, whether forfeited by the court in
view of the prohibition on donations under Article 739 of the Civil Code or by the
insurers themselves for reasons based on the insurance contracts, must be awarded to
the said illegitimate children, the designated beneficiaries, to the exclusion of
petitioners. It is only in cases where the insured has not designated any
beneficiary,23 or when the designated beneficiary is disqualified by law to receive the
proceeds,24 that the insurance policy proceeds shall redound to the benefit of the
estate of the insured.
Violeta Lalican v. The Insular Life Assurance Company, Limited, G.R. No. 183526,
August 25, 2009
LALICAN VS. INSULAR LIFE ASSURANCE COMPANY LIMITED
G.R. No. 183526
August 25, 2009
Chico-Nazario, J.;
FACTS: Violeta is the widow of the deceased Eulogio C. Lalican (Eulogio). During his
lifetime, Eulogio applied for an insurance policy with Insular Life. On 24 April 1997,
Insular Life, through Josephine Malaluan (Malaluan), its agent in Gapan City, issued in
favor of Eulogio Policy No. 9011992, which contained a 20-Year Endowment Variable
Income Package Flexi Plan worth P500,000.00, with two riders valued at P500,000.00
each. Thus, the value of the policy amounted to P1,500,000.00. Violeta was named as
the primary beneficiary.
Under the terms of Policy No. 9011992, Eulogio was to pay the premiums on
a quarterly basis in the amount of P8,062.00, payable every 24 April, 24 July, 24
October and 24 January of each year, until the end of the 20-year period of the policy.
According to the Policy Contract, there was a grace period of 31 days for the payment
of each premium subsequent to the first. If any premium was not paid on or before the
due date, the policy would be in default, and if the premium remained unpaid until the
end of the grace period, the policy would automatically lapse and become void.

However, he failed to pay the premium due on 24 January 1998, even after the lapse of
the grace period of 31 days. Policy No. 9011992, therefore, lapsed and became void.
Eulogio submitted to the Cabanatuan District Office of Insular Life, through
Malaluan, on 26 May 1998, an Application for Reinstatement of Policy No. 9011992,
together with the amount of P8,062.00 to pay for the premium due on 24 January
1998. In a letter dated 17 July 1998, Insular Life notified Eulogio that his Application
for Reinstatement could not be fully processed because, although he already deposited
P8,062.00 as payment for the 24 January 1998 premium, he left unpaid the overdue
interest thereon amounting to P322.48. Thus, Insular Life instructed Eulogio to pay the
amount of interest and to file another application for reinstatement. Eulogio was
likewise advised by Malaluan to pay the premiums that subsequently became due on 24
April 1998 and 24 July 1998, plus interest.
On 17 September 1998, Eulogio went to Malaluan’s house and submitted a
second Application for Reinstatement of Policy No. 9011992, including the amount of
P17,500.00, representing payments for the overdue interest on the premium for 24
January 1998, and the premiums which became due on 24 April 1998 and 24 July
1998. As Malaluan was away on a business errand, her husband received Eulogio’s
second Application for Reinstatement and issued a receipt for the amount Eulogio
deposited.
A while later, on the same day, 17 September 1998, Eulogio died of cardiorespiratory arrest secondary to electrocution.
Without knowing of Eulogio’s death, Malaluan forwarded to the Insular Life
Regional Office in the City of San Fernando, on 18 September 1998, Eulogio’s second
Application for Reinstatement of Policy No. 9011992 and P17,500.00 deposit.
However, Insular Life no longer acted upon Eulogio’s second Application for
Reinstatement, as the former was informed on 21 September 1998 that Eulogio had
already passed away.
On 28 September 1998, Violeta filed with Insular Life a claim for payment of
the full proceeds of Policy No. 9011992.
Without waiting for the result of the re-evaluation by Insular Life, Violeta filed
with the RTC, on 11 October 1999, a Complaint for Death Claim Benefit, which was
docketed as Civil Case No. 2177. Violeta alleged that Insular Life engaged in unfair claim
settlement practice and deliberately failed to act with reasonable promptness on her
insurance claim. Violeta prayed that Insular Life be ordered to pay her death claim
benefits on Policy No. 9011992, in the amount of P1,500,000.00, plus interests,
attorney’s fees, and cost of suit.
After trial, the RTC rendered, on 30 August 2007, a decision in favor of Insular

Eulogio paid the premiums due on 24 July 1997 and 24 October 1997.
2-S, 2013-2014

Life.
24

Insurance Case Digests
Atty. Bathan-Basuel

Violeta then appealed directly to the Supreme Court alleging a question of law
arising in this case involving the interpretation of the second sentence of Section 19 of
the Insurance Code, which provides:
“Interest in the life or health of a person insured must exist when
the insurance takes effect, but need not exist thereafter or when
the loss occurs.”
ISSUE: Whether or not Eulogio still had insurable interest in his own life when he
reinstated Policy No. 9011992 just before he passed away on September 17, 1998
HELD: An insurable interest is one of the most basic and essential requirements in an
insurance contract. In general, an insurable interest is that interest which a person is
deemed to have in the subject matter insured, where he has a relation or connection
with or concern in it, such that the person will derive pecuniary benefit or advantage
from the preservation of the subject matter insured and will suffer pecuniary loss or
damage from its destruction, termination, or injury by the happening of the event
insured against. The existence of an insurable interest gives a person the legal right to
insure the subject matter of the policy of insurance. Section 10 of the Insurance Code
indeed provides that every person has an insurable interest in his own life. Section 19 of
the same code also states that an interest in the life or health of a person insured must
exist when the insurance takes effect, but need not exist thereafter or when the loss
occurs.
Upon more extensive study of the Petition, it becomes evident that the matter of
insurable interest is entirely irrelevant in the case at bar. It is actually beyond question
that while Eulogio was still alive, he had an insurable interest in his own life, which he did
insure under Policy No. 9011992.

-

ISSUE: W/N Insurance Company of North America can claim against Gaisano Cagayan
for the debt that was isnured
HELD: YES. petition is partly GRANTED. order to pay P535,613 is DELETED








Gaisano Cagayan, Inc. v. Insurance Company of North America, G.R. No. 147839,
June 8, 2006
FACTS:
-

-

-

-


Intercapitol Marketing Corporation (IMC) is the maker of Wrangler Blue Jeans.
while Levi Strauss (Phils.) Inc. (LSPI) is the local distributor of products
bearing trademarks owned by Levi Strauss & Co
IMC and LSPI separately obtained from Insurance Company of North
America fire insurance policies for their book debt endorsements related to
their ready-made clothing materials which have been sold or delivered to
various customers and dealers of the Insured anywhere in the Philippines which
are unpaid 45 days after the time of the loss
February 25, 1991: Gaisano Superstore Complex in Cagayan de Oro City,
owned by Gaisano Cagayan, Inc., containing the ready-made clothing materials
sold and delivered by IMC and LSPI was consumed by fire.
February 4, 1992: Insurance Company of North America filed a complaint for
damages against Gaisano Cagayan, Inc. alleges that IMC and LSPI filed their
claims under their respective fire insurance policies which it paid thus it was

2-S, 2013-2014

subrogated to their rights
o
Gaisano Cagayan, Inc: not be held liable because it was destroyed
due to fortuities event or force majeure
RTC: IMC and LSPI retained ownership of the delivered goods until fully paid, it
must bear the loss (res perit domino)
CA: Reversed - sales invoices is an exception under Article 1504 (1) of the
Civil Code to res perit domino



-

insurance policy is clear that the subject of the insurance is the book debts
and NOT goods sold and delivered to the customers and dealers of the
insured
ART. 1504. Unless otherwise agreed, the goods remain at the seller's risk until
the ownership therein is transferred to the buyer, but when the ownership
therein is transferred to the buyer the goods are at the buyer's risk whether
actual delivery has been made or not, except that:


(1) Where delivery of the goods has been made to the buyer or to a bailee for
the buyer, in pursuance of the contract and the ownership in the goods has
been retained by the seller merely to secure performance by the buyer of his
obligations under the contract, the goods are at the buyer's risk from the time
of such delivery;
IMC and LSPI did not lose complete interest over the goods. They have an
insurable interest until full payment of the value of the delivered goods. Unlike
the civil law concept of res perit domino, where ownership is the basis for
consideration of who bears the risk of loss, in property insurance, one's
interest is not determined by concept of title, but whether insured has
substantial economic interest in the property

Section 13 of our Insurance Code defines insurable interest as "every interest
in property, whether real or personal, or any relation thereto, or liability in
respect thereof, of such nature that a contemplated peril might directly
damnify the insured." Parenthetically, under Section 14 of the same Code, an
insurable interest in property may consist in: (a) an existing interest; (b) an
inchoate interest founded on existing interest; or (c) an expectancy, coupled
with an existing interest in that out of which the expectancy arises.
Anyone has an insurable interest in property who derives a benefit from its
existence or would suffer loss from its destruction.

it is sufficient that the insured is so situated with reference to the
property that he would be liable to loss should it be injured or destroyed
by the peril against which it is insured an insurable interest in property
does not necessarily imply a property interest in, or a lien upon, or
possession of, the subject matter of the insurance, and neither the title
nor a beneficial interest is requisite to the existence of such an interest
insurance in this case is not for loss of goods by fire but for
25

Insurance Case Digests
Atty. Bathan-Basuel

-

-

-

petitioner's accounts with IMC and LSPI that remained unpaid 45 days after
the fire - obligation is pecuniary in nature obligor should be held exempt from
liability when the loss occurs thru a fortuitous event only holds true when the
obligation consists in the delivery of a determinate thing and there is no
stipulation holding him liable even in case of fortuitous event
Article 1263 of the Civil Code in an obligation to deliver a generic thing, the
loss or destruction of anything of the same kind does not extinguish the
obligation (Genus nunquan perit)
The subrogation receipt, by itself, is sufficient to establish not only the
relationship of respondent as insurer and IMC as the insured, but also the
amount paid to settle the insurance claim
Art. 2207. If the plaintiff's property has been insured, and he has received
indemnity from the insurance company for the injury or loss arising out of the
wrong or breach of contract complained of, the insurance company shall be
subrogated to the rights of the insured against the wrongdoer or the person
who has violated the contract.
o
As to LSPI, no subrogation receipt was offered in evidence.
o
Failure to substantiate the claim of subrogation is fatal to petitioner's
case for recovery of the amount of P535,613
Garcia vs. The Hongkong Fire & Marine Insurance Co., Ltd, G.R. No. 20341,
September 1, 1923

G.R. No. 20341; September 1, 1923
DOMINGO GARCIA and PNB, plaintiffs-appellees vs. THE HONGKONG FIRE &
MARINE INSURANCE CO., LTD., defendant-appellant
FACTS: In 1918, in Manila, Domingo Garcia, merchant-owner of a bazaar known as "Las
Novedades" in Legaspi, Albay, entered into a contract with Hongkong Fire to insure his
merchandise for P15,000 at a premium of P300 per annum. The latter issued fire
insurance policy No. 1951, not on the merchandise but on the building which contained
it. The policy does not contain the true agreement and intent of the parties. Garcia was
not the owner of the building.
In 1919, Garcia executed a mortgage to PNB on the merchandise to secure a loan. The
bank advised Hongkong Fire of the same. In 1920, and while the policy was in force and
effect, a fire took place which destroyed the merchandise valued at P20,000, together
with the building itself. Garcia demanded P15,000, as provided, but payment was
refused. The lower court decided in favor of Garcia.
ISSUE: Whether Garcia is entitled to the insurance.
HELD: YES. The policy was in English, of which Garcia is ignorant. He knew that
P15,000 was correct, and never investigated. When Hongkong Fire surrendered the
policy to PNB, the word "merchandise" was written in the letter. It is contended that
when the letter was written, the Bank, which then had the possession of the policy,
knew that it covered the building and did not insure the merchandise.
2-S, 2013-2014

Garcia had his dealings with the officials of the branch Bank at Legaspi where he was
doing business as a merchant, of which the officials of that Bank had knowledge. Under
such facts, the presumption of knowledge, if any, on the part of the Bank would be that
the policy was on the merchandise. Be that as it may, when the defendant received the
letter from the Bank, it knew from its own records that the policy was issued on the
building, and, as a matter of fair dealing, it should have notified the Bank that the policy
was on the building.
In the final analysis, Garcia wanted insurance upon a stock of goods, which he owned,
and he received and paid for a policy on a building, which he did not own, and while the
policy was in force and effect, both the building, which he did not own, and the stock of
merchandise, which he did own, were completely destroyed by fire.
The defense is purely technical, and is founded upon the contention that Garcia cannot
recover, because the policy covers loss on a building, and does not cover loss of
merchandise. It is very apparent that a mistake was made in the issuance of the policy.
In its opinion the trial court says: Under these circumstances it seems clear and
manifest that all have been in the belief that it was not the building but the
merchandise that was insured, for the reason that none of them paid attention to the
context of the policy. The opinion of the trial court further points out that, under the
pleadings and proof, there is ground for the contention that the plaintiff would be
entitled to recover on the policy for the loss of the building. Lower court decision is
affirmed.
Rizal Commercial Banking Corporation (RCBC) vs. Court of Appeals GR 128833, 20
April 1998
Rizal Commercial Banking Corporation (RCBC) vs. Court of Appeals [GR
128833, 20 April 1998];
Facts:
Goyu & Sons, Inc. (Goyu) applied for credit facilities and accommodations with
Rizal Commercial Banking Corporation (RCBC) at its Binondo Branch. After due
evaluation, RCBC Binondo Branch, through its key officers, petitioners Uy Chun Bing and
Eli D. Lao, recommended Goyu's application for approval by RCBC's executive
committee. A credit facility in the amount of P30 million was initially granted. Upon
Goyu's application and Uy's and Lao's recommendation, RCBC's executive committee
increased Goyu's credit facility to P50 million, then to P90 million, and finally to P117
million.
As security for its credit facilities with RCBC, Goyu executed two real estate
mortgages and two chattel mortgages in favor of RCBC, which were registered with the
Registry of Deeds at Valenzuela, Metro Manila. Under each of these four mortgage
contracts, Goyu committed itself to insure the mortgaged property with an insurance
company approved by RCBC, and subsequently, to endorse and deliver the insurance
policies to RCBC. Goyu obtained in its name a total of 10 insurance policies from MICO.
26

Insurance Case Digests
Atty. Bathan-Basuel

In February 1992, Alchester Insurance Agency, Inc., the insurance agent
where Goyu obtained the Malayan insurance policies, issued 9 endorsements in favor of
RCBC seemingly upon instructions of Goyu. On 27 April 1992, one of Goyu's factory
buildings in Valenzuela was gutted by fire. Consequently, Goyu submitted its claim for
indemnity on account of the loss insured against. MICO denied the claim on the ground
that the insurance policies were either attached pursuant to writs of
attachments/garnishments issued by various courts or that the insurance proceeds
were also claimed by other creditors of Goyu alleging better rights to the proceeds than
the insured.
Goyu filed a complaint for specific performance and damages which was
docketed at the Regional Trial Court of the National Capital Judicial Region (Manila,
Branch 3) as Civil Case 93-65442. RCBC, one of Goyu's creditors, also filed with MICO
its formal claim over the proceeds of the insurance policies, but said claims were also
denied for the same reasons that AGCO denied Goyu's claims. In an interlocutory order
dated 12 October 1993, the Regional Trial Court of Manila (Branch 3), confirmed that
Goyu's other creditors, namely, Urban Bank, Alfredo Sebastian, and Philippine Trust
Company obtained their respective writs of attachments from various courts, covering
an aggregate amount of P14,938,080.23, and ordered that the proceeds of the 10
insurance policies be deposited with the said court minus the aforementioned
P14,938,080.23. Accordingly, on 7 January 1994, MICO deposited the amount of
P50,505,594.60 with Branch 3 of the Manila RTC.
In the meantime, another notice of garnishment was handed down by another
Manila RTC sala (Branch 28) for the amount of P8,696,838.75. After trial, Branch 3 of
the Manila RTC rendered judgment in a favor of Goyu, ordering Malayan to pay Goyu its
fire loss claims in the total amount of P74,040,518.58 less the amount of
P50,000,000.00 which is deposited with the Court; damages by way of interest for the
duration of the delay since 27 July 1992 (90 days after Malayan's receipt of the
required proof of loss and notice of loss) at the rate of twice the ceiling prescribed by
the Monetary Board, on the amounts of (1) P50,000,000.00 from 27 July 1992 up to
the time said amount was deposited with the Court on 7 January 1994; and (2)
P24,040,518.58 — from 17 July 1992 up to the time when the writs of attachments
were received by Malayan. The court also ordered RCBC to pay Goyu actual and
compensatory damages in the amount of P2,000,000.00, and both Malayan and RCBC
to solidarily pay Goyu (1) P1,000,000.00 as exemplary damages; (2) P1,000,000.00
as, and for, attorneys fees; and (3) Costs of suit. The Court, on the Counterclaim of
RCBC, ordered Goyu to pay its loan obligations with RCBC in the amount of
P68,785,069.04, as of 27 April 1992, with interest thereon at the rate stipulated in
the respective promissory notes (without surcharges and penalties).
From this judgment, all parties interposed their respective appeals.
Issue: Whether RCBC, as mortgagee, has any right over the insurance policies taken by
Goyu, the mortgagor, in case of the occurrence of loss
Held: YES. It is settled that a mortgagor and a mortgagee have separate and distinct
insurable interests in the same mortgaged property, such that each one of them may
insure the same property for his own sole benefit. There is no question that Goyu could
insure the mortgaged property for its own exclusive benefit. Herein, although it appears
2-S, 2013-2014

that Goyu obtained the subject insurance policies naming itself as the sole payee, the
intentions of the parties as shown by their contemporaneous acts, must be given due
consideration in order to better serve the interest of justice and equity. It is to be
noted that nine endorsement documents were prepared by Alchester in favor of RCBC.
The Court is in a quandary how Alchester could arrive at the idea of endorsing any
specific insurance policy in favor of any particular beneficiary or payee other than the
insured had not such named payee or beneficiary been specifically disclosed by the
insured itself.
It is also significant that Goyu voluntarily and purposely took the insurance
policies from MICO, a sister company of RCBC, and not just from any other insurance
company. Alchester would not have found out that the subject pieces of property were
mortgaged to RCBC had not such information been voluntarily disclosed by Goyu itself.
Had it not been for Goyu, Alchester would not have known of Goyu's intention of
obtaining insurance coverage in compliance with its undertaking in the mortgage
contracts with RCBC, and verify, Alchester would not have endorsed the policies to
RCBC had it not been so directed by Goyu. On equitable principles, particularly on the
ground of estoppel, the Court is constrained to rule in favor of mortgagor RCBC.
RCBC, in good faith, relied upon the endorsement documents sent to it as this
was only pursuant to the stipulation in the mortgage contracts. Such reliance is justified
under the circumstances of the case. Goyu failed to seasonably repudiate the authority
of the person or persons who prepared such endorsements. Over and above this, Goyu
continued, in the meantime, to enjoy the benefits of the credit facilities extended to it
by RCBC. After the occurrence of the loss insured against, it was too late for Goyu to
disown the endorsements for any imagined or contrived lack of authority of Alchester
to prepare and issue said endorsements. If there had not been actually an implied
ratification of said endorsements by virtue of Goyu's inaction in this case, Goyu is at the
very least estopped from assailing their operative effects.
To permit Goyu to capitalize on its non- confirmation of these endorsements
while it continued to enjoy the benefits of the credit facilities of RCBC which believed in
good faith that there was due endorsement pursuant to their mortgage contracts, is to
countenance grave contravention of public policy, fair dealing, good faith, and justice.
Such an unjust situation, the Court cannot sanction. Under the peculiar circumstances,
the Court is bound to recognize RCBC's right to the proceeds of the insurance policies if
not for the actual endorsement of the policies, at least on the basis of the equitable
principle of estoppel.
Filipino Merchants Insurance Co. Inc. vs. Court of Appeals GR 85141, 28, November
1989
Filipino Merchants Insurance Co. Inc. vs. Court of Appeals
GR 85141, 28, November 1989

FACTS:
Choa insured with Filipino Merchants the shipment of supposedly 600 metric
tons of fishmeal from Bangkok, Thailand to Manila against all risks under warehouse to
warehouse terms. However, what was imported was only 59.940 metric tons at
27

Insurance Case Digests
Atty. Bathan-Basuel

$395.42 per ton. The fishmeal in 666 new gunny bags arrived in Manila and were
unloaded from the ship SS Bougainville. Upon receipt, the arrastre contractor E. Razon
and Filipino Merchant's surveyor certified that 227 bags were in bad order condition
amounting to 12,148 kilos. Choa made a formal claim against Filipino Merchant for
P51,568.63 (amount of loss). However, Filipino Merchant refused to pay the claim.
Consequently, Choa filed an action against Filipino Merchant seeking to recover
P51,568.62 representing damages to said shipment which has been insured by the
defendant insurance company.
RTC Manila rendered judgment in favor of Choa.
On appeal, CA affirmed the decision of RTC.
ISSUE:
1.

2.

WON Filipino Merchant may be held liable under the “all risks” clause of the
marine insurance policy in the absence of proof of some fortuitous event,
casualty, or accidental cause to which the loss is attributable.
WON the marine insurance policy taken out by Choa is null and void
considering that he had no insurable interest in the subject cargo.

HELD:
1. The SC ruled that Filipino Merchant is liable under the “all risks” clause of the marine
insurance policy it had with Choa.
Filipino Merchant contended that an "all risks" marine policy has a
technical meaning in insurance in that before a claim can be compensable it is essential
that
there must be "some fortuity, " "casualty" or "accidental cause" to which the alleged
loss is
attributable and the failure of herein private respondent, upon whom lay the burden, to
adduce evidence showing that the alleged loss to the cargo in question was due to a
fortuitous event precludes his right to recover from the insurance policy.

when the policy attached and that the cargo was damaged when unloaded from the
vessel; thereafter, the burden then shifts to the insurer to show the exception to the
coverage.
In the present case, there being no showing that the loss was caused by any
of the excepted perils, the insurer is liable under the policy. There is no evidence
presented to show that the condition of the gunny bags in which the fishmeal was
packed was such that they could not hold their contents in the course of the necessary
transit, much less any evidence that the bags of cargo had burst as the result of the
weakness of the bags themselves. Had there been such a showing that spillage would
have been a certainty, there may have been good reason to plead that there was no risk
covered by the policy.
2. Choa as consignee of the goods in transit under an invoice containing the terms
under "C & F Manila," has insurable interest in said goods.
Choa as vendee/consignee of the goods in transit has such existing interest
therein as may be the subject of a valid contract of insurance. His interest over the
goods is based on the perfected contract of sale. The perfected contract of sale
between him and the shipper of the goods operates to vest in him an equitable title
even before delivery or before be performed the conditions of the sale. The contract of
shipment, whether under F.O.B., C.I.F., or C. & F. as in this case, is immaterial in the
determination of whether the vendee has an insurable interest or not in the goods in
transit. The perfected contract of sale even without delivery vests in the vendee an
equitable title, an existing interest over the goods sufficient to be the subject of
insurance.
Also pursuant to Article 1523 of the NCC, the delivery of the goods on board
the carrying vessels partake of the nature of actual delivery since, from that time, the
foreign buyers assumed the risks of loss of the goods and paid the insurance premium
covering them.
4.

The SC ruled in the negative. It explained that an "all risks policy" should be
read literally as meaning all risks whatsoever and covering all losses by an accidental
cause of any kind. The terms "accident" and "accidental", as used in insurance
contracts, have not acquired any technical meaning. They are construed by the courts
in their ordinary and common acceptance. The term "all risks" cannot be given a
strained technical meaning, the language of the clause
under the Institute Cargo Clauses being unequivocal and clear, to the effect that it
extends to all damages/losses suffered by the insured cargo except (a) loss or damage
or expense proximately caused by delay, and (b) loss or damage or expense proximately
caused by the inherent vice or nature of the subject matter insured. Generally, the
burden of proof is upon the insured to show that a loss arose from a covered peril, but
under an "all risks" policy the burden is not on the insured to prove the precise cause of
loss or damage for which it seeks compensation. The insured under an "all risks
insurance policy" has the initial burden of proving that the cargo was in good condition
2-S, 2013-2014

Concealment and Representation
Ma. Lourdes S. Florendo v. Philam Plans, Inc., Perla Abcede and Ma. Celeste Abcede,
G.R. No. 186983, February 22, 2012

(1) G.R. No. 186983
February 22, 2012
MA.
LOURDES
S.
FLORENDO, Petitioner,
vs.
PHILAM PLANS, INC., PERLA ABCEDE MA. CELESTE ABCEDE, Respondents.
ABAD, J.:
FACTS: On October 23, 1997 Manuel Florendo filed an application for
comprehensive pension plan with respondent Philam Plans, Inc. (Philam
Plans) after some convincing by respondent Perla Abcede. The plan had a pre-need
price of P997,050.00, payable in 10 years, and had a maturity value
28

Insurance Case Digests
Atty. Bathan-Basuel

of P2,890,000.00 after 20 years.1 Manuel signed the application and left to Perla
the task of supplying the information needed in the application.2 Respondent Ma.
Celeste Abcede, Perla’s daughter, signed the application as sales counselor.3
The comprehensive pension plan also provided life insurance coverage to
Florendo.4This was covered by a Group Master Policy that Philam Life issued to
Philam Plans.5 Under the master policy, Philam Life was to automatically provide life
insurance coverage, including accidental death, to all who signed up for Philam Plans’
comprehensive pension plan.6 If the plan holder died before the maturity of the plan,
his beneficiary was to instead receive the proceeds of the life insurance, equivalent to
the pre-need price. Further, the life insurance was to take care of any unpaid premium
until the pension plan matured, entitling the beneficiary to the maturity value of the
pension plan.7
On October 30, 1997 Philam Plans issued Pension Plan Agreement to Manuel, with
petitioner Ma. Lourdes S. Florendo, his wife, as beneficiary. In time, Manuel paid his
quarterly premiums.9
Eleven months later or on September 15, 1998, Manuel died of blood poisoning.
Subsequently, Lourdes filed a claim with Philam Plans for the payment of the benefits
under her husband’s plan.10 Because Manuel died before his pension plan matured and
his wife was to get only the benefits of his life insurance, Philam Plans forwarded her
claim to Philam Life.11
On May 3, 1999 Philam Plans wrote Lourdes declining her claim. Philam Life found that
Manuel was on maintenance medicine for his heart and had an implanted
pacemaker. Further, he suffered from diabetes mellitus and was taking insulin.
Lourdes renewed her demand for payment under the plan13 but Philam Plans rejected
it,14prompting her to file the present action against the pension plan company
before the RTC of QC. 1 5
On March 30, 2006 the RTC ruled that Manuel was not guilty of concealing the
state of his health from his pension plan application; ordered Philam Plans,
Perla and Ma. Celeste, solidarily, to pay Lourdes all the benefits from her
husband’s pension plan, namely: P997,050.00, the proceeds of his term insurance,
and P2,890,000.00 lump sum pension benefit upon maturity of his plan; P100,000.00
as moral damages; and to pay the costs of the suit.
On December 18, 2007, CA reversed the RTC decision,17 holding that insurance
policies are traditionally contracts uberrimae fidae or contracts of utmost good faith. It
required Manuel to disclose conditions affecting the risk of which he was
aware or material facts that he knew or ought to know. 1 8
ISSUES: (1) Whether Manuel is guilty of concealing his illness when he kept blank and did
not answer questions in his pension plan application.
2-S, 2013-2014

(2) Whether Manuel was bound by the failure of Perla and Ma. Celeste to declare the
condition of Manuel’s health.
(3) Whether Philam Plans’ approval of Manuel’s pension plan application and acceptance
of his premium payments precluded it from denying Lourdes’ claim.
HELD: (1) YES. Lourdes contends that Philam Plans should have returned the application
to him for completion. Since it approved the application just as it was, it cannot cry
concealment on Manuel’s part, that Philam Plans never queried Manuel directly regarding
the state of his health.
Since Philam Plans waived medical examination for Manuel, it had to rely largely on his
stating the truth regarding his health in his application. He knew more than anyone that
he had been under treatment for heart condition and diabetes for more than five years
preceding his application. But he kept those crucial facts from Philam Plans.
When Manuel signed the application, he adopted as his own the written
representations and declarations embodied in it. It is clear from these
representations that he concealed his chronic heart ailment and diabetes
from Philam Plans. The pertinent portion of his representations and declarations read
as follows:
(c) I have never been treated for heart condition, high blood pressure, cancer,
diabetes, lung, kidney or stomach disorder or any other physical impairment in
the last five years.
(d) I am in good health and physical condition.
Manuel signed the application without filling in the details regarding his continuing
treatments for heart condition and diabetes. The assumption is that he has never been
treated for the said illnesses in the last five years preceding his application.
Lourdes insists that Perla, the soliciting agent, knew that Manuel had a pacemaker
before he signed up for the pension plan.23 But by its tenor, the responsibility
for preparing the application belonged to Manuel. Nothing in it implies that
someone else may provide the information that Philam Plans needed. Manuel cannot
sign the application and disown the responsibility for having it filled up. If
he furnished Perla the needed information and delegated to her the filling up of the
application, then she acted on his instruction, not on Philam Plans’ instruction. Manuel
still had his pacemaker when he applied for a pension plan and it is an admission that he
remained under treatment for irregular heartbeat within five years preceding that
application.
Manuel had been taking medicine when he submitted his pension plan application. These
clearly fell within the five-year period. It is not claimed that Perla was aware of his two
other afflictions that needed medical treatments. Pursuant to Section 27 of IC,
Manuel’s concealment entitles Philam Plans to rescind its contract of
insurance with him.
29

Insurance Case Digests
Atty. Bathan-Basuel

(2) Lourdes contends that the mere fact that Manuel signed the application in blank and
let Perla fill in the details did not make her his agent and bind him to her concealment of
his true state of health. There is no evidence of collusion between them.
Manuel, in signing the pension plan application, he certified that he wrote
all the information stated in it or had someone do it under his direction.
Assuming that it was Perla who filled up the application form, Manuel is still bound by
what it contains since he certified that he authorized her action. Philam Plans
had every right to act on the faith of that certification. Manuel was made aware when
he signed the pension plan application that, in granting the same, Philam Plans and
Philam Life were acting on the truth of the representations contained in that
application.
Manuel, a civil engineer and manager of a construction company, could be
expected to know that one must read every document, especially if it
creates rights and obligations affecting him, before signing the same. It
could reasonably be expected that he would not trifle with something that would
provide additional financial security to him and to his wife in his twilight years.
(3) Lourdes contends that any defect or insufficiency in the information provided by
his pension plan application should be deemed waived after the same has been
approved, the policy has been issued, and the premiums have been collected. 34
The Court cannot agree. The comprehensive pension plan that Philam Plans
issued contains a one-year incontestability period.
The incontestability clause precludes the insurer from disowning liability under the
policy it issued on the ground of concealment or misrepresentation regarding the health
of the insured after a year of its issuance.
Since Manuel died on the eleventh month following the issuance of his
plan, 3 6 the one year incontestability period has not yet set in.
CA decision AFFIRMED.
Vda. De Canilang v. CA, 223 SCRA 443 (1993)
Facts:
-

-

-

Canilang consulted Dr. Claudio and was diagnosed as suffering from "sinus
tachycardia." Mr. Canilang consulted the same doctor again on 3 August 1982
and this time was found to have "acute bronchitis."
On the next day, 4 August 1982, Canilang applied for a "non-medical"
insurance policy with Grepalife naming his wife, as his beneficiary. Canilang
was issued ordinary life insurance with the face value of P19,700.
On 5 August 1983, Canilang died of "congestive heart failure," "anemia," and

2-S, 2013-2014

-

"chronic anemia." The wife as beneficiary, filed a claim with Grepalife which
the insurer denied on the ground that the insured had concealed material
information from it.
Vda Canilang filed a complaint with the Insurance Commissioner against
Grepalife contending that as far as she knows her husband was not suffering
from any disorder and that he died of kidney disorder.
Grepalife was ordered to pay the widow by the Insurance Commissioner
holding that there was no intentional concealment on the Part of Canilang and
that Grepalife had waived its right to inquire into the health condition of the
applicant by the issuance of the policy despite the lack of answers to "some of
the pertinent questions" in the insurance application. CA reversed.

Issue: Whether or not Grepalife is liable.
Held: SC took note of the fact that Canilang failed to disclose that hat he had twice
consulted Dr. Wilfredo B. Claudio who had found him to be suffering from "sinus
tachycardia" and "acute bronchitis. Under the relevant provisions of the Insurance
Code, the information concealed must be information which the concealing party knew
and "ought to [have] communicate[d]," that is to say, information which was "material
to the contract.
The information which Canilang failed to disclose was material to the ability of Grepalife
to estimate the probable risk he presented as a subject of life insurance. Had Canilang
disclosed his visits to his doctor, the diagnosis made and the medicines prescribed by
such doctor, in the insurance application, it may be reasonably assumed that Grepalife
would have made further inquiries and would have probably refused to issue a nonmedical insurance policy or, at the very least, required a higher premium for the same
coverage.
The materiality of the information withheld by Canilang from Grepalife did not depend
upon the state of mind of Jaime Canilang. A man's state of mind or subjective belief is
not capable of proof in our judicial process, except through proof of external acts or
failure to act from which inferences as to his subjective belief may be reasonably drawn.
Neither does materiality depend upon the actual or physical events which ensue.
Materiality relates rather to the "probable and reasonable influence of the facts" upon
the party to whom the communication should have been made, in assessing the risk
involved in making or omitting to make further inquiries and in accepting the application
for insurance; that "probable and reasonable influence of the facts" concealed must, of
course, be determined objectively, by the judge ultimately.
SC found it difficult to take seriously the argument that Grepalife had waived inquiry
into the concealment by issuing the insurance policy notwithstanding Canilang's failure
to set out answers to some of the questions in the insurance application. Such failure
precisely constituted concealment on the part of Canilang. Petitioner's argument, if
accepted, would obviously erase Section 27 from the Insurance Code of 1978.
Ng Gan Zee v. Asian Crusader Life, 122 SCRA 461
30

Insurance Case Digests
Atty. Bathan-Basuel

NG GAN ZEE vs. ASIAN CRUSADER LIFE ASSURANCE CORPORATION
Facts:
-

-

-

-

Kwong Nam applied for a 20-year endowment insurance on his life for the sum
of P20,000.00, with his wife, Ng Gan Zee as beneficiary. On the same date,
ACAC, upon receipt of the required premium from the insured, approved the
application and issued the corresponding policy.
Kwong Nam died of cancer of the liver with metastasis. All premiums had been
religiously paid at the time of his death.
Kwong Nam widow Ng Gan Zee presented a claim in due form to appellant for
payment of the face value of the policy. She submitted the required proof of
death of the insured. ACAC denied the claim on the ground that the answers
given by the insured to the questions appealing in his application for life
insurance were untrue.
Ng Gan Zee brought the matter to the attention of the Insurance
Commissioner, the Hon. Francisco Y. Mandamus, and the latter, after
conducting an investigation, wrote the ACAC that he had found no material
concealment on the part of the insured and that, therefore, Ng Gan Zee should
be paid the full face value of the policy. This opinion of the Insurance
Commissioner notwithstanding, ACAC refused to settle its obligation.
ACAC alleged that the insured was guilty of misrepresentation when he
answered "No" to the following question appearing in the application for life
insurance:

Has any life insurance company ever refused your application for insurance
or for reinstatement of a lapsed policy or offered you a policy different
from that applied for? If, so, name company and date.

-

-

-

-

According to ACAC
"The insured applied for reinstatement of his lapsed life insurance policy with the Insular
Life Insurance Co., Ltd, but this was declined by the insurance company, although later
on approved for reinstatement with a very high premium as a result of his medical
examination. Thus notwithstanding the said insured answered 'No' to the question
propounded to him"
Issue: Whether Ng Gan Zee can collect the insurance claim.
CFI: ordered ACAC to pay the face value of the insurance policy

-

Held:
-

There is no evidence that the Insular Life Assurance Co., Ltd. ever refused any
application of Kwong Nam for insurance. Neither is there any evidence that any

2-S, 2013-2014

-

other insurance company has refused any application of Kwong Nam for
insurance. The evidence shows that the Insular Life Assurance Co., Ltd.
approved Kwong Nam's request for reinstatement and amendment of his
lapsed insurance policy. The Court notes from said application for
reinstatement and amendment, that the amount applied for was P20,000.00
only and not for P50,000.00 as it was in the lapsed policy. The amount of the
reinstated and amended policy was also for P20,000.00. It results, therefore,
that when Kwong Nam answered 'No' to the question whether any life
insurance company ever refused his application for reinstatement of a lapsed
policy he did not misrepresent any fact.
Sec. 27. Such party a contract of insurance must communicate to the other,
in good faith, all facts within his knowledge which are material to the contract,
and which the other has not the means of ascertaining, and as to which he
makes no warranty.
Thus, "concealment exists where the assured had knowledge of a fact material
to the risk, and honesty, good faith, and fair dealing requires that he should
communicate it to the assurer, but he designedly and intentionally withholds
the same." It has also been held "that the concealment must, in the absence
of inquiries, be not only material, but fraudulent, or the fact must have been
intentionally withheld."
Assuming that the aforesaid answer given by the insured is false, as claimed
by the ACAC. Sec. 27 of the Insurance Law, nevertheless requires that
fraudulent intent on the part of the insured be established to entitle the
insurer to rescind the contract. And as correctly observed by the lower court,
"misrepresentation as a defense of the insurer to avoid liability is an
'affirmative' defense. The duty to establish such a defense by satisfactory and
convincing evidence rests upon the defendant. The evidence before the Court
does not clearly and satisfactorily establish that defense."
It bears emphasis that Kwong Nam had informed the ACAC's medical examiner
that the tumor for which he was operated on was "associated with ulcer of the
stomach." In the absence of evidence that the insured had sufficient medical
knowledge as to enable him to distinguish between "peptic ulcer" and "a
tumor", his statement that said tumor was "associated with ulcer of the
stomach, " should be construed as an expression made in good faith of his
belief as to the nature of his ailment and operation. Indeed, such statement
must be presumed to have been made by him without knowledge of its
incorrectness and without any deliberate intent on his part to mislead the
appellant.
While it may be conceded that, from the viewpoint of a medical expert, the
information communicated was imperfect, the same was nevertheless
sufficient to have induced appellant to make further inquiries about the
ailment and operation of the insured.
Section 32. The right to information of material facts maybe waived either by
the terms of insurance or by neglect to make inquiries as to such facts where
they are distinctly implied in other facts of which information is communicated
It has been held that where, upon the face of the application, a question
appears to be not answered at all or to be imperfectly answered, and the
31

Insurance Case Digests
Atty. Bathan-Basuel

-

insurers issue a policy without any further inquiry, they waive the imperfection
of the answer and render the omission to answer more fully immaterial.
As aptly noted by the lower court, "if the ailment and operation of Kwong Nam
had such an important bearing on the question of whether the defendant
would undertake the insurance or not, the court cannot understand why the
defendant or its medical examiner did not make any further inquiries on such
matters from the Chinese General Hospital or require copies of the hospital
records from the appellant before acting on the application for insurance. The
fact of the matter is that the defendant was too eager to accept the
application and receive the insured's premium. It would be inequitable now to
allow the defendant to avoid liability under the circumstances."

Henson v. Philamlife, 56 OG 7328
Facts:
-

-

Celestino Henson was insured by Philamlife in 1954 upon his application or a
20-yr endowment life policy.
In 1955, the policy lapsed due to non-payment of the premiums.
Upon payment of the premiums due, the policy was reinstated, but in the
application for reinstatement, Henson did not disclose the fact that he had
been previously diagnosed for pyelonephritis, enlarged liver and hernia. He
also did not disclose that he had been examined by a physician.
In 1956, Henson died, and his beneficiaries’ claim was rejected by Philamlife on
the ground of concealment.
The company then filed for rescission. Beneficiaries’ contend that the intent
to conceal must be proven to warrant rescission.

Issue: Whether or not there is need to prove intent to conceal to warrant rescission.
Held: NO.
Sec. 26 provides that “a concealment whether intentional or unintentional entitles the
injured party to rescind the contract of insurance”. And aside from this, intent, being a
state of the mind is hard to prove.
According to Sec. 30 of the Insurance Code: Materiality is to be determined not by the
event, but solely by the probable and reasonable influence of the facts upon the party
to whom the communication is due, in forming his estimate of the disadvantages of the
proposed contract, or in making his inquiries. In essence therefore, the insured need not
have died of the very diseases he had failed to reveal to the insurance company. It is
sufficient that his non-revelation had misled the insurer in forming its estimate of the
disadvantages of the proposed policy reinstatement or in making its inquiries, in order
to entitle the latter to rescind the contract.
Yu Pang Cheng v. CA, 105 PHIL 1930
YU PANG CHENG alias YU PANG CHING, petitioner,
vs.
2-S, 2013-2014

THE COURT OF APPEALS, ET AL., respondents.
FACTS: Plaintiff brought this action to collect from defendant the sum of P10,000.00,
value of an insurance policy taken upon the life of one Yu Pang Eng, plus interest
thereon at the legal rate, the sum of P10,000.00 as moral damages the further sum of
P3,000.00 as attorney's fees, and the costs of action.
Defendant, in its answer, set up the defense that the insured was guilty of
misrepresentation and concealment of material facts in that he gave false and
untruthful answers to certain questions asked him in his application for insurance which
were material to the risk insured against and have the effect of avoiding the insurance
policy.
Rtc ordered defendant to pay. On appeal, the Court of Appeals reversed the decision of
the trial court, holding that the insured was guilty of concealment of material facts
which relieves defendant from liability.
The ISSUE to be determined is whether the insured is guilty of concealment of some
facts material to the risk insured against which has the effect of avoiding the policy as
found by respondent court.
HELD: The insured, in his application for insurance, particularly in his declarations to the
examining physician, stated the following in answering the questions propounded to
him:
14. Have you ever had any of the following diseases or symptoms? Each question must
be read and answered "Yes" or "No".
xxx
xxx
xxx
Gastritis, Ulcer of the Stomach or any disease of that organ? No.
Vertigo, Dizziness, Fainting-spells or Unconscious? No.
Cancer, Tumors or Ulcers of any kind? No.
15. Have you ever consulted any physician not included in any of the above answers?
Give names and address or physicians list ailments or accidents and date. No.
It appears that the insured entered the Chinese General Hospital for medical treatment
on January 29, 1950 having stayed there up to February 11, 1950. Upon entering the
hospital, he complained of dizziness, anemia, abdominal pains and tarry stools, and in
the evening of his admission he had several abdominal pains and his discharges were
with black tarry stools and felt dizzy and weak. The history of his illness shows that the
same "started a year ago as frequent dizziness." An X-Ray picture of his stomach was
taken and the diagnosis made of him by his doctors showed that his illness was "peptic
ulcer, bleeding."
It should be noted that the insured's confinement in the Chinese General Hospital took
place from January 29, 1950 to February 11, 1950, whereas his application for
insurance wherein he stated his answer to the questions propounded to him by the
examining physician of defendant was submitted to defendant on September 5, 1950.
It is apparent that when the insured gave his answers regarding his previous ailment,
particularly with regard to "Gastritis, Ulcer of the Stomach or any disease of that organ"
and "Vertigo, Dizziness, Fainting-spells or Unconsciousness", he concealed the ailment
of which he was treated in the Chinese General, Hospital which precisely has direct
connection with the subject of the questions propounded. The negative answers given
32

Insurance Case Digests
Atty. Bathan-Basuel

by the insured regarding his previous ailment, or his concealment of the fact that he
was hospitalized and treated for sometime of peptic ulcer and had suffered from
"dizziness, anemia, abdominal pains and tarry stools", deprived defendant of the
opportunity to make the necessary inquiry as to the nature of his past illness so that as
it may form its estimate relative to the approval of his application. Had defendant been
given such opportunity, considering the previous illness of the insured as disclosed by
the record of the Chinese General Hospital, defendant would probably had never
consented to the issuance of the policy in question. In fact, according to the death
certificate, the insured died of "infiltrating medullary carcinoma, Grade 4, advanced
cardiac and of lesser curvature, stomach metastases spleen", which may have direct
connection with his previous illness.
Argente v. West Coast Life Insurance Co., 51 PHIL 725
ARGENTE VS. WEST COAST LIFE INSURANCE CO.
G.R. No. 28499 March 19, 1928
Malcom, J.;
FACTS:
On February 9, 1925, Bernardo Argente signed an application for joint
insurance with his wife in the sum of P2,000. The wife, Vicenta de Ocampo, signed a
like application for the same policy. Both applications, with the exception of the names
and the signatures of the applicants, were written by Jose Geronimo del Rosario, an
agent for the West Coast Life Insurance Co. But all the information contained in the
applications was furnished the agent by Bernardo Argente.
Pursuant to his application, Bernardo Argente was examined by Dr. Cesareo
Sta. Ana, a medical examiner for the West Coast Life Insurance Co., on February 10,
1925, in the office of the Customs House. The result of such examination was recorded
in the Medical Examiner's Report, and with the exception of the signature of Bernardo
Argente, was in the hand-writing of Doctor Sta. Ana. But the information or answers to
the questions contained on the face of the Medical Examiner's Report were furnished
the doctor by the applicant, Bernardo Argente.
Pursuant to her application, Vicenta de Ocampo, wife of the plaintiff, was
examined by Dr. Cesareo Sta. Ana on February 10, 1925, at her residence in Manila. The
result of the medical examination, including among other things, the answers given by
Vicenta de Ocampo to the questions propounded to her by the physician, appears in the
Medical Examiner's Report.
On May 9, 1925, Bernardo Argente and his wife submitted to the West Coast
Life Insurance Co. an amended application for insurance, increasing the amount thereof
to P15,000, and asked that the policy be dated May 15, 1925.
A temporary policy for P15,000 was issued to Bernardo Argente and his wife
as of May 15, but it was not delivered to Bernardo Argente until July 2, 1925, when the
2-S, 2013-2014

first quarterly premium on the policy was paid. In view of the fact that more than thirty
days had elapsed since the applicants were examined by the company's physician, each
of them was required to file a certificate of health before the policy was delivered to
them.
On November 18, 1925, Vicenta de Ocampo died of cerebral apoplexy.
Thereafter Bernardo Argente presented a claim in due form to the West Coast Life
Insurance Co. for the payment of the sum of P15,000 the amount of the joint life
Insurance policy. Following investigation conducted by the Manager of the Manila office
of the insurance company, it was apparently disclosed that the answers given by the
insured in their medical examinations with regard to their health and previous illness and
medical attendance were untrue. For that reason, the West Coast Life Insurance Co.
refused to pay the claim of Bernardo Argente, and on May 25, 1926, wrote him to the
effect that the claim was rejected because the insurance was obtained through fraud
and misrepresentation.
It is admitted that it appears in the Medical Examiner's Report that Bernardo
Argente, in response to the question asked by the medical examiner, "Have you ever
consulted a physician for, or have you ever suffered from any ailment or disease of, the
brain or nervous system?" answered "No." To the question, "Have you consulted a
physician for any ailment or disease not included in your above answer," answered "Yes.
Nature of Ailment, Disease or Injury. Scabies, Number of attacks 1, Date 1911. It is,
however, not disputed that on January 10, 11, and 13, 1923, Bernardo Argente was
confined in the Philippine General Hospital where he was treated by Dr. Agerico B. M.
Sison for cerebral congestion and Bell's Palsy.
It is further admitted that it appears in the Medical Examiner's Report that
Vicenta de Ocampo, in response to the question asked by the medical examiner, "How
frequently, if at all, and in what quantity do you use beer, wine, spirits or other
intoxicants?" answered "Beer only in small quantities occasionally." To the question,
"Have you ever consulted a physician for or have you ever suffered from any ailment or
disease of the brain or nervous system?" answered "No." To the question, "What
physician or physicians, if any, not named above, have you consulted or been treated
by, within the last five years and for what illness or ailment? (If none, so state)"
answered "None." And to the question, "Are you in good health as far as you know and
believe?" answered "Yes."It is, however, not disputed that Vicenta de Ocampo was
taken by a patrolman, at the request of her husband, Bernardo Argente, on May 19,
1924, to the Meisic police station, and from there was transferred to the San Lazaro
Hospital. In San Lazaro Hospital, her case was diagnosed by the admitting physician as
"alcoholism," but later Doctor Domingo made a diagnosis of probable "manic-depressive
psychosis," and still, later in Mary Chiles Hospital, made a final diagnosis of "phychoneurosis."
The plaintiff, Bernardo Argente, while readily conceding most of the facts
herein narrated, yet alleges that both he and his wife revealed to the company's
physician. Doctor Sta. Ana, all the facts concerning the previous illnesses and medical
33

Insurance Case Digests
Atty. Bathan-Basuel

attendance, but that Doctor Sta. Ana, presumably acting in collusion, with the insurance
agent, Jose Geronimo del Rosario, failed to record them in the medical reports.
ISSUE:
Whether or not the alleged concealment was immaterial and insufficient to
avoid the policy
HELD:
In an action on a life insurance policy where the evidence conclusively shows
that the answers to questions concerning diseases were untrue, the truth of falsity of
the answers become the determining factor. In the policy was procured by fraudulent
representations, the contract of insurance apparently set forth therein was never legally
existent. It can fairly be assumed that had the true facts been disclosed by the assured,
the insurance would never have been granted.
It is also held that the concealment must, in the absence of inquiries, be not
only material, but fraudulent, or the fact must have been intentionally withheld; so it is
held under English law that if no inquiries are made and no fraud or design to conceal
enters into the concealment the contract is not avoided.
But it would seem that if a material fact is actually known to the assured, its
concealment must of itself necessarily be a fraud, and if the fact is one which the
assured ought to know, or is presumed to know, the presumption of knowledge ought
to place the assured in the same position as in the former case with relation to material
facts; and if the jury in such cases find the fact material, and one tending to increase
the risk, it is difficult to see how the inference of a fraudulent intent or intentional
concealment can be avoided.
And if insured while being examined for life insurance and knowing that she
had heart disease, falsely stated that she was in good health, and though she could not
read the application, it was explained to her and the questions asked through an
interpreter, and the application like the policy contained and provision that no liability
should be incurred unless the policy was delivered while the insured was in good health,
the court properly directed a verdict for the insurer, though a witness who was present
at the examination testified that the insured was not asked whether she had heart
disease.
The basis of the rule vitiating the contract in case of concealment is that it
misleads or deceives the insurer into accepting the risk, or accepting it at the rate of
premium agreed upon. If the assured has exclusive knowledge of material facts, he
should fully and fairly disclose the same, whether he believes them material or not. The
determination of the point whether there has or has not been a material concealment
must rest largely in all cases upon the form of the questions propounded and the exact
terms of the contract.
Saturnino v. Philamlife7 SCRA 316
2-S, 2013-2014

Appellant: IGNACIO SATURNINO, in his own behalf and as the JUDICIAL
GUARDIAN OF CARLOS SATURNINO
Appellees: THE PHILIPPINE AMERICAN LIFE INSURANCE COMPANY,
Ponente: MAKALINTAL, J.

Facts:
Ignacio Saturnino sought to recover the sum of P5,000.00, corresponding to the
face value 20-year endowment non-medical insurance on the life of Estefania A.
Saturnino. This type of policy dispenses with the medical examination of the applicant
usually required in ordinary life policies but a detailed information is called for in the
application concerning the applicant's health and medical history.
The application was submitted by Saturnino to Philamlife on November 16, 1957,
witnessed by appellee's agent Edward A. Santos.. On September 19, 1958 Saturnino
died of pneumonia, secondary to influenza so her surviving husband and minor
childdemanded payment of the face value of the policy. The claim was rejected on the
ground that two months prior to the issuance of the policy, Saturnino was operated on
for cancer, involving complete removal of the right breast. She was discharged and the
surgeon said she could not be considered cured for her cancer was malignant type. And
Estefania A. Saturnino did not make a disclosure thereof in her application for insurance.
On the contrary, she stated therein that she did not have, nor had she ever had, among
other ailments listed in the application, cancer or other tumors; that she had not
consulted any physician, undergone any operation or suffered any injury within the
preceding five years; and that she had never been treated for nor did she ever have any
illness or disease peculiar to her sex, particularly of the breast, ovaries, uterus, and
menstrual disorders. The application also recites that the foregoing declarations
constituted "a further basis for the issuance of the policy."
Issue:
whether or not the insured made such false representations of material facts as to
avoid the policy.
Held:
The Insurance Law (Section 30) provides that "materiality is to be determined not by
the event, but solely by the probable and reasonable influence of the facts upon the
party to whom the communication is due, in forming his estimate of the proposed
contract, or in making his inquiries." the waiver of medical examination renders even
more material the information required of the applicant concerning previous condition of
health and diseases suffered, for such information necessarily constitutes an important
factor which the insurer takes into consideration in deciding whether to issue the policy
or not. It is logical to assume that if appellee had been properly apprised of the
insured's medical history she would at least have been made to undergo medical
examination in order to determine her insurability.
There was no negligence on the part of the insurance company for it was precisely
because the insured had given herself a clean bill of health that appellee no longer
considered an actual medical checkup necessary.
Saturnino contend there was no fraudulent concealment of the truth inasmuch as the
insured herself did not know, since her doctor never told her, that the disease for which
she had been operated on was cancer. However, In this jurisdiction a concealment,
34

Insurance Case Digests
Atty. Bathan-Basuel

whether intentional or unintentional, entitles the insurer to rescind the contract of
insurance, concealment being defined as "negligence to communicate that which a
party knows and ought to communicate" (Sections 24 & 26, Act No. 2427). n the case
of Argente v. West Coast Life Insurance Co., 51 Phil. 725, 732, this Court said, quoting
from Joyce, The Law of Insurance, 2nd ed., Vol. 3:
"The basis of the rule vitiating the contract in cases of concealment is that it
misleads or deceives the insurer into accepting the risk, or accepting it at the
rate of premium agreed upon. The insurer, relying upon the belief that the
assured will disclose every material fact within his actual or presumed
knowledge, is misled into a belief that the circumstance withheld does not
exist, and he is thereby induced to estimate the risk upon a false basis that it
does not exist."
Judgment dismissing the complaint affirmed.
Edillon v. Manila Bankers Life Insurance Corp., G.R. No. 34200, September 30,
1982
Edillon v Manila Bankers Life G.R. No. L-34200 September 30, 1982
J. Vasquez
Facts:
Carmen O, Lapuz applied with Manila Bankers for insurance coverage
against accident and injuries. She gave the date of her birth as July 11, 1904. She
paid the sum of P20.00 representing the premium for which she was issued the
corresponding receipt. The policy was to be effective for 90 days.
During the effectivity, Carmen O. Lapuz died in a vehicular accident in the North
Diversion Road.
Petitioner Regina L. Edillon, a sister of the insured and the beneficiary in the policy,
filed her claim for the proceeds of the insurance. Her claim having been denied, Regina
L. Edillon instituted this action in the trial court.
The insurance corporation relies on a provision contained in the contract excluding its
liability to pay claims under the policy in behalf of "persons who are under the age of
sixteen (16) years of age or over the age of sixty (60) years" They pointed out that
the insured was over sixty (60) years of age when she applied for the insurance
coverage, hence the policy became void.
The trial court dismissed the complaint and ordered Edillon to pay P1000. The reason
was that a policy of insurance being a contract of adhesion, it was the duty of the
insured to know the terms of the contract he or she is entering into.
The insured could not have been qualified under the conditions stated in
said contract and should have asked for a refund of the premium.
Issue:
Whether the acceptance by the insurance corporation of the premium and the
issuance of the corresponding certificate of insurance should be deemed a waiver of
the exclusionary condition of coverage stated in the policy.
2-S, 2013-2014

Held: Yes. Petition granted.
Ratio:
The age of Lapuz was not concealed to the insurance company. Her application clearly
indicated her age of the time of filing the same to be almost 65 years of age. Despite
such information which could hardly be overlooked, the insurance corporation received
her payment of premium and issued the corresponding certificate of insurance without
question.
There was sufficient time for the private respondent to process the application and to
notice that the applicant was over 60 years of age and cancel the policy.
Under the circumstances, the insurance corporation is already deemed in estoppel. It
inaction to revoke the policy despite a departure from the exclusionary condition
contained in the said policy constituted a waiver of such condition, similar to Que Chee
Gan vs. Law Union Insurance.
The insurance company was aware, even before the policies were issued, that in the
premises insured there were only two fire hydrants contrary to the requirements of the
warranty in question.
It is usually held that where the insurer, at the time of the issuance of a policy
of insurance, has knowledge of existing facts which, if insisted on, would invalidate the
contract from its very inception, such knowledge constitutes a waiver of conditions in
the contract inconsistent with the known facts, and the insurer is stopped thereafter
from asserting the breach of such conditions.
To allow a company to accept one's money for a policy of insurance which it then
knows to be void and of no effect, though it knows as it must, that the assured
believes it to be valid and binding, is so contrary to the dictates of honesty and fair
dealing.
Capital Insurance & Surety Co., Inc. vs. - involved a violation of the provision of the
policy requiring the payment of premiums before the insurance shall become effective.
The company issued the policy upon the execution of a promissory note for the
payment of the premium. A check given subsequent by the insured as partial payment
of the premium was dishonored for lack of funds. Despite such deviation from the terms
of the policy, the insurer was held liable.
“... is that although one of conditions of an insurance policy is that "it shall not be
valid or binding until the first premium is paid", if it is silent as to the mode of
payment, promissory notes received by the company must be deemed to have been
accepted in payment of the premium. In other words, a requirement for the payment of
the first or initial premium in advance or actual cash may be waived by acceptance of
a promissory note...”
Sun Life v. CA, 245 SCRA 268 (1995)
G.R. No. 105135; June 22, 1995
SUNLIFE ASSURANCE COMPANY OF CANADA, petitioner vs. CA and Spouses
ROLANDO and BERNARDA BACANI, respondents
QUIASON, J.:
FACTS:
35

Insurance Case Digests
Atty. Bathan-Basuel

In 1986, Robert John Bacani procured a life insurance contract for himself from Sun Life
Assurance Company of Canada. He was issued a policy valued at P100,000, with double
indemnity in case of accidental death. The designated beneficiary was his mother,
Bernarda Bacani. In 1987, Robert died in a plane crash. Bernarda filed a claim with Sun
Life, seeking the benefits of the insurance policy taken by her son. Sun Life conducted
an investigation which prompted it to reject the claim because Robert did not disclose
material facts (consultation within the past 5 years and seeking medical advice)
relevant to the issuance of the policy, thus rendering the contract of insurance
voidable. A check representing the total premiums paid (P10,172) was given to
Bernarda. Sun Life discovered that two weeks prior to Robert’s application for
insurance, he was examined and confined at the Lung Center of the Philippines, where
he was diagnosed for renal failure.
In 1988, Bernarda and her husband Rolando filed an action for specific performance
against Sun Life in RTC Valenzuela, which ruled in favor of the spouses concluding that
the facts concealed by the insured were made in good faith and under a belief that they
need not be disclosed. Moreover, it held that the health history of the insured was
immaterial since the insurance policy was "non-medical". CA affirmed. MR denied. Hence,
the appeal.
ISSUE:
Whether the concealment is enough to rescind the insurance contract.
ISSUE:
YES. Sec. 26 of The Insurance Code is explicit in requiring a party to a contract of
insurance to communicate to the other, in good faith, all facts within his knowledge
which are material to the contract and as to which he makes no warranty, and which the
other has no means of ascertaining. Said Section provides: “A neglect to communicate
that which a party knows and ought to communicate, is called concealment.”
The terms of the contract are clear. The insured is specifically required to disclose to
the insurer matters relating to his health. The information which the insured failed to
disclose were material and relevant to the approval and issuance of the insurance policy.
The matters concealed would have definitely affected petitioner's action on his
application. Thus, "good faith" is no defense in concealment.
The argument, that petitioner's waiver of the medical examination of the insured
debunks the materiality of the facts concealed, is untenable. Such argument would
make Sec. 27, which allows the injured party to rescind a contract of insurance where
there is concealment, ineffective.
Even if the facts concealed had no bearing to the cause of death of the insured, it is
well settled that the insured need not die of the disease he had failed to disclose to the
insurer. Petitioner properly exercised its right to rescind the contract of insurance by
reason of the concealment employed by the insured. It must be emphasized that
rescission was exercised within the two-year contestability period as recognized in
Section 48 of The Insurance Code. Petition GRANTED.
2-S, 2013-2014

Insular Life v. Feliciano 74 PHIL 468
Insular Life Assurance Co. v. Feliciano, 74 Phil. 468 (1943);
FACTS:
Evaristo, who died on September 29, 1935, was suffering with advanced
pulmonary tuberculosis when he signed his application for insurance with the
petitioner on October 12, 1934. On that same date Doctor Trepp, who had taken Xray pictures of his lungs, informed the respondent Dr. Serafin , brother of Evaristo,
that the latter "was already in a very serious and practically hopeless condition."
Nevertheless the question contained in the application — "Have you ever suffered
from any ailment or disease of the lungs, pleurisy, pneumonia or asthma?" — appears
to have been answered, "No." And above the signature of the applicant, following the
answers to the various questions propounded to him, is the following printed
statement:
"I declare on behalf of myself and of any person who shall
have or claim any interest in any policy issued hereunder, that each
of the above answers is full, complete and true, and that to the
best of my knowledge and belief I am a proper subject for life
insurance." (Exhibit K.)
The false answer above referred to, as well as the others, was written by
the Company's soliciting agent Romulo M. David, in collusion with the medical
examiner Dr. Gregorio Valdez, for the purpose of securing the Company's approval
of the application so that the policy to be issued thereon might be credited to said
agent in connection with the inter-provincial contest which the Company was then
holding among its soliciting agents to boost the sales of its policies. Agent David
bribed Medical Examiner Valdez with money which the former borrowed from the
applicant's mother by way of advanced payment on the premium, according to the
finding of the Court of Appeals. Said court also found that before the insured
signed the application he, as well as the members of his family, told the agent and
the medical examiner that he had been sick and coughing for some time and that
he had gone three times to the Santol Sanatorium and had X-ray pictures of his
lungs taken; but that in spite of such information the agent and the medical
examiner told them that the applicant was a fit subject for insurance.
In September 1935, he died. His heirs filed an insurance claim but Insular
Life denied the application as it averred that Feliciano’s application was attended
by fraud. It was later found in court that the insurance agent and the medical
examiner of Insular Life who assisted Feliciano in signing the application knew that
Feliciano was already suffering from tuberculosis; that they were aware of the true
medical condition of Feliciano yet they still made it appear that he was healthy in
the insurance application form; that Feliciano signed the application in blank and
the agent filled the information for him. In Insular Life v. Feliciano 73 Phil
201, the Court held that Insular life cannot avoid the insurance policy. The weight
of authority is that if an agent of the insurer, after obtaining from an applicant for
insurance a correct and truthful answer to interrogatories contained in the
36

Insurance Case Digests
Atty. Bathan-Basuel

application for insurance, without knowledge of the applicant fills in false answers,
either fraudulently or otherwise, the insurer cannot assert the falsity of such
answers as a defense to liability on the policy, and this is true generally without
regard to the subject matter of the answers or the nature of the agent’s duties or
limitations on his authority, at least if not brought to the attention of the
applicant. Thus this motion for reconsideration.

ISSUE: Whether or not Insular life is correct in contending that Feliciano is not
entitled to the claim because the insurance policy is void ab initio; that he connived
with the insurance agent and the medical examiner; and that at best, Feliciano is only
entitled to refund or the reimbursement of what he has paid in premium

HELD: Yes. This time, the Supreme Court held that Insular Life’s contention is
correct. When Evaristo Feliciano, the applicant for insurance, signed the application in
blank and authorized the soliciting agent and/or medical examiner of Insular to write
the answers for him, he made them his own agents for that purpose, and he was
responsible for their acts in that connection. If they falsified the answers for him, he
could not evade the responsibility for the falsification. He was not supposed to sign
the application in blank. He knew that the answers to the questions therein contained
would be “the basis of the policy,” and for that very reason he was required with his
signature to vouch for truth thereof.
Although the agent and the medical examiner knew that statement to be
false, no valid contract of insurance was entered into because there was no real
meeting of the minds of the parties.
From all the facts and circumstances of the case, we are constrained to
conclude that the insured was a coparticipant, and coresponsible with Agent David
and Medical Examiner Valdez, in the fraudulent procurement of the policies in
question and that by reason thereof said policies are void ab initio.

Issue: Whether or not Insular Life was bound by their agent’s acts.
Held: Yes. The insurance business has grown so vast and lucrative within the past
century.
Nowadays, even people of modest means enter into insurance
contracts. Agents who solicit contracts are paid large commissions on the policies
secured by them. They act as general representatives of insurance companies.
IN the case at bar, the true state of health of the insured was concealed by the agents
of the insurer. The insurer’s medical examiner approved the application knowing fully
well that the applicant was sick. The situation is one in which of two innocent parties
must bear a loss for his reliance upon a third person. In this case, it is the one who
drafted and accepted the policy and consummated the contract. It seems reasonable
that as between the two of them, the one who employed and gave character to the
third person as its agent should be the one to bear the loss. Hence, Insular is liable to
the beneficiaries.
Aranilla v. Insular Life, 69 OG No. 4 637
Facts:
-

-

In 1959, Jose Aranilla applied for life insurance with Insular. In his application,
these 2 questions appeared:
WON he has suffered from any disease of the kidney and urinary tract, to
which he answered NO.
WON he has been confined in a hospital for consultation and treatment, to
which he answered that in 1947, he was confined due to influenza.
The truth however, was that a few months prior to his application, he was
confined and treated for nephritis, a disease of the kidney and urinary tract,
and he was accordingly informed of the cause.
When Aranilla died of cirrhosis of the liver, Insular refused to pay the proceeds
due to concealment.

Issue: Whether the contract can be rescinded.
Insular Life v. Feliciano 73 Phil 201
Facts:
-

-

-

Evaristo Feliciano filed an application with Insular Life upon the solicitation of
one of its agents.
It appears that during that time, Evaristo was already suffering from
tuberculosis. Such fact appeared during the medical exam, but the examiner
and the company’s agent ignored it.
After that, Evaristo was made to sign an application form and thereafter the
blank spaces were filled by the medical examiner and the agent making it
appear that Evaristo was a fit subject of insurance. (Evaristo could not read
and understand English)
When Evaristo died, Insular life refused to pay the proceeds because of
concealment.

2-S, 2013-2014

Held: Yes. If an answer given by the insured to a specific question asked by the insurer
in an application for life insurance turns out to be false, it is a concealment of a material
fact which entitles the insurer to rescind, even if the insured died of an ailment which
has NO connection with the specific questions falsely answered by him. This is because
materiality is to be determined NOT by the event but ONLY by the probable and
reasonable influence of the facts upon the party to whom the communication is due, in
forming his estimate of the disadvantages of the proposed contract or in making his
inquiries.
Ng Zee v. Asian Crusader Life Assurance Corp, 122 SCRA 461 [1983]
NG GAN ZEE vs. ASIAN CRUSADER LIFE ASSURANCE CORPORATION
Facts:
37

Insurance Case Digests
Atty. Bathan-Basuel

-

-

-

-

Kwong Nam applied for a 20-year endowment insurance on his life for the sum
of P20,000.00, with his wife, Ng Gan Zee as beneficiary. On the same date,
ACAC, upon receipt of the required premium from the insured, approved the
application and issued the corresponding policy.
Kwong Nam died of cancer of the liver with metastasis. All premiums had been
religiously paid at the time of his death.
Kwong Nam widow Ng Gan Zee presented a claim in due form to appellant for
payment of the face value of the policy. She submitted the required proof of
death of the insured. ACAC denied the claim on the ground that the answers
given by the insured to the questions appealing in his application for life
insurance were untrue.
Ng Gan Zee brought the matter to the attention of the Insurance
Commissioner, the Hon. Francisco Y. Mandamus, and the latter, after
conducting an investigation, wrote the ACAC that he had found no material
concealment on the part of the insured and that, therefore, Ng Gan Zee should
be paid the full face value of the policy. This opinion of the Insurance
Commissioner notwithstanding, ACAC refused to settle its obligation.
ACAC alleged that the insured was guilty of misrepresentation when he
answered "No" to the following question appearing in the application for life
insurance:

Has any life insurance company ever refused your application for insurance
or for reinstatement of a lapsed policy or offered you a policy different
from that applied for? If, so, name company and date.

-

-

-

-

According to ACAC
"The insured applied for reinstatement of his lapsed life insurance policy with the Insular
Life Insurance Co., Ltd, but this was declined by the insurance company, although later
on approved for reinstatement with a very high premium as a result of his medical
examination. Thus notwithstanding the said insured answered 'No' to the question
propounded to him"

-

Issue: Whether Ng Gan Zee can collect the insurance claim.
CFI: ordered ACAC to pay the face value of the insurance policy

-

Held:
-

There is no evidence that the Insular Life Assurance Co., Ltd. ever refused any
application of Kwong Nam for insurance. Neither is there any evidence that any
other insurance company has refused any application of Kwong Nam for
insurance. The evidence shows that the Insular Life Assurance Co., Ltd.
approved Kwong Nam's request for reinstatement and amendment of his
lapsed insurance policy. The Court notes from said application for
reinstatement and amendment, that the amount applied for was P20,000.00
only and not for P50,000.00 as it was in the lapsed policy. The amount of the
reinstated and amended policy was also for P20,000.00. It results, therefore,

2-S, 2013-2014

-

-

that when Kwong Nam answered 'No' to the question whether any life
insurance company ever refused his application for reinstatement of a lapsed
policy he did not misrepresent any fact
Sec. 27. Such party a contract of insurance must communicate to the other,
in good faith, all facts within his knowledge which are material to the contract,
and which the other has not the means of ascertaining, and as to which he
makes no warranty.
Thus, "concealment exists where the assured had knowledge of a fact material
to the risk, and honesty, good faith, and fair dealing requires that he should
communicate it to the assurer, but he designedly and intentionally withholds
the same." It has also been held "that the concealment must, in the absence
of inquiries, be not only material, but fraudulent, or the fact must have been
intentionally withheld."
Assuming that the aforesaid answer given by the insured is false, as claimed
by the ACAC. Sec. 27 of the Insurance Law, nevertheless requires that
fraudulent intent on the part of the insured be established to entitle the
insurer to rescind the contract. And as correctly observed by the lower court,
"misrepresentation as a defense of the insurer to avoid liability is an
'affirmative' defense. The duty to establish such a defense by satisfactory and
convincing evidence rests upon the defendant. The evidence before the Court
does not clearly and satisfactorily establish that defense."
It bears emphasis that Kwong Nam had informed the ACAC's medical examiner
that the tumor for which he was operated on was "associated with ulcer of the
stomach." In the absence of evidence that the insured had sufficient medical
knowledge as to enable him to distinguish between "peptic ulcer" and "a
tumor", his statement that said tumor was "associated with ulcer of the
stomach, " should be construed as an expression made in good faith of his
belief as to the nature of his ailment and operation. Indeed, such statement
must be presumed to have been made by him without knowledge of its
incorrectness and without any deliberate intent on his part to mislead the
appellant.
While it may be conceded that, from the viewpoint of a medical expert, the
information communicated was imperfect, the same was nevertheless
sufficient to have induced appellant to make further inquiries about the
ailment and operation of the insured.
Section 32. The right to information of material facts maybe waived either by
the terms of insurance or by neglect to make inquiries as to such facts where
they are distinctly implied in other facts of which information is communicated
It has been held that where, upon the face of the application, a question
appears to be not answered at all or to be imperfectly answered, and the
insurers issue a policy without any further inquiry, they waive the imperfection
of the answer and render the omission to answer more fully immaterial.
As aptly noted by the lower court, "if the ailment and operation of Kwong Nam
had such an important bearing on the question of whether the defendant
would undertake the insurance or not, the court cannot understand why the
defendant or its medical examiner did not make any further inquiries on such
matters from the Chinese General Hospital or require copies of the hospital
38

Insurance Case Digests
Atty. Bathan-Basuel

records from the appellant before acting on the application for insurance. The
fact of the matter is that the defendant was too eager to accept the
application and receive the insured's premium. It would be inequitable now to
allow the defendant to avoid liability under the circumstances."
5.

agreement is not an insurance contract; hence the "incontestability clause" under the
Insurance Code does not apply.
PETITIONER ARGUES: The incontestability clause does not apply, as the same requires
an effectivity period of at least two years.

Incontestability
Philamcare Health Care System v. CA GR No. 125678, March 18, 2002

ISSUES:
a.) Whether there has been a concealment.
b.) Whether the incontestability clause apply in the case at bar.
HELD:

FACTS:
Ernani Trinos, deceased husband of respondent Julita Trinos, applied for a
health care coverage with petitioner Philamcare Health Systems, Inc. In the standard
application form, he answered no to the following question:

Have you or any of your family members ever consulted or been treated for
high blood pressure, heart trouble, diabetes, cancer, liver disease, asthma or
peptic ulcer? (If Yes, give details).
The application was approved for a period of one year. Under the agreement,
respondent’s husband was entitled to avail of hospitalization benefits, whether ordinary
or emergency, listed therein. He was also entitled to avail of "out-patient benefits" such
as annual physical examinations, preventive health care and other out-patient services.
Upon the termination of the agreement, the same was extended for another year. The
amount of coverage was increased to a maximum sum of P75,000.00 per disability.
During the period of his coverage, Ernani suffered a heart attack and was confined at
the Manila Medical Center (MMC) for one month beginning March 9, 1990. While her
husband was in the hospital, respondent tried to claim the benefits under the health
care agreement. However, petitioner denied her claim saying that the Health Care
Agreement was void. According to petitioner, there was a concealment regarding
Ernani’s medical history. Doctors at the MMC allegedly discovered at the time of Ernani’s
confinement that he was hypertensive, diabetic and asthmatic, contrary to his answer in
the application form. Thus, respondent paid the hospitalization expenses herself,
amounting to about P76,000.00.
In the morning of April 13, 1990, Ernani had fever and was feeling very weak.
Respondent was constrained to bring him back to the Chinese General Hospital where he
died on the same day.
Respondent instituted with the Regional Trial Court of Manila, Branch 44, an action for
damages against petitioner and its president, Dr. Benito Reverente. She asked for
reimbursement of her expenses plus moral damages and attorney’s fees. After trial, the
lower court ruled against petitioners. On appeal, the Court of Appeals affirmed the
decision of the trial court but deleted all awards for damages and absolved petitioner
Reverente. Petitioner’s motion for reconsideration was denied. Hence, petitioner brought
the instant petition for review, raising the primary argument that a health care
2-S, 2013-2014

a.) Petitioner argues that respondent’s husband concealed a material fact in
his application. It appears that in the application for health coverage, petitioners
required respondent’s husband to sign an express authorization for any person,
organization or entity that has any record or knowledge of his health to furnish any and
all information relative to any hospitalization, consultation, treatment or any other
medical advice or examination. Specifically, the Health Care Agreement signed by
respondent’s husband states:

We hereby declare and agree that all statement and answers contained herein
and in any addendum annexed to this application are full, complete and true
and bind all parties in interest under the Agreement herein applied for, that
there shall be no contract of health care coverage unless and until an
Agreement is issued on this application and the full Membership Fee according
to the mode of payment applied for is actually paid during the lifetime and
good health of proposed Members; that no information acquired by any
Representative of PhilamCare shall be binding upon PhilamCare unless set out
in writing in the application; that any physician is, by these presents,
expressly authorized to disclose or give testimony at anytime
relative to any information acquired by him in his professional
capacity upon any question affecting the eligibility for health care
coverage of the Proposed Members and that the acceptance of any
Agreement issued on this application shall be a ratification of any correction in
or addition to this application as stated in the space for Home Office
Endorsement.
In addition to the above condition, petitioner additionally required the applicant for
authorization to inquire about the applicant’s medical history, thus:

hereby authorize any person, organization, or entity that has any record or
knowledge of my health and/or that of __________ to give to the
PhilamCare Health Systems, Inc. any and all information relative to any
hospitalization, consultation, treatment or any other medical
advice or examination. This authorization is in connection with the
application for health care coverage only. A photographic copy of this
authorization shall be as valid as the original.
39

Insurance Case Digests
Atty. Bathan-Basuel

Petitioner cannot rely on the stipulation regarding "Invalidation of agreement"
which reads:

Failure to disclose or misrepresentation of any material information by the
member in the application or medical examination, whether intentional or
unintentional, shall automatically invalidate the Agreement from the very
beginning and liability of Philamcare shall be limited to return of all Membership
Fees paid. An undisclosed or misrepresented information is deemed material if
its revelation would have resulted in the declination of the applicant by
Philamcare or the assessment of a higher Membership Fee for the benefit or
benefits applied for.

(U)nder the title Claim procedures of expenses, the defendant Philamcare
Health Systems Inc. had twelve months from the date of issuance of the
Agreement within which to contest the membership of the patient if he had
previous ailment of asthma, and six months from the issuance of the
agreement if the patient was sick of diabetes or hypertension. The periods
having expired, the defense of concealment or misrepresentation no longer lie.
Tan Chay Heng v. West Coast Life, 51 Phil 80 [1927]
Facts:
-

The answer assailed by petitioner was in response to the question relating to the
medical history of the applicant. This largely depends on opinion rather than fact,
especially coming from respondent’s husband who was not a medical doctor. Where
matters of opinion or judgment are called for, answers made in good faith and without
intent to deceive will not avoid a policy even though they are untrue.

-

The fraudulent intent on the part of the insured must be established to warrant
rescission of the insurance contract. Concealment as a defense for the health care
provider or insurer to avoid liability is an affirmative defense and the duty to establish
such defense by satisfactory and convincing evidence rests upon the provider or
insurer. In any case, with or without the authority to investigate, petitioner is liable for
claims made under the contract. Having assumed a responsibility under the agreement,
petitioner is bound to answer the same to the extent agreed upon. In the end, the
liability of the health care provider attaches once the member is hospitalized for the
disease or injury covered by the agreement or whenever he avails of the covered
benefits which he has prepaid.

-

Under Section 27 of the Insurance Code, "a concealment entitles the injured party to
rescind a contract of insurance." The right to rescind should be exercised previous to
the commencement of an action on the contract. In this case, no rescission was made.
Besides, the cancellation of health care agreements as in insurance policies require the
concurrence of the following conditions:
1. Prior notice of cancellation to insured;
2. Notice must be based on the occurrence after effective date of the policy of one or
more of the grounds mentioned;
3. Must be in writing, mailed or delivered to the insured at the address shown in the
policy;
4. Must state the grounds relied upon provided in Section 64 of the Insurance Code and
upon request of insured, to furnish facts on which cancellation is based.
None of the above pre-conditions was fulfilled in this case.
b.)
Anent the incontestability of the membership of respondent’s
husband, we quote with approval the following findings of the trial court:
2-S, 2013-2014

-

-

In 1926, Tan Chay Heng sued West Coast on the policy allegedly issued to his
“uncle”, Tan Caeng who died in 1925. He was the sole beneficiary thereof.
West Coast refused on the ground that the policy was obtained by Tan Caeng
with the help of agents Go Chuilian, Francisco Sanchez and Dr. Locsin of West
Coast.
West Coast said that it was made to appear that Tan Caeng was single, a
merchant, health and not a drug user, when in fact he was married, a laborer,
suffering form tuberculosis and addicted to drugs.
West Coast now denies liability based on these misrepresentations.
Tan Chay contends that West Coast may not rescind the contract because an
action for performance has already been filed.
Trial court found for Tan Chay holding that an insurer cannot avoid a policy
which has been procured by fraud unless he brings an action to rescind it
before he is sued thereon.

Issue: Whether or not West Coast’s action for rescission is therefore barred by the
collection suit filed by Tan Chay.
Held: NO.
Precisely, the defense of West Cast was that through fraud in its execution, the policy
is void ab initio, and therefore, no valid contract was ever made. Its action then cannot
be fore rescission because an action to rescind is founded upon and presupposes the
existence of the contract. Hence, West Coast’s defense is not barred by Sec. 47.
In the instant case, it will be noted that even in its prayer, the defendant does not seek
to have the alleged insurance contract rescinded. It denies that it ever made any
contract of insurance on the life of Tan Caeng, or that any such a contract ever existed,
and that is the question which it seeks to have litigated by its special defense. In the
very nature of things, if the defendant never made or entered into the contract in
question, there is no contract to rescind, and, hence, section 47 upon which the lower
court based its decision in sustaining the demurrer does not apply.
As stated, an action to rescind a contract is founded upon and presupposes the
existence of the contract which is sought to be rescinded. If all of the material matters
set forth and alleged in the defendant's special plea are true, there was no valid
contract of insurance, for the simple reason that the minds of the parties never met
40

Insurance Case Digests
Atty. Bathan-Basuel

and never agreed upon the terms and conditions of the contract. We are clearly of the
opinion that, if such matters are known to exist by a preponderance of the evidence,
they would constitute a valid defense to plaintiff's cause of action. Upon the question
as to whether or not they are or are not true, we do not at this time have or express
any opinion, but we are clear that section 47 does not apply to the allegations made in
the answer, and that the trial court erred in sustaining the demurrer.
Tan v. Court of Appeals, 174 SCRA 403 [1989]
Facts:
-

Tan Lee Siong was issued a policy by Philamlife on Nov. 6, 1973.
On Aprl 26, 1975, Tan died of hepatoma. His beneficiaries then filed a claim
with Philamlife for the proceeds of the insurance.
Philamlife wrote the beneficiaries in Sep. 1975 denying their claim and
rescinding the contract on the ground of misrepresentation. The beneficiaries
contend that Philamlife can no longer rescind the contract on the ground of
misrepresentation as rescission must allegedly be done “during the lifetime of
the insured” within two years and prior to the commencement of the action
following the wording of Sec. 48, par. 2.

Issue: Whether or not Philamlife can rescind the contract.
Held: YES.
The phrase “during the lifetime” found in Sec. 48 simply means that the policy is no
longer in force after the insured has died. The key phrase in the second paragraph is
“for a period of two years”.
Soliman v. US Life, 104 Phil 1046 [1958]
Facts:
-

-

US Life issued a 20 yr endowment life policy on the joint lives of Patricio
Soliman and his wife Rosario, each of them being the beneficiary of the other.
In Mar. 1949, the spouses were informed that the premium for Jan 1949 was
still unpaid notwithstanding that the 31-day grace period has already expired,
and they were furnished at the same time long-form health certificates for the
reinstatement of the policies.
In Apr 1949, they submitted the certificates and paid the premiums.
In Jan. 1950, Rosario died of acute dilation of the heart, and thereafter,
Patricio filed a claim for the proceeds of the insurance.
US life denied the claim and filed for the rescission of the contract on the
ground that the certificates failed to disclose that Rosario had been suffering
from bronchial asthma for 3 years prior to their submission.

Issue: Whether or not the contract can still be rescinded.

into the veracity of the facts represented by the insured in the application for
reinstatement. When US life sought to rescind the contract on the ground of
concealment/misrepresentation, two years had not yet elapsed. Hence, the contract
can still be rescinded.
Gonzalez Lao v. Yek Tong Lin Fire & Marine Insurance, 55 PHIL 386
EMILIO GONZALES LAO, plaintiff-appellee, vs. THE YEK TONG LIN FIRE AND
MARINE INSURANCE CO., LTD., defendant-appellant
VILLAMOR, J.:
FACTS: Plaintiff Gonzales Lao was issued 2(two) fire insurance policies by Yek for P
100,000 covering his leaf tobacco products.These products were stored in Gonzales’
building on No. 188 Soler St. Plaintiff filed action to recover of the defendant the Yek
Tong Lin Fire & Marine Insurance Co., Ltd., the amount of two insurance policies. It was
alleged that on January 11, 1928, the leaf tobacco products was damaged by the fire
that destroyed the building where said tobacco was stored, The defendant filed a
general and specific denial of each and every allegation of the complaint, set up three
special defenses, and prayed to be absolved from the complaint with costs against the
plaintiff. In Art. 3 of the Insurance policies, it was provided that: “Any insurance in force
upon all or part of the things unsured must be declared in writing by the insured and he
(insured) should cause the company to insert or mention it in the policy. Without such
requisite, such policy will be regarded as null and void and the insured will be deprived
of all rights of indemnity in case of loss." Notwithstanding said provision, Gonzales
entered into other insurance contracts. When he sought to claim from Yek after the
fire, the latter denied any liability on the ground of violation of Art. 3 of the said
policies. Plaintiff Gonzales however proved that the insurer knew of the other insurance
policies obtained by him long efore the fire, and the insurer did NOT rescind the
insurance polices in question but demanded and collected from the insured the
premiums.
ISSUE: Whether defendant Yek is still entitled to annul the contract
HELD : NO. The defendant - appellant cannot invoke the violation of article 3 of the
conditions of the insurance policies for the first time on appeal, having failed to do so in
its answer; besides, as the appellee correctly contends in his brief, Guillermo Cu Unjieng,
who was then president and majority shareholder of the appellant company, the Yek
Tong Lin Fire & Marine Insurance Co., knew that there were other insurances, at least
from the attempt to raise the insurance premium on the warehouse and the appellee's
tobacco deposited therein to 1 per centum, and it was later reduced upon petition of
the appellant itself and other assurance companies to 0.75 per centum presented to
the association of assurance companies in the year 1927, and notwithstanding this, said
appellant did not rescind the insurance policies in question, but demanded and collected
from the appellee the increased premium.

Held: Yes.
The insurer is once again given two years from the date of reinstatement to investigate
2-S, 2013-2014

41

Insurance Case Digests
Atty. Bathan-Basuel

That the defendant had knowledge of the existence of other policies obtained by the
plaintiff from other insurance companies, is specifically shown by the defendant's
answer wherein it alleges, by way of special defense, the fact that there exist other
policies issued by the companies mentioned therein. If, with the knowledge of existence
of other insurances which the defendant deemed violations of the contract, it has
preferred to continue the policy, its action amounts to a waiver of the annulment of the
contract.
Pacific Bank Corporation v. CA, 163 SCRA 1
Pacific Banking Corporation v. CA
G.R. No. L-41014 November 28, 1988
Paras, J.

At the trial, Pacific presented in evidence Exhibit "H", which is a communication dated
December 22, 1965 of the insurance adjuster, H.H. Bayne Adjustment Co. to Asian
Surety Insurance Co., Inc., revealing undeclared co-insurances with the following:
P30,000.00 with Wellington Insurance; P25,000. 00 with Empire Surety and
P250,000.00 with Asian Surety; undertaken by insured Paramount on the same
property covered by its policy with private respondent whereas the only co-insurances
declared in the subject policy are those of P30,000.00 with Malayan P50,000.00 with
South Sea and P25.000.00 with Victory
Issue:
1. WON Pacific unrevealed co-insurances which amounted to violation of policy
conditions No. 3 and
2. WON Pacific failed to file the required proof of loss prior to court action.

Facts:
On October 21,1963, Fire Policy No. F-3770, an open policy, was issued to
the
Paramount Shirt Manufacturing Co. by which Oriental Assurance Corporation bound
itself to indemnify the insured for any loss or damage, not exceeding P61,000.00,
caused by fire to its property
consisting of stocks, materials and supplies usual to a shirt factory, including furniture,
fixtures, machinery and equipment while contained in the ground, second and third
floors of the building situated at number 256 Jaboneros St., San Nicolas, Manila, for a
period of one year commencing from that date to October 21, 1964.
Paramount was at the time of the issuance of the policy and is up to this time,
a debtor of
Pacific Banking Corp (Pacific) in the amount of not less P800,000.00 and the goods
described in the policy were held in trust by the insured for the PBC under thrust
receipts. Said policy was duly endorsed to Pacific as mortgagee/ trustor of the
properties insured, with the knowledge and consent of Oriental to the effect that "loss
if any under this policy is payable to the Pacific. While the aforesaid policy was in full
force and effect, a fire broke out on the subject premises destroying the goods
contained in its ground and second floors. The counsel for the Pacific sent a letter of
demand to Oriental for indemnity due to the loss of property by fire under the
endorsement of said policy However, Oriental informed counsel for the Pacific that it
was not yet ready to accede to the latter's demand as the former is awaiting the final
report of the insurance adjuster, H.H. Bayne Adjustment Company. Later, the said
insurance adjuster notified counsel for the Pacific that the insured under the policy had
not filed any claim with it, nor submitted proof of loss which is a clear violation of Policy
Condition No.11, and for which reason, determination of the liability of Oriental could
not be had. Pacific's counsel replied to aforesaid letter asking the insurance adjuster to
verify from the records of the Bureau of Customs the entries of merchandise taken into
the customs bonded warehouse razed by fire as a reliable proof of loss. For failure of
the insurance company to pay the loss as demanded, Pacific filed in the court a quo an
action for a sum of money against the private respondent, Oriental Assurance
Corporation, in the principal sum of P61,000.00 issued in favor of Paramount Shirt
Manufacturing Co. ).
2-S, 2013-2014

Policy Condition No. 3 explicitly provides:
3. The Insured shall give notice to the Company of any insurance
already effected, or which may subsequently be effected, covering
any of the property hereby insured, and unless such notice be given
and the particulars of such insurance or insurances be stated in or
endorsed on this Policy by or on behalf of the Company before the
occurrence of any loss or damage, all benefit under this policy shall
be forfeited. (Record on Appeal, p. 12)
Held:
1. Yes, It is not disputed that the insured failed to reveal before the loss three other
insurances.
As found by the Court of Appeals, by reason of said unrevealed insurances,
the insured had been guilty of a false declaration; a clear misrepresentation and a vital
one because where the insured had been asked to reveal but did not, that was
deception. Otherwise stated, had the insurer known that there were many coinsurances, it could have hesitated or plainly desisted from entering into such contract.
Hence, the insured was guilty of clear fraud
Petitioner's contention that the allegation of fraud is but a mere inference or
suspicion is untenable. In fact, concrete evidence of fraud or false declaration by the
insured was furnished by the petitioner itself when the facts alleged in the policy under
clauses "Co-Insurances Declared" and "Other Insurance Clause" are materially different
from the actual number of co-insurances taken over the subject property.
Consequently, "the whole foundation of the contract fails, the risk does not attach and
the policy never becomes a contract between the parties. Representations of facts are
the foundation of the contract and if the foundation does not exist, the superstructure
does not arise. Falsehood in such representations is not shown to vary or add to the
contract, or to terminate a contract which has once been made, but to show that no
contract has ever existed.
Undoubtedly, it is but fair and just that where the insured who is primarily
entitled to receive the proceeds of the policy has by its fraud and/or misrepresentation,
42

Insurance Case Digests
Atty. Bathan-Basuel

forfeited said right, with more reason petitioner which is merely claiming as indorsee of
said insured, cannot be entitled to such proceeds.

VIRGINIA A. PEREZ, petitioner , vs . COURT OF APPEALS and BF LIFEMAN
INSURANCE CORPORATION, respondents .

2. Yes.

G.R. No. 112329. January 28, 2000]

As the insurance policy against fire expressly required that notice should be
given by the insured of other insurance upon the same property, the total absence of
such notice nullifies the policy. The argument that notice of co-insurances may be made
orally is preposterous and negates policy condition No. 20 which requires every notice
and other communications to the insurer to be written or printed.
The evidence adduced shows that twenty-four (24) days after the fire,
petitioner merely wrote letters to private respondent to serve as a notice of loss,
thereafter, the former did not furnish the latter whatever pertinent documents were
necessary to prove and estimate its loss. Instead, petitioner shifted upon private
respondent the burden of fishing out the necessary information to ascertain the
particular account of the articles destroyed by fire as well as the amount of loss. It is
noteworthy that private respondent and its adjuster notified petitioner that insured had
not yet filed a written claim nor submitted the supporting documents in compliance
with the requirements set forth in the policy. Despite the notice, the latter remained
unheedful. Since the required claim by insured, together with the preliminary submittal
of relevant documents had not been complied with, it follows that private respondent
could not be deemed to have finally rejected petitioner's claim and therefore the
latter's cause of action had not yet arisen. Compliance with condition No. 11 is a
requirement sine qua non to the right to maintain an action as prior thereto no
violation of petitioner's right can be attributable to private respondent. This is so, as
before such final rejection, there was no real necessity for bringing suit. Petitioner
should have endeavored to file the formal claim and procure all the documents, papers,
inventory needed by private respondent or its adjuster to ascertain the amount of loss
and after compliance await the final rejection of its claim. Indeed, the law does not
encourage unnecessary litigation.
Verily, petitioner prematurely filed Civil Case No. 56889 and dismissal thereof
was warranted under the circumstances. While it is a cardinal principle of insurance law
that a policy or contract of insurance is to be construed liberally in favor of the insured
and strictly as against the insurer company yet, contracts of insurance, like other
contracts, are to be construed according to the sense and meaning of the terms which
the parties themselves have used.
It appearing that insured has violated or failed to perform the conditions under
No. 3 and 11 of the contract, and such violation or want of performance has not been
waived by the insurer, the insured cannot recover, much less the herein petitioner.
Courts are not permitted to make contracts for the parties; the function and duty of
the courts is simply to enforce and carry out the contracts actually made.
6.

The Policy
Perez v. CA, 323 SCRA 613 (2000)

2-S, 2013-2014

FACTS:
Primitivo B. Perez had been insured with the BF Lifeman Insurance Corporation since
1980 for P20,000.00. Sometime in October 1987, an agent of the insurance
corporation, Rodolfo Lalog, visited Perez in Guinayangan, Quezon and convinced him to
apply for additional insurance coverage of P50,000.00, to avail of the ongoing
promotional discount of P400.00 if the premium were paid annually.
On October 20, 1987, Primitivo B. Perez accomplished an application form for the
additional insurance coverage of P50,000.00. On the same day, petitioner Virginia A.
Perez, Primitivo’s wife, paid P2,075.00 to Lalog. The receipt issued by Lalog indicated
the amount received was a "deposit." Unfortunately, Lalog lost the application form
accomplished by Perez and so he asked the latter to fill up another application form. On
November 1, 1987, Perez was made to undergo the required medical examination,
which he passed.
Pursuant to the established procedure of the company, Lalog forwarded the application
for additional insurance of Perez, together with all its supporting papers, to the office of
BF Lifeman Insurance Corporation at Gumaca, Quezon which office was supposed to
forward the papers to the Manila office.
On November 25, 1987, Perez died in an accident. He was riding in a banca which
capsized during a storm. At the time of his death, his application papers for the
additional insurance of P50,000.00 were still with the Gumaca office. Lalog testified
that when he went to follow up the papers, he found them still in the Gumaca office and
so he personally brought the papers to the Manila office of BF Lifeman Insurance
Corporation. It was only on November 27, 1987 that said papers were received in
Manila. Without knowing that Perez died on November 25, 1987, BF Lifeman Insurance
Corporation approved the application and issued the corresponding policy for
the P50,000.00 on December 2, 1987.
Petitioner Virginia Perez went to Manila to claim the benefits under the insurance
policies of the deceased. She was paid P40,000.00 under the first insurance policy
for P20,000.00 (double indemnity in case of accident) but the insurance company
refused to pay the claim under the additional policy coverage of P50,000.00, the
proceeds of which amount to P150,000.00 in view of a triple indemnity rider on the
insurance policy. In its letter of January 29, 1988 to Virginia A. Perez, the insurance
company maintained that the insurance for P50,000.00 had not been perfected at the
time of the death of Primitivo Perez. Consequently, the insurance company refunded
the amount of P2,075.00 which Virginia Perez had paid.

43

Insurance Case Digests
Atty. Bathan-Basuel

"there shall be no contract of insurance unless and until a
policy is issued on this application and that the said policy
shall not take effect until the premium has been paid and the
policy delivered to and accepted by me/us in person while
I/We, am/are in good health."

Then, private respondent BF Lifeman Insurance Corporation filed a complaint against
Virginia A. Perez seeking the rescission and declaration of nullity of the insurance
contract in question.
Petitioner Virginia A. Perez, on the other hand, averred that the deceased had fulfilled
all his prestations under the contract and all the elements of a valid contract are
present. She then filed a counterclaim against private respondent for the collection
of P150,000.00
as
actual
damages, P100,000.00
as
exemplary
damages, P30,000.00 as attorney’s fees and P10,000.00 as expenses for litigation.
The RTC of Manila ruled in favor of Virginia. The premium for the additional insurance of
P50,000.00 had been fully paid and even if the sum of P2,075.00 were to be
considered merely as partial payment, the same does not affect the validity of the
policy. The trial court further stated that the deceased had fully complied with the
requirements of the insurance company. He paid, signed the application form and
passed the medical examination. He should not be made to suffer the subsequent delay
in the transmittal of his application form to private respondent’s head office since these
were no longer within his control.

The assent of private respondent BF Lifeman Insurance Corporation therefore
was not given when it merely received the application form and all the requisite
supporting papers of the applicant. Its assent was given when it issues a corresponding
policy to the applicant. Also, it when Primitivo died on November 25, 1987, his
application papers for additional insurance coverage were still with the branch office of
respondent corporation in Gumaca and it was only two days later when Lalog personally
delivered the application papers to the head office in Manila. Consequently, there was
absolutely no way the acceptance of the application could have been communicated to
the applicant for the latter to accept inasmuch as the applicant at the time was already
dead.

2.
The SC did not agree. In the case at bar, the following conditions were
imposed by the respondent company for the perfection of the contract of insurance:
The CA reversed the decision of the trial court and held that the insurance contract for
P50,000.00 could not have been perfected since at the time that the policy was issued,
Primitivo was already dead. The Court of Appeals held that the contract of insurance
had to be assented to by both parties and so long as the application for insurance has
not been either accepted or rejected, it is merely an offer or proposal to make a
contract.

1.

a policy must have been issued;

2.

the premiums paid; and

3.

the policy must have been delivered to and accepted by the applicant while he
is in good health.

ISSUES:
-

Whether, as between Primitivo and BF Lifeman Insurance Corp, there was a
perfected contract of insurance.

-

Whether the condition that the policy issued by the corporation be delivered
and received by the applicant in good health, is potestative, being dependent
upon the will of the insurance company, and is therefore null and void.

HELD:
1.
None. When Primitivo filed an application for insurance, paid P2,075.00 and
submitted the results of his medical examination, his application was subject to the
acceptance of private respondent BF Lifeman Insurance Corporation. The perfection of
the contract of insurance between the deceased and respondent corporation was
further conditioned upon compliance with the following requisites stated in the
application form:

2-S, 2013-2014

The condition imposed by the corporation that the policy must have been
delivered to and accepted by the applicant while he is in good health can hardly be
considered as a potestative or facultative condition. On the contrary, the health of the
applicant at the time of the delivery of the policy is beyond the control or will of the
insurance company. Rather, the condition is a suspensive one whereby the acquisition of
rights depends upon the happening of an event which constitutes the condition. In this
case, the suspensive condition was the policy must have been delivered and accepted
by the applicant while he is in good health. There was non-fulfillment of the condition,
however, inasmuch as the applicant was already dead at the time the policy was issued.
Hence, the non-fulfillment of the condition resulted in the non-perfection of the
contract.

As stated above, a contract of insurance, like other contracts, must be assented to by
both parties either in person or by their agents. So long as an application for insurance
has not been either accepted or rejected, it is merely an offer or proposal to make a
contract. The contract, to be binding from the date of application, must have been a
44

Insurance Case Digests
Atty. Bathan-Basuel

completed contract, one that leaves nothing to be done, nothing to be completed,
nothing to be passed upon, or determined, before it shall take effect. There can be no
contract of insurance unless the minds of the parties have met in agreement.

Commissioner of Internal Revenue v. Lincoln Philippine Life Insurance Co., 379
SCRA423

COMMISSIONER OF INTERNAL REVENUE v. LINCOLN PHILIPPINE LIEF
INSURANCE COMPANY INC. (now JARDINE-CMA LIFE INSURANCE COMPANY,
INC.)
Lincoln Philippine Life Insurance Co., Inc., (now Jardine-CMA Life Insurance Company,
Inc.) is a domestic corporation engaged in life insurance business. In the years prior to
1984, it issued a special kind of life insurance policy known as the “Junior Estate Builder
Policy,” the distinguishing feature of which is a clause providing for an automatic
increase in the amount of life insurance coverage upon attainment of a certain age by
the insured without the need of issuing a new policy. The clause was to take effect in
the year 1984.
That year, the insurance company also issued shares of stock dividends. Documentary
stamp taxes were paid based only on the par value. Subsequently, the Commissioner on
Internal Revenue issued deficiency documentary stamps tax assessment for the year
1984 in the amounts corresponding to the amount of automatic increase of the sum
assured on the policy issued by respondent, and another corresponding to the book
value in excess of the par value of the stock dividends. The insurance company
questioned the deficiency assessments before the Court of Tax Appeals.
The CTA found no valid basis for the deficiency tax assessment on the stock dividends,
as well as on the insurance policy. The CIR appealed the decision to the Court of
Appeals, which affirmed the CTA decision insofar as it nullified the deficiency
assessment on the insurance policy, but reversing the same with regard to the
deficiency assessment on the stock dividends.
Both parties appealed. The insurance company questions the validity of the deficiency
tax assessment on stock dividends. The CIR, meanwhile, questioned the decision
invalidating the deficiency assessment on the insurance policy. The CIR claims that the
automatic increase clause in the subject insurance policy is separate and distinct from
the main agreement and involves another transaction, and that, while no new policy was
issued, the original policy was essentially re-issued when the additional obligation was
assumed upon the effectivity of this “automatic increase clause” in 1984. Hence, a
deficiency assessment based on the additional insurance not covered in the main policy
is in order.

2-S, 2013-2014

The Court of Appeals sustained the CTA’s ruling that there was only one transaction
involved in the issuance of the insurance policy and that the “automatic increase
clause” is an integral part of that policy.
ISSUE
Whether or not there was a separate transaction
RULING
The subject insurance policy at the time it was issued contained an “automatic increase
clause.” Although the clause was to take effect only in 1984, it was written into the
policy at the time of its issuance. The distinctive feature of the “junior estate builder
policy” called the “automatic increase clause” already formed part and parcel of the
insurance contract, hence, there was no need for an execution of a separate agreement
for the increase in the coverage that took effect in 1984 when the assured reached a
certain age.
It is clear from Section 173 of the NIRC that the payment of documentary stamp taxes
is done at the time the act is done or transaction had and the tax base for the
computation of documentary stamp taxes on life insurance policies under Section 183
is the amount fixed in policy, unless the interest of a person insured is susceptible of
exact pecuniary measurement. What then is the amount fixed in the policy? Logically,
we believe that the amount fixed in the policy is the figure written on its face and
whatever increases will take effect in the future by reason of the “automatic increase
clause” embodied in the policy without the need of another contract.
Here, although the automatic increase in the amount of life insurance coverage was to
take effect later on, the date of its effectivity, as well as the amount of the increase,
was already definite at the time of the issuance of the policy. Thus, the amount insured
by the policy at the time of its issuance necessarily included the additional sum covered
by the automatic increase clause because it was already determinable at the time the
transaction was entered into and formed part of the policy.
The “automatic increase clause” in the policy is in the nature of a conditional obligation
under Article 1181 by which the increase of the insurance coverage shall depend upon
the happening of the event which constitutes the obligation. In the instant case, the
additional insurance that took effect in 1984 was an obligation subject to a suspensive
obligation, but still a part of the insurance sold to which private respondent was liable
for the payment of the documentary stamp tax.
The deficiency of documentary stamp tax imposed on private respondent is definitely
not on the amount of the original insurance coverage, but on the increase of the
amount insured upon the effectivity of the “Junior Estate Builder Policy.”
Finally, it should be emphasized that while tax avoidance schemes and arrangements are
not prohibited,10 tax laws cannot be circumvented in order to evade the payment of
just taxes. In the case at bar, to claim that the increase in the amount insured (by
virtue of the automatic increase clause incorporated into the policy at the time of
45

Insurance Case Digests
Atty. Bathan-Basuel

issuance) should not be included in the computation of the documentary stamp taxes
due on the policy would be a clear evasion of the law requiring that the tax be
computed on the basis of the amount insured by the policy.WHEREFORE, the petition is
hereby given DUE COURSE.

-

Lim v. Sun Life Assurance Co of Canada 41 Phil 265
Facts:
-

-

-

On July 6, 1917, Luis Lim Y Garcia of Zamboanga applied for a policy of life
insurance with Sunlife in the amount of 5T.
He designated his wife Pilar Lim as the beneficiary. The first premium of P433
was paid by Lim and company issued a “provisional policy”
Such policy contained the following provisions “xx the abovementioned life is
to be assured in accordance with the terms and conditions contained or
inserted by the Company in the policy which may be granted by it in this
particular case for 4 months only from the date of the application, PROVIDED
that the company shall confirm this agreement by issuing a policy on said
application xxx. Should the company NOT issue such a policy, then this
agreement shall be null and void ab initio and the Company shall be held not to
have been on the risk at all, but in such case, the amount herein shall be
returned.
Lim died on Aug. 23, 1917 after the issuance of the provisional policy but
before the approval of the application by the home office of the insurance
company.
The instant action is brought by the beneficiary to recover from Sun Life the
sum of 5T.

-

-

-

Issue: Whether or not the beneficiary can collect the 5T.

-

Held: NO.
The contract of insurance was not consummated by the parties. The above quoted
agreement clearly stated that the agreement should NOT go into effect until the home
office of the Company shall confirm it by issuing a policy. It was nothing but an
acknowledgment by the Company that it has received a sum of money agreed upon as
the first year’s premium upon a policy to be issued upon the application if it is accepted
by the Company.

-

When an agreement is made between the applicant and the agent whether by signing an
application containing such condition or otherwise, that no liability shall attach until the
principal approves the risk and a receipt is given by the agent, such acceptance is
merely conditional and is subordinated to the company’s act in approving or rejecting;
so in life insurance a “binding slip or receipt” does not insure itself.
Enriquez v Sun Life Assurance Co 41 Phil 269

RAFAEL ENRIQUEZ v. SUN LIFE ASSURANCE COMPANY OF CANADA
2-S, 2013-2014

Rafael Enriquez, administrator of the estate of the late Joaquin Herrera made
application to the Sun Life Assurance Company of Canada for a life annuity.
The application was immediately forwarded to the head office of the company
in Canada, which gave notice of acceptance by cable to Manila. The policy was
issued at Montreal on December 4, 1917. However, on December 18, Herrera
informed the Manila office that he desired to withdraw his application. The
local office replied to Torres stating that the policy had been issued. This
letter was received by the attorney on the morning of December 21. Mr.
Herrera died on December 20, 1917.
According to the SC, the letter of November 26, 1917, notifying Mr. Herrera
that his application had been accepted, was prepared and signed in the local
office of the insurance company, was placed in the ordinary channels for
transmission, but as far as the SC knows was never actually mailed and thus
was never received by the applicant.
RULING
The law applicable to the case is found to be the second paragraph of article
1262 of the Civil Code providing that an acceptance made by letter shall not
bind the person making the offer except from the time it came to his
knowledge.
The pertinent fact is, that according to the provisional receipt, three things
had to be accomplished by the insurance company before there was a
contract: (1) There had to be a medical examination of the applicant; (2) there
had to be approval of the application by the head office of the company; and
(3) this approval had in some way to be communicated by the company to the
applicant.
The further admitted facts are that the head office in Montreal did accept the
application, did cable the Manila office to that effect, did actually issue the
policy and did, through its agent in Manila, actually write the letter of
notification and place it in the usual channels for transmission to the
addressee.
The fact as to the letter of notification thus fails to concur with the essential
elements of the general rule pertaining to the mailing and delivery of mail
matter as announced by the American courts, namely, when a letter or other
mail matter is addressed and mailed with postage prepaid there is a rebuttable
presumption of fact that it was received by the addressee as soon as it could
have been transmitted to him in the ordinary course of the mails. But if any
one of these elemental facts fails to appear, it is fatal to the presumption. For
instance, a letter will not be presumed to have been received by the addressee
unless it is shown that it was deposited in the post-office, properly addressed
and stamped.
We hold that the contract for a life annuity in the case at bar was not
perfected because it has not been proved satisfactorily that the acceptance of
the application ever came to the knowledge of the applicant.

Development Bank of the Phils v CA 231 SCRA 370
46

Insurance Case Digests
Atty. Bathan-Basuel

Development Bank of the Philippines v CA - March 21, 1994
Facts:
Juan B. Dans, together with his family applied for a loan of P500,000 with DBP. As
principal mortgagor, Dans, then 76 years of age was advised by DBP to obtain a
mortgage redemption insurance (MRI) with DBP MRI pool. A loan in the reduced amount
was approved and released by DBP. From the proceeds of the loan, DBP deducted the
payment for the MRI premium. The MRI premium of Dans, less the DBP service fee of
10%, was credited by DBP to the savings account of DBP MRI-Pool. Accordingly, the
DBP MRI Pool was advised of the credit.
Dans died of cardiac arrest. DBP MRI Pool notified DBP that Dans was not eligible for MRI
coverage, being over the acceptance age limit of 60 years at the time of application.
DBP apprised Candida Dans of the disapproval of her late husband’s MRI application. DBP
offered to refund the premium which the deceased had paid, but Candida Dans refused
to accept the same demanding payment of the face value of the MRI or an amount
equivalent of the loan. She, likewise, refused to accept an ex gratia settlement which
DBP later offered. Hence the case at bar.
Issue:
Whether or not the DBP MRI Pool should be held liable on the ground that the contract
was already perfected?
Held:
No, it is not liable. The power to approve MRI application is lodged with the DBP MRI
Pool. The pool, however, did not approve the application. There is also no showing that
it accepted the sum which DBP credited to its account with full knowledge that it was
payment for the premium. There was as a result no perfected contract of insurance’
hence the DBP MRI Pool cannot be held liable on a contract that does not exist
In dealing with Dans, DBP was wearing 2 legal hats: the first as a lender and the second
as an insurance agent. As an insurance agent, DBP made Dans go through the motion of
applying for said insurance, thereby leading him and his family to believe that they had
already fulfilled all the requirements for the MRI and that the issuance of their policy
was forthcoming. DBP had full knowledge that the application was never going to be
approved. The DBP is not authorized to accept applications for MRI when its clients are
more than 60 years of age. Knowing all the while that Dans was ineligible, DBP exceeded
the scope of its authority when it accepted the application for MRI by collecting the
insurance premium and deducting its agent’s commission and service fee. Since the
2-S, 2013-2014

third person dealing with an agent is unaware of the limits of the authority conferred by
the principal on the agent and he has been deceived by the non-disclosure thereof by
the agent, then the latter is liable for damages to him.
Ang Giok Chip v. Springfield 56 Phil 375.
FACTS: Ang Giok Chip doing business under the name and style of Hua Bee
Kong Si was formerly the owner of a warehouse in Manila.The contents of the
warehouse were insured with three insurance companies for the total sum of
P60,000. One insurance policy, in the amount of P10,000, was taken out with the
Springfield Fire & Marine Insurance Company. The warehouse was destroyed by fire
on January 11, 1928, while the policy issued by the latter company was in force.
Ang Giok instituted an action to recover the insurance proceeds from Springfield.
Springfield contended that Ang GIok violated warranty F in the insurance policy
because the warehouse contained 39% hazardous material when said warranty only
allows 3% hazardous material of the total value of the goods stored in the
warehouse.
"WARRANTY F
"It is hereby declared and agreed that during the currency
of this policy no hazardous goods be stored in the Building to
which this insurance applies or in any building communicating
therewith, provided, always, however, that the Insured be
permitted to store a small quantity of the hazardous goods
specified below, but not exceeding in all 3 per cent of the total
value of the whole of the goods or merchandise contained in said
warehouse, viz;. . . "

Ang Giok contends that Warranty F is in the nature of a rider included in
an insurance policy which is null and void because the same does not comply with
Section 65 of the Insurance Act which provides that, "Every express warranty,
made at or before the execution of a policy, must be contained in the policy itself,
or in another instrument signed by the insured and referred to in the policy, as
making a part of it."
ISSUE: Whether a warranty referred to in the policy as forming part of the
contract of insurance and in the form of a rider to the insurance policy, is null and
void?
HELD: NO. Based on prevailing doctrines, it is well settled that a rider
attached to a policy is a part of the contract, to the same extent and with like
effect as if actually embodied therein. It is equally well settled that an express
warranty must appear upon the face of the policy, or be clearly incorporated
therein and made a part thereof by explicit reference, or by words clearly
evidencing such intention. In the case at bar, the rider, warranty F, is contained in
the policy itself, because by the contract of insurance agreed to by the parties it is
made to form a part of the same, but is not another instrument signed by the
insured and referred to in the policy as forming a part of it.
47

Insurance Case Digests
Atty. Bathan-Basuel

Moreover, it is admitted that the policy before us was accepted by the
plaintiff. The receipt of this policy by the insured without objection binds both the
acceptor and the insured to the terms thereof. The insured may not thereafter be
heard to say that he did not read the policy or know its terms, since it is his duty
to read his policy and it will be assumed that he did so.
We have here a standard insurance policy and the issuance of the policy in
this case with its attached rider conforms to well established practice in the
Philippines and elsewhere. To nullify such policies would place an unnecessary
hindrance in the transactions of insurance business in the Philippines. These are
matters of public policy.
Sindayen v Insular Life 62 Phil 9
Facts:
-

-

-

-

-

-

-

-

-

Fortunata thereby releases forever and discharges Insular from any and all
claims and obligations she may have against the latter.
A check for the above-mentioned amount was drawn in the name of Fortunata,
but the same was never encashed.
Instead, it was returned to Insular and this complaint to enforce payment
under the policy was instituted.
The application which Sindayen signed in Camiling contained the following
provisions:
“xxx
(3) That the said policy shall not take effect until the first premium has been
paid and the policy has been delivered to and accepted by me, while I am in
good health.”
o

Arturo Sindayen was a linotype operator in the Bureau of Printing. He and his
wife Fortunat went to Camiling to spend Christmas with his aunt Felicidad
Estrada.
On Dec. 26, 1932, while still in Camiling, he made a written application to
Insular Life, through its agent, Cristobal Hendoza, for a policy of insurance on
his life in the sum of 1,000.
He paid the agent P15 as part of the first premium. It was agreed that the
policy, when and if issued, should be delivered to Felicidad with whom
Sindayen left the sum P25.06 to complete the payment of the first annual
premium of P40.06.
On Jan 1, 1933, Sindayen was examined by Insular’s doctor who made a
favorable report to Insular.
The next day, Sindayen returned to Manila and resumed his work. On Jan. 11,
1933, Insular accepted the risk and issued a policy, and mailed the same to its
agent for delivery to the insured.
On Jan. 12, 1933, Sindayen complained of a severe headache. ON Jan. 15,
1933, he called a physician who found that Sindayen was suffering from acute
nephritis and uremia. His illness did not yield to treatment and on Jan. 19,
1933, he died.
The policy which the company issued and mailed in manila on Jan. 11 1933
was received by its agent in Camilin on Jan. 16, 1933. On Jan 18, 1933, the
agent, in accordance with his agreement with the insured delivered the policy
to Felicided upon her payment of the balance of the 1st year’s premium.
The agent asked Felicidad if her nephew was in good health and she replied
that she believed so because she had no information that he was sick, and
thereupon , the policy was handed to her by the agent.
On Jan. 20, 1933, the agent learned of the death of Sindayen, afterwhich he
called upon Felicidad and asked her to return the policy. Felicidad did so.
On Feb. 4, 1933, the company obtained from Sindayen’s widow Fortunata
(also the beneficiary), her signature on a legal document whereby in
consideration of the sum 40.06 representing the amount of premium paid,

2-S, 2013-2014

The main defense of the company is the policy never took effect
because of par. 3 of the application, since at the time of the delivery
of the agent, the insured was not in good health.

Issue: Whether or not the policy took effect.
Held: YES.
There is one line of American cases which holds that the stipulation contained par. 3 is
in the nature of a condition precedent, that is to say, that there can be no valid delivery
to the insured unless he is in good health at that time; that this condition precedent
goes to the very essence of the contract and cannot be waived by the agent making
delivery of the policy; HOWEVER, there is also a number of American decision which
state the contrary.
These decisions say that an agent to whom a life insurance policy (similar to the one at
bar) was sent with instruction to deliver it to the insured, has authority to bind the
company by making such delivery, ALTHOUGH the insured was NOT in good health at
the time of delivery, on the theory that the delivery of the policy being the final act to
the consummation of the contract, the condition as to the insured’s good health was
WAIVED by the company.
These same cases further hold that the delivery of the policy by the agent to the
insured consummates the contract even though the agent knew that the insured was
NOT in good health at the time, the theory being, that his knowledge is the company’s
knowledge; and his delivery is the company’s delivery; that when the delivery is made
notwithstanding this knowledge of the defect, the company is deemed to have WAIVED
such defect.
The agent, Mendoza was duly licensed by the Insurance Commission to act for Insular
Life. He had the authority given by him by the company to withhold the delivery of the
policy to the insured until the first premium has been paid and the policy has been
delivered to and accepted by the insured while he is in good health. Whether that
condition had been met or not plainly calls for the exercise of discretion. Mendoza’s
decision that the condition had been met by the insured and that it was proper to make
48

Insurance Case Digests
Atty. Bathan-Basuel

delivery of the policy to him is just as binding on the company as if the decision had
been made by its Board of Directors. Admittedly, Mendoza made a mistake of
judgment because he acted on insufficient evidence as to the state of health of the
insured, and this mistake cannot be said to be induced by any misconduct on the part
of the insured.

against occurred does not militate against the validity of Pacific’s contention, for no
such premium could have been paid, since by the nature of the cover note, it did not
contain, as all cover notes do not contain, particulars of the shipment that would serve
as basis for the computation of the premiums. As a logical consequence, no separate
premiums are required to be paid on a cover note.

It is in the interest of not only of the applicant but of all insurance companies as well
that there should be some act which gives the applicant the definite assurance that the
contract has been consummated. This sense of security and of piece of mind that
one’s dependents are provided for without risk of either loss or of litigation is the
bedrock of life insurance.

If the note is to be treated as a separate policy instead of integrating it to the regular
policies subsequently issued, its purpose would be meaningless for it is in a real sense a
contract, not a mere application.

A cloud will be thrown over the entire insurance business if the condition of health of
the insured at the time of the delivery of the policy may be inquired into years
afterwards with the view of avoiding the policy on the ground that it never took effect
because of an alleged lack of good health at the time of delivery.

GLORIA V. PHILAMLIFE INSURANCE CO.
73 OG 8660
Facts:
In 1966, Roberto Narito applied for a 100T life insurance policy with Philamlife
Insurance Company. Narito was examined by Dra. Vergel de dios, the insurer’s
medical examiner.
She opined that Narito was insurable. Her opinion was confirmed by Dr.
Orobia, the Associate Medical Director of the insurer.
On Oct. 31, 1966, an agent of the insured prepared an application for the life
insurance whose annual premium was P1,178. On the same date, the
application was signed by Narito.
Narito paid the first annual premium on the policy applied for. The insurer’s
application form contained a so-called “Binding Receipt” which was detachable.
It is not sure whether or not Narito was given the Binding Receipt upon his
payment of the first premium, but what is certain is that he was handed a
Cashier’s Receipt.
From the time the insured received the application from its agent on Nov. 5,
1966, up to Dec. 6, 1966, it did not take any action with regard to the
controverted insurance coverage.
On Dec. 6, 1966, Narito was shot and killed. The beneficiaries submitted a
claim to the insurer. After an underwriting analysis conducted by the insurer,
it found out that Narito was unacceptable as an insurance risk. The claim was
denied.

It is therefore in the public interest that we are constrained to hold, as we
do, that the delivery of the policy to the insured by an agent of the
company who is authorized to make delivery or withhold delivery is the
final act which binds the company and the insured, in the absence of fraud
or other legal grounds for rescission. The fact that the agent to whom it has
entrusted this duty is derelict or negligent or even dishonest in the performance of the
duty which has been entrusted to him would create an obligation based upon the
authorized acts of the agent toward a third party who was not in collusion with the
agent.
Pacific Timber Export Corporation v. CA 112 SCRA 199
Facts:
-

-

-

On March 13, 1963, Pacific secured temporary insurance from the
Workemen’s Insurance Co. for its exportation of logs to Japan. Workmen
issued on said date Cover Note 1010 insuring said cargo.
The regular marine policies were issued by the company in favor of Pacific on
Apr 2, 1963. The 2 marine policies bore the number 53H01032 and
53H01033.
After the issuance of the cover note but BEFORE the issuance of the 2
policies, some of the logs intended to be exported were lost due to a typhoon.
Pacific filed its claim with the company, but the latter refused, contending that
said loss may not be considered as covered under the cover note because
such became null and void by virtue of the issuance of the marine policies.

Issue: Whether or not the cover not was without consideration, thus null and void.
Held: It was with consideration.
SC upheld Pacific’s contention that said cover not was with consideration. The fact
that no separate premium was paid on the cover note before the loss was insured
2-S, 2013-2014

Gloria v. Philamlife Insurance Co., 73 OG 8660

Issue: Whether or not

the beneficiaries can claim.

Held: YES.
The application for insurance signed by the deceased contained the following
stipulation: “The binding receipt must NOT be issued unless a binding deposit is paid
which must be at least equal to the first full premium.” The preponderance of evidence
is to the effect that the binding receipt was not issued to the deceased when he paid
the company’s agent, the first annual premium of P1,178. Hence the rights of the
beneficiaries and the obligation of the company have to be determined solely in the
application for insurance and in the Cashier’s receipt.
49

Insurance Case Digests
Atty. Bathan-Basuel

The application for insurance contained the following clause:“There shall be no contract
of insurance unless a policy is issued on this application and the full first premium
thereon actually paid.” It should be conceded that there shall be a contract of
insurance once the first premium is paid and a policy is issued. There is no question
that the first premium was paid.
The problem is to resolve whether or not it can be said that the policy has been
issued. IN this connection, what may be noted is that, in contrast to the requirement
of actual payment of the premium, it was NOT required that the policy be actually
issued. And assuming that no policy had indeed been issued, it should still be held that
the application for insurance was approved by the company, with the actual issuance of
the policy being a mere technicality. When an insurer accepts and retains the first
premium for an unreasonable length of time, it should be presumed that the insurer had
assumed the risk. It should therefore be liable for loss before the application is
subsequently rejected. In the case at bar, the company did NOT act on the application
for insurance, one way or the other, from Nov. 2 to Dec. 5, 1966, and no justification
for the delay had been proven.
Hence, it should be held that the application for insurance of the deceased had been
approved prior to his death, although the policy had not actually been issued, for which
reason, the company should be liable to the beneficiaries.
Del Val v Del Val
FRANCISCO DEL VAL, ET AL. vs. ANDRES DEL VAL
GR NO. 9374. FEBRUARY 16, 1915
MORELAND
FACTS: Plaintiffs and defendants are brothers and sisters, the only heirs at law and next
of kin of Gregorio Nacianceno Del Val who died in Manila on August 4, 1910 intestate.
An administrator was appointed for the estate of the decease, and, after a partial
administration, it was closed and the administrator discharged by order of CFI Manila
dated December 9, 1911.
During the lifetime of the deceased, he took out insurance on his life for P40,000 and
made it payable to the defendant as sole beneficiary.
After his death, the defendant collected the face of the policy. Of said policy, he paid
P18,365.20 to redeem certain real estate which the decedent had sold to third persons
with a right to repurchase. The redemption of said premises was made by the attorney
of the defendant in the name of the plaintiffs and the defendant as heirs of the
deceased vendor. The redemption in the name of the plaintiffs was, so defendant
declares, without his knowledge or consent. Since the redemption of said premises, they
have been in the possession of the plaintiffs who have had the use and benefit thereof.
During that time, plaintiffs paid no taxes and made no repairs.

2-S, 2013-2014

Further, the defendant, on the death of the deceased, took possession of most of his
personal property, which he still has in his possession, and that he has also the balance
on said insurance policy amounting to P21,634.80.
Plaintiffs contend that the amount of the insurance policy belonged to the estate of the
deceased and not to the defendant personally, therefore, they are entitled to a partition
not only of the real and personal property, but also of the P40,000 life insurance.
The
left
the
the

complaint prays a partition of all the property, both real and personal,
by the deceased, that the defendant account for P21,634.80 and that
sum be divided equally among the plaintiffs and defendant along with
other property of the deceased.

The defendant denies the material allegations of the complaint and sets up as special
defense and counterclaim that the redemption of the real estate sold by his father was
made in the name of the plaintiffs and himself instead of in his name alone without his
knowledge or consent and that it was not his intention to use the proceeds of the
insurance policy for the benefit of any person but himself alleging that he was and is the
sole owner thereof and it is his individual property. He asks that he be declared the
owner of the real estate redeemed by the payment of the P18,365.20, the owner of
the remaining P21,634.80, the balance of the insurance policy, and that the plaintiffs
account for the use and occupation of the premises so redeemed since the date of the
redemption.
CFI Manila refused to give relief to either party and dismissed the action due to the
complaint’s failure to comply with Code. Civ. Pro. Sec. 183 because it does not contain
an adequate description of the real property of which partition is demanded; and since
it is an exclusive real property action, the institution thereof gives the court no
jurisdiction over chattels but no relief could possibly be granted in this action as to any
property except the last (real estate), for the law contemplated that all the personal
property of an estate be distributed before the administration is closed. Indeed, it is
only in exceptional cases that the partition of the real estate is provided for, and this
too is evidently intended to be effected as a part of the administration, but here the
complaint alleges that the estate was finally closed on December 9, 1911, and we find
upon referring to the record in that case that subsequent motions to reopen the same
were denied; so that the matter of the distribution of the personal property at least
must be considered res judicata (for the final judgment in the administration
proceedings must be treated as concluding not merely what was adjudicated, but what
might have been). So far, therefore, as the personal property at least is concerned,
plaintiffs' only remedy was an appeal from said order.

NOTE: Even though the complaint is defective to the extent of failing in allegations
necessary to constitute a cause of action, if, on the trial of the cause, evidence is
offered which establishes the cause of action which the complaint intended to allege,
and such evidence is received without objection, the defect is thereby cured and cannot
be made the ground of a subsequent objection. If, therefore, evidence was introduced
on the trial in this case definitely and clearly describing the real estate sought to be
50

Insurance Case Digests
Atty. Bathan-Basuel

partitioned, the defect in the complaint was cured in that regard and should not have
been used to dismiss the action. Courts do not stop to inquire whether such evidence
was or was not introduced on the trial, inasmuch as this case must be returned for a
new trial with opportunity to both parties to present such evidence as is necessary to
establish their respective claims.
The courts of the Islands have jurisdiction to divide personal property between the
common owners thereof and that power is as full and complete as is the power to
partition real property. If an actual partition of personal property cannot be made it will
be sold under the direction of the court and the proceeds divided among the owners
after the necessary expenses have been deducted.
The administration of the estate of the decedent consisted simply in the payment of
the debts. No division of the property, either real or personal, seems to have been
made. On the contrary, the property appears to have been turned over to the heirs in
bulk. The failure to partition the real property may have been due either to the lack of
request to the court by one or more of the heirs to do so, as the court has no authority
to make a partition of the real estate without such request; or it may have been due to
the fact that all the real property of decedent had been sold under pacto de retro and
that, therefore, he was not the owner of any real estate at the time of his death. As to
the personal property, it does not appear that it was disposed of in the manner
provided by law. (Sec. 753, Code of Civil Procedure.) So far as this action is concerned,
however, it is sufficient that none of the property was actually divided among the heirs
in the administration proceedings and that they remain co-owners and tenants-incommon thereof at the present time. To maintain an action to partition real or personal
property it is necessary to show only that it is owned in common.

“An heir by force of law surviving with others of the same character to a succession
must bring into the hereditary estate the property or securities he may have received
from the deceased during the life of the same, by way of dowry, gift, or for any good
consideration, in order to compute it in fixing the legal portions and in the account of
the division."
Counsel also claims that the proceeds of the insurance policy were a donation or gift
made by the father during his lifetime to the defendant and that, as such, its ultimate
destination is determined by those provisions of the Civil Code which relate to
donations, especially Art. 819 which provides that "gifts made to children which are not
betterments shall be considered as part of their legal portion”.
HELD: YES. The proceeds of the life- insurance policy belong exclusively to
the defendant as his individual and separate property.
That the proceeds of an insurance policy belong exclusively to the beneficiary and not
to the estate of the person whose life was insured, and that such proceeds are the
separate and individual property of the beneficiary, and not of the heirs of the person
whose life was insured, is the doctrine in America. The same doctrine obtains in the
Islands by virtue of Section 428 of the Code of Commerce, which reads:

"The amounts which the underwriter must deliver to the person insured, in fulfillment of
the contract, shall be the property of the latter, even against the claims of the
legitimate heirs or creditors of any kind whatsoever of the person who effected the
insurance in favor of the former."

The order finally closing the administration and discharging the administrator, referred
to in the opinion of the trial court, has nothing to do with the division of either the real
or the personal property. The heirs have the right to ask the probate court to turn over
to them both the real and personal property without division; and where that request is
unanimous it is the duty of the court to comply with it, and there is nothing in section
753 of the Code of Civil Procedure which prohibits it. In such case an order finally
settling the estate and discharging the administrator would not bar a subsequent action
to require a division of either the real or personal property. If, on the other hand, an
order had been made in the administration proceedings dividing the personal or the real
property, or both, among the heirs, then it is quite possible that, to a subsequent
action brought by one of the heirs for a partition of the real or personal property, or
both, there could have been interposed a plea of res judicata based on such order. As
the matter now stands, however, there is no ground on which to base such a plea.
Moreover, no such plea has been made and no evidence offered to support it.

The contract of life insurance is a special contract and the destination of
the proceeds thereof is determined by special laws which deal exclusively
with that subject. The Civil Code has no provisions which relate directly
and specifically to life-insurance contracts or to the destination of life
insurance proceeds. That subject is regulated exclusively by the Code of
Commerce which provides for the terms of the contract, the relations of
the parties and the destination of the proceeds of the policy.

ISSUE: Whether the proceeds of the life- insurance policy belong exclusively to the
defendant as his individual and separate property.

This cannot hold true unless the fact appear or be shown that the defendant acted as
he did with the intention that the other heirs should enjoy with him the ownership of
the estate—in other words, that he proposed, in effect, to make a gift of the real
estate to the other heirs. If it is established by the evidence that that was his intention
and that the real estate was delivered to the plaintiffs with that understanding, then it

The attorney for the plaintiffs claims that Sec. 428 of the Code of Commerce just q is
subordinated to the provisions of the Civil Code as found in Art. 1035, which reads:
2-S, 2013-2014

The proceeds of the life-insurance policy being the exclusive property of the defendant
and he having used a portion thereof in the repurchase of the real estate sold by the
decedent prior to his death with right to repurchase, and such repurchase having been
made and the conveyance taken in the names of all of the heirs instead of the
defendant alone, plaintiffs claim that the property belongs to the heirs in common and
not to the defendant alone.

51

Insurance Case Digests
Atty. Bathan-Basuel

is probable that their contention is correct and that they are entitled to share equally
with the defendant therein. If, however, it appears from the evidence in the case that
the conveyances were taken in the name of the plaintiffs without his knowledge or
consent, or that it was not his intention to make a gift to them of the real estate, then
it belongs to him. If the f acts are as stated, he has two remedies. The one is to compel
the plaintiffs to reconvey to him and the other is to let the title stand with them and to
recover from them the sum he paid on their behalf.
The case was REMANDED to the trial court with instructions to permit the parties to
frame such issues as will permit the settlement of all the questions involved and to
introduce such evidence as may be necessary for the full determination of the issues
framed. Upon such issues and evidence taken thereunder the court will decide the
questions involved according to the evidence, subordinating his conclusions of law to
the rules laid down.
Bonifacio Bros. v. Mora, 20 SCRA 262
BONIFACIO BROS., INC., ET AL vs. ENRIQUE MORA, ET AL.
Enrique Mora, owner of an Oldsmobile sedan model 1956 mortgaged the same to the
H.S. Reyes, Inc., with the condition that the former would insure the automobile with
the latter as beneficiary. The automobile was thereafter insured with the State Bonding
& Insurance Co, Inc., and a motor car insurance was to Mora. The insurer obligated itself
to indemnify the Insured against loss of or damages to the Motor Vehicle and its
accessories and spare parts whilst thereon; (a) by accidental collision or overturning or
collision or overturning consequent upon mechanical breakdown or consequent upon
wear and tear.
The Company may pay in cash the the amount the loss or damage or may repair,
reinstate, or replace the Motor Vehicle or any part thereof or its accessories or spare
parts. The liability of the Company shall not exceed the value of the parts whichever is
the less. The Insured's estimate of value stated in the schedule will be the maximum
amount payable by the Company in respect of any claim for loss or damage.
The Insured may authorize the repair of the Motor Vehicle necessitated by damage for
which the Company may be liable under this Policy provided that:—(a) The estimated
cost of such repair does not exceed the Authorized Repair Limit, (b) A detailed estimate
of the cost is forwarded to the Company without delay, subject to the condition that
'Loss, if any, is payable to H.S. Reyes, Inc..', by virtue of the fact that said Oldsmobile
sedan was mortgaged in favor of the said H.S. Reyes, Inc. and that under a clause in
said insurance policy, any loss was made payable to the H.S, Reyes, Inc. as Mortgagee;
During the effectivity of the insurance contract, the car met with an accident. The
insurance company then assigned the accident to the H.H. Bayne Adjustment Co. for
investigation and appraisal of the damage. Enrique Mora, without the knowledge and
consent of the H.S. Reyes, Inc., authorized the Bonifacio Bros. Inc. to furnish the labor
and materials, some of which were supplied by the Ayala Auto Parts Co. For the cost of
2-S, 2013-2014

labor and materials, Enrique Mora was billed at P2,102.73 through the H.H. Bayne
Adjustment Co. The insurance company, after claiming a franchise in the amount of
?100, drew a check in the amount of P2,002.73, as proceeds of the insurance policy,
payable to the order of Enrique Mora or H.S. Reyes, Inc., and entrusted the check to the
H.H. Bayne Adjustment Co. for disposition and delivery to the proper party. In the
meantime, the car was delivered to Enrique Mora without the consent of the H.S, Reyes,
Inc., and without payment to the Bonifacio Bros. Inc. and the Ayala Auto Parts Co. of
the cost of repairs and materials,
Upon the theory that the insurance proceeds should be paid directly to them, the
Bonifacio Bros. Inc. and the Ayala Auto Parts Co. a complaint with the against Enrique
Mora and the State Bonding & Insurance Co., Inc. for the collection of the sum of
P2,002.73. The insurance company filed its answer with a counterclaim for interpleader,
requiring the Bonifacio Bros. Inc. and the H.S. Reyes, Inc. to interplead in order to
determine who has better right to the insurance proceeds in question. Enrique Mora was
declared in default for failure to appear at the hearing, and evidence against him was
received ex parte. However, the counsel for the Bonifacio Bros. Inc., Ayala Auto Parts
Co. and State Bonding & Insurance Co. Inc. submitted a stipulation of facts, on the basis
of which the Municipal Court rendered a decision declaring the H.S. Reyes, Inc. as having
a better right to the disputed amount, and ordering the State Bonding & Insurance Co.
Inc. to pay to the H.S. Reyes, Inc. the said sum of P2,002.73. From this decision, the
herein appellants elevated the case to the Court of First Instance of Manila before which
the stipulation of facts was reproduced. The latter court rendered a decision, affirming
the decision of the Municipal Court. The Bonifacio Bros. Inc. and the Ayala Auto Parts
Co. moved for reconsideration of the decision, but the trial court denied the motion.
Hence, this appeal.
ISSUE
Whether or not there is a privity of contract between Bonifacio Bros and Ayala Auto
Parts on one hand and the insurance company on the other
RULING
The appellants argue that the insurance company and Enrique Mora are parties to the
repair of the car as well as the towage thereof performed. The authority for this
assertion is to be found, it is alleged, in paragraph 4 of the insurance contract which
provides that "the insured may authorize the repair of the Motor Vehicle necessitated
by damage for which the company may be liable under the policy provided that (a) the
estimated cost of such repair does not exceed the Authorized Repair Limit, and (b) a
detailed estimate of the cost is forwarded to the company without delay." It is stressed
that the H.H. Bayne Adjustment Company's recommendation of payment of the
appellants' bill for materials and repairs for which the latter drew a check for P2,002.73
indicates that Mora and the H.H. Bayne Adjustment Co. acted for and in representation
of the insurance company.
This argument is, in our view, beside the point, because from the undisputed facts and
from the pleadings it will be seen that the appellants' alleged cause of action rests
exclusively upon the terms of the insurance contract. The appellants seek to recover
52

Insurance Case Digests
Atty. Bathan-Basuel

the insurance proceeds, and for this purpose, they rely upon paragraph 4 of the
insurance contract document executed by and between the State Bonding & Insurance
Company, Inc. and Enrique Mora. The appellants are not mentioned in the contract as
parties thereto; nor is there any clause or provision thereof from which we can infer
that there is an obligation on the part of the insurance company to pay the cost of
repairs directly to them. It is fundamental' that contracts take effect only between the
parties thereto, except in some specific instances provided by law where the contract
contains some stipulation in favor of a third person. Such stipulation is known as
stipulation pour autrui or a provision in favor of a third person not a party to the
contract. Under this doctrine, a third person is allowed to avail himself of a benefit
granted to him by the terms of the contract, provided that the contracting parties have
clearly and deliberately conferred a favor upon such person. Consequently, a third
person not a party to the contract has no action against the parties thereto, and
cannot generally demand the enforcement of the same. The question of whether a
third person has an enforcible interest in a contract must be settled by determining
whether the contracting parties intended to tender him such an interest by deliberately
inserting terms in their agreement with the avowed purpose of conferring a favor upon
such third person. In this connection, this Court has laid down the rule that the fairest
test to determine whether the interest of a third person in a contract is a stipulation
pour autrui or merely an incidental interest, is to rely upon the intention of the parties
as disclosed by their contract. In the instant case the insurance contract does not
contain any words or clauses to disclose an intent to give any benefit to any repairmen
or materialmen in case of repair of the car in question. The parties to the insurance
contract omitted such stipulation, which is a circumstance that supports the said
conclusion. On the other hand, the "loss payable" clause of the insurance policy
stipulates that "Loss, if any, is payable to H.S. Reyes, Inc." indicating that it was only
the H.S. Reyes, Inc. which they intended to benefit.
We likewise observe from the brief of the State Bonding & Insurance Company that it
has vehemently opposed the assertion or pretension of the appellants that they are
privy to the contract. If it were the intention of the insurance company to make itself
liable to the repair shop or materialmen, it could have easily inserted in the contract a
stipulation to that effect. To hold now that the original parties to the insurance
contract intended to confer upon the appellants the benefit claimed by them would
require us to ignore the indispensable requisite that a stipulation pour autrui must be
clearly expressed by the parties, which we cannot do.
As regards paragraph 4 of the insurance contract, a perusal thereof would show that
instead of establishing privity between the appellants and the insurance company, such
stipulation merely establishes the procedure that the insured has to follow in order to
be entitled to indemnity for repair. This paragraph therefore should not be construed as
bringing into existence in favor of the appellants a right of action against the insurance
company as such intention can never be inferred therefrom.
Another cogent reason for not recognizing a right of action by the appellants against
the insurance company is that "a policy of insurance is a distinct and independent
contract between the insured and insurer, and third persons have no right either in a
2-S, 2013-2014

court of equity, or in a court of law, to the proceeds of it, unless there be some
contract or trust, expressed or implied between the insured and third person." In this
case, no contract of trust, expressed or implied exists. We, therefore, agree with the
trial court that no cause of action exists in favor of the appellants in so far as the
proceeds of insurance are concerned. The appellants' claim, if at all, is merely equitable
in nature and must be made effective through Enrique Mora who entered into a contract
with the Bonifacio Bros. Inc. This conclusion is deducible not only from the principle
governing the operation and effect of insurance contracts in general, but is clearly
covered by the express provisions of section 50 of the Insurance Act which read:
"The insurance shall be applied exclusively to the proper interests of the person in
whose name it is made unless otherwise specified in the policy."
The policy in question has been so framed that "Loss, if any, is payable to H.S. Reyes,
Inc.," which unmistakably shows the intention of the parties.
The final contention of the appellants is that the right of the H.S. Reyes, Inc. to the
insurance proceeds arises only if there was loss and not where there is mere damage as
in the instant case. Suffice it to say that any attempt to draw a distinction between
"loss" and "damage" is uncalled for, because the word "loss" in insurance law embraces
injury or damage.
Coquia v. Fieldmen’s Insurance, 26 SCRA 172
FACTS:
1. Fieldmen's Insurance Company, Inc. issued, in favor of the Manila Yellow Taxicab
Co., Inc. a common carrier accident insurance policy, for a period of one year. It
was stipulated in said policy that:

The Company will, subject to the Limits of Liability and under the Terms of
this Policy, indemnify the Insured in the event of accident caused by or arising
out of the use of Motor Vehicle against all sums which the Insured will become
legally liable to pay in respect of: Death or bodily injury to any fare-paying
passenger including the Driver, Conductor and/or Inspector who is riding in the
Motor Vehicle insured at the time of accident or injury.

2.

Thereafter, a taxicab of the Insured, driven by Carlito Coquia, met a vehicular
accident at Mangaldan, Pangasinan, in consequence of which Carlito died.

3.

Thus, The Manila Yellow Taxicab filed therefor a claim for P5,000.00 to which the
Fieldmen’s Insurance replied with an offer to pay P2,000.00, by way of
compromise. The Insured rejected the same and made a counter-offer for
P4,000.00, but the Fieldman did not accept it.
53

Insurance Case Digests
Atty. Bathan-Basuel

4.
5.

Hence,Yellow Taxicab and Carlito's parents (Melecio Coquia and Maria Espanueva)
filed a complaint against the Fieldmen’s Insurance to collect the proceeds of the
policy.
ANSWER OF Fieldmen’s Insurance: admitted the existence of the policy, but
pleaded lack of cause of action on the part of the plaintiffs (parents).

Decision of TC: sentencing the Fieldmen’s Insurance to pay to the plaintiffs the sum of
P4,000.00 and the costs.
Contentions of Fieldmen’s Insurance before C.A - contends that plaintiffs (parents)
have no cause of action because: 1) the Coquias have no contractual relation with them
and 2) the Insured has not complied with the provisions of the policy concerning
arbitration.
ISSUES: Whether the plaintiffs (parents) have a cause of action against Fielmen’s
Insurance.
Whether the insured has complied with the provisions of the policy
concerning arbitration.
HELD:
(1)YES.
It should be noted that, although, in general, only parties to a contract may bring an
action based thereon, this rule is subject to exceptions, one of which is found in the
second paragraph of Article 1311 of the Civil Code:

If a contract should contain some stipulation in favor of a third person, he may
demand its fulfillment provided he communicated his acceptance to the obligor
before its revocation. A mere incidental benefit or interest of a person is not
sufficient. The contracting parties must have clearly and deliberately conferred
a favor upon a third person.
This is but the restatement of a well-known principle concerning contracts pour autrui,
the enforcement of which may be demanded by a third party for whose benefit it was
made, although not a party to the contract, before the stipulation in his favor has been
revoked by the contracting parties.
Pursuant to these stipulations, the Company "will indemnify any authorized Driver who
is driving the Motor Vehicle" of the Insured and, in the event of death of said driver, the
Company shall, likewise, "indemnify his personal representatives." In fact, the Company
"may, at its option, make indemnity payable directly to the claimants or heirs of
claimants ... it being the true intention of this Policy to protect ... the liabilities of the
2-S, 2013-2014

Insured towards the passengers of the Motor Vehicle and the Public" — in other words,
third parties.
Thus, the policy under consideration is typical of contracts pour autrui, this character
being made more manifest by the fact that the deceased driver paid fifty percent of the
corresponding premiums, which were deducted from his weekly commissions. Under
these conditions, it is clear that the Coquias — who, admittedly, are the sole heirs of
the deceased — have a direct cause of action against the Company, and, since they
could have maintained this action by themselves, without the assistance of the Insured,
it goes without saying that they could and did properly join the latter in filing the
complaint herein.

(2) BOTH PARTIES WAIVED SUCH PROVISION.
The second defense set up by the Company is based upon Section 17 of the policy
reading:

If any difference or dispute shall arise with respect to the amount of the
Company's liability under this Policy, the same shall be referred to the decision
of a single arbitrator to be agreed upon by both parties or failing such
agreement of a single arbitrator, to the decision of two arbitrators, one to be
appointed in writing by each of the parties within one calendar month after
having been required in writing so to do by either of the parties and in case of
disagreement between the arbitrators, to the decision of an umpire who shall
have been appointed in writing by the arbitrators before entering on the
reference and the costs of and incident to the reference shall be dealt with in
the Award. And it is hereby expressly stipulated and declared that it shall be a
condition precedent to any right of action or suit upon this Policy that the
award by such arbitrator, arbitrators or umpire of the amount of the
Company's liability hereunder if disputed shall be first obtained.
The record shows, however, that none of the parties to the contract invoked this
section, or made any reference to arbitration, during the negotiations preceding the
institution of the present case. In fact, counsel for both parties stipulated, in the trial
court, that none of them had, at any time during said negotiations, even suggested the
settlement of the issue between them by arbitration, as provided in said section. Their
aforementioned acts or omissions had the effect of a waiver of their respective right to
demand an arbitration.
The test for determining whether there has been a waiver in a particular case is stated
by the author of an exhaustive annotation in 117 A.L.R. p. 304, as follows: "Any
conduct of the parties inconsistent with the notion that they treated the arbitration
provision as in effect, or any conduct which might be reasonably construed as showing
54

Insurance Case Digests
Atty. Bathan-Basuel

that they did not intend to avail themselves of such provision, may amount to a waiver
thereof and estop the party charged with such conduct from claiming its benefits".
The decisive facts here are that both parties from the inception of their dispute
proceeded in entire disregard of the provisions of the contract relating to arbitration
and that neither at any stage of such dispute, either before or after commencement of
the action, demanded arbitration, either by oral or written demand, pleading, or
otherwise. Their conduct was as effective a rejection of the right to arbitrate as if, in
the best Coolidge tradition, they had said, "We do not choose to arbitrate". As
arbitration under the express provisions of article 40 was "at the choice of either
party," and was chosen by neither, a waiver by both of the right to arbitration followed
as a matter of law.
Guingon v. Del Monte 20 SCRA 1043
GUINGON v DEL MONTE

FACTS: Julio Aguilar owned and operated several jeepneys in the City of Manila. He
entered into a contract with the Capital Insurance & Surety Co., Inc. insuring the
operation of his jeepneys against accidents with third-party liability. The pertinent
provisions of which in so far as this case is concerned contains the following:
Section II —LIABILITY TO THE PUBLIC
1. The Company, will, subject to the limits of liability, indemnify the Insured in
the event of accident caused by or arising out of the use of the Motor
Vehicle/s or in connection with the loading or unloading of the Motor
Vehicle/s, against all sums including claimant's costs and expenses which the
Insured shall become legally liable to pay in respect of:
a. death of or bodily injury to any person
b. damage to property
During the effectivity of such insurance policy on February 20, 1961 Iluminado del
Monte, one of the drivers of the jeepneys operated by Aguilar, while driving along the
intersection of Juan Luna and Moro streets, City of Manila, bumped with the jeepney
one Gervacio Guingon who had just alighted from another jeepney and as a
consequence the latter died some days thereafter.
A corresponding information for homicide thru reckless imprudence was filed against
Iluminado del Monte, who pleaded guilty.
2-S, 2013-2014

Heirs of Gervacio Guingon filed an action for damages praying that the sum of
P82,771.80 be paid to them jointly and severally by Iluminado del Monte, owner and
operator Julio Aguilar, and the Capital Insurance & Surety Co.. Capital Insurance &
Surety Co., Inc. answered, alleging that the plaintiff has no cause of action against it.
the Court of First Instance of Manila rendered its judgment sentencing defendants liable.
The case was appealed to the Court of Appeals which appellate court certified the case
to the SC because the appeal raises purely questions of law.
ISSUE: Plaintiffs not being parties to the insurance contract, do they have a cause of
action against the company;
HELD: YES.

The policy contains a clause stating:
E. Action Against Company
No action shall lie against the Company unless, as a condition precedent
thereto, the Insured shall have fully complied with all of the terms of this
Policy, nor until the amount of the Insured's obligation to pay shall have been
finally determined either by judgment against the Insured after actual trial or
by written agreement of the Insured, the claimant, and the Company.
The policy in the present case, as aforequoted, is one whereby the insurer agreed to
indemnify the insured "against all sums . . . which the Insured shall become
legally liable to pay in respect of: a. death of or bodily injury to any person . . . ."
Clearly, therefore, it is one for indemnity against liability;1 from the fact then that the
insured is liable to the third person, such third person is entitled to sue the
insurer.1äwphï1.ñët
the test applied has been this: Where the contract provides for indemnity
against liability to third persons, then third persons to whom the insured is liable, can
sue the insurer. Where the contract is for indemnity against actual loss or payment,
then third persons cannot proceed against the insurer, the contract being solely to
reimburse the insured for liability actually discharged by him thru payment to third
persons, said third persons' recourse being thus limited to the insured alone.2
The "no action" clause in the policy of insurance cannot prevail over the Rules of Court
provision aimed at avoiding multiplicity of suits.
RCBC v. CA, 289 SCRA 292 (1998)
55

Insurance Case Digests
Atty. Bathan-Basuel

Rizal Commercial Banking Corporation (RCBC) vs. Court of Appeals [GR
128833, 20 April 1998];
Facts:
Goyu & Sons, Inc. (Goyu) applied for credit facilities and accommodations with
Rizal Commercial Banking Corporation (RCBC) at its Binondo Branch. After due
evaluation, RCBC Binondo Branch, through its key officers, petitioners Uy Chun Bing and
Eli D. Lao, recommended Goyu's application for approval by RCBC's executive
committee. A credit facility in the amount of P30 million was initially granted. Upon
Goyu's application and Uy's and Lao's recommendation, RCBC's executive committee
increased Goyu's credit facility to P50 million, then to P90 million, and finally to P117
million.
As security for its credit facilities with RCBC, Goyu executed two real estate
mortgages and two chattel mortgages in favor of RCBC, which were registered with the
Registry of Deeds at Valenzuela, Metro Manila. Under each of these four mortgage
contracts, Goyu committed itself to insure the mortgaged property with an insurance
company approved by RCBC, and subsequently, to endorse and deliver the insurance
policies to RCBC. Goyu obtained in its name a total of 10 insurance policies from MICO.
In February 1992, Alchester Insurance Agency, Inc., the insurance agent
where Goyu obtained the Malayan insurance policies, issued 9 endorsements in favor of
RCBC seemingly upon instructions of Goyu. On 27 April 1992, one of Goyu's factory
buildings in Valenzuela was gutted by fire. Consequently, Goyu submitted its claim for
indemnity on account of the loss insured against. MICO denied the claim on the ground
that the insurance policies were either attached pursuant to writs of
attachments/garnishments issued by various courts or that the insurance proceeds
were also claimed by other creditors of Goyu alleging better rights to the proceeds than
the insured.
Goyu filed a complaint for specific performance and damages which was
docketed at the Regional Trial Court of the National Capital Judicial Region (Manila,
Branch 3) as Civil Case 93-65442. RCBC, one of Goyu's creditors, also filed with MICO
its formal claim over the proceeds of the insurance policies, but said claims were also
denied for the same reasons that AGCO denied Goyu's claims. In an interlocutory order
dated 12 October 1993, the Regional Trial Court of Manila (Branch 3), confirmed that
Goyu's other creditors, namely, Urban Bank, Alfredo Sebastian, and Philippine Trust
Company obtained their respective writs of attachments from various courts, covering
an aggregate amount of P14,938,080.23, and ordered that the proceeds of the 10
insurance policies be deposited with the said court minus the aforementioned
P14,938,080.23. Accordingly, on 7 January 1994, MICO deposited the amount of
P50,505,594.60 with Branch 3 of the Manila RTC.
In the meantime, another notice of garnishment was handed down by another
Manila RTC sala (Branch 28) for the amount of P8,696,838.75. After trial, Branch 3 of
the Manila RTC rendered judgment in a favor of Goyu, ordering Malayan to pay Goyu its
fire loss claims in the total amount of P74,040,518.58 less the amount of
P50,000,000.00 which is deposited with the Court; damages by way of interest for the
duration of the delay since 27 July 1992 (90 days after Malayan's receipt of the
required proof of loss and notice of loss) at the rate of twice the ceiling prescribed by
the Monetary Board, on the amounts of (1) P50,000,000.00 from 27 July 1992 up to
the time said amount was deposited with the Court on 7 January 1994; and (2)
2-S, 2013-2014

P24,040,518.58 — from 17 July 1992 up to the time when the writs of attachments
were received by Malayan. The court also ordered RCBC to pay Goyu actual and
compensatory damages in the amount of P2,000,000.00, and both Malayan and RCBC
to solidarily pay Goyu (1) P1,000,000.00 as exemplary damages; (2) P1,000,000.00
as, and for, attorneys fees; and (3) Costs of suit. The Court, on the Counterclaim of
RCBC, ordered Goyu to pay its loan obligations with RCBC in the amount of
P68,785,069.04, as of 27 April 1992, with interest thereon at the rate stipulated in
the respective promissory notes (without surcharges and penalties).
From this judgment, all parties interposed their respective appeals.
Issue: Whether RCBC, as mortgagee, has any right over the insurance policies taken by
Goyu, the mortgagor, in case of the occurrence of loss
Held: YES. It is settled that a mortgagor and a mortgagee have separate and distinct
insurable interests in the same mortgaged property, such that each one of them may
insure the same property for his own sole benefit. There is no question that Goyu could
insure the mortgaged property for its own exclusive benefit. Herein, although it appears
that Goyu obtained the subject insurance policies naming itself as the sole payee, the
intentions of the parties as shown by their contemporaneous acts, must be given due
consideration in order to better serve the interest of justice and equity. It is to be
noted that nine endorsement documents were prepared by Alchester in favor of RCBC.
The Court is in a quandary how Alchester could arrive at the idea of endorsing any
specific insurance policy in favor of any particular beneficiary or payee other than the
insured had not such named payee or beneficiary been specifically disclosed by the
insured itself.
It is also significant that Goyu voluntarily and purposely took the insurance
policies from MICO, a sister company of RCBC, and not just from any other insurance
company. Alchester would not have found out that the subject pieces of property were
mortgaged to RCBC had not such information been voluntarily disclosed by Goyu itself.
Had it not been for Goyu, Alchester would not have known of Goyu's intention of
obtaining insurance coverage in compliance with its undertaking in the mortgage
contracts with RCBC, and verify, Alchester would not have endorsed the policies to
RCBC had it not been so directed by Goyu. On equitable principles, particularly on the
ground of estoppel, the Court is constrained to rule in favor of mortgagor RCBC.
RCBC, in good faith, relied upon the endorsement documents sent to it as this
was only pursuant to the stipulation in the mortgage contracts. Such reliance is justified
under the circumstances of the case. Goyu failed to seasonably repudiate the authority
of the person or persons who prepared such endorsements. Over and above this, Goyu
continued, in the meantime, to enjoy the benefits of the credit facilities extended to it
by RCBC. After the occurrence of the loss insured against, it was too late for Goyu to
disown the endorsements for any imagined or contrived lack of authority of Alchester
to prepare and issue said endorsements. If there had not been actually an implied
ratification of said endorsements by virtue of Goyu's inaction in this case, Goyu is at the
very least estopped from assailing their operative effects.
To permit Goyu to capitalize on its non- confirmation of these endorsements
while it continued to enjoy the benefits of the credit facilities of RCBC which believed in
good faith that there was due endorsement pursuant to their mortgage contracts, is to
56

Insurance Case Digests
Atty. Bathan-Basuel

countenance grave contravention of public policy, fair dealing, good faith, and justice.
Such an unjust situation, the Court cannot sanction. Under the peculiar circumstances,
the Court is bound to recognize RCBC's right to the proceeds of the insurance policies if
not for the actual endorsement of the policies, at least on the basis of the equitable
principle of estoppel.

On February 12, 1959, plaintiffs filed a reply to the above answer of the Fulton Fire
Insurance, alleging that on May 11, 1956, plaintiffs had instituted Civil Case No. 2949 in
the Court of First Instance of Manila, to assert the claim; that this case was dismissed
without prejudice on September 3, 1957 and that deducting the period within which
said action was pending, the present action was still within the 12 month period from
April 12, 1956.

Ang v. Fulton Fire Ins Co 2 SCRA 945
The trial court held that the bringing of the action tolled the prescription ruling:
FACTS: On September 9, 1953, defendant Fulton Fire Insurance Company issued a
policy No. F-4730340, in favor of P. & S Department Store (Sally C. Ang) over stocks of
general merchandise, consisting principally of dry goods, contained in a building
occupied by the plaintiffs at Laoag, Ilocos Norte.
The premium is P500.00 annually. The insurance was issued for one year, but the same
was renewed for another year on September 31, 1954. On December 17, 1954, the
store containing the goods insured was destroyed by fire. On December 30, following,
plaintiffs executed the first claim form. The claim together with all the necessary papers
relating thereto, were forwarded to the Manila Adjustment Company, the defendants'
adjusters and received by the latter on Jane 8, 1955.

The complaint, Exh. 'C', was dismissed by the Court without prejudice (Exh. 'H1') on September 3, 1957, and motion for reconsideration dated September
21, 1957. The instant complaint was filed on May 8, 1958. As correctly
pointed out by the plaintiffs' counsel, by simple mathematical computation,
the present action was filed leas thin nine (9) months after the notice of
rejection received by plaintiffs on April 19, 1956, because the filing of the
original complaint stopped the running of the period."
The clause subject of the issue is paragraph 13 of the policy, which reads as follows:
13. If the claim be in any respect fraudulent, or if any false declaration is made
or used in support thereof, or if any fraudulent means or devices are used by
the Insured or any one acting on his behalf to obtain any benefit under this
Policy, or, if the loss or damage be occasioned by the willful act or with
connivance of the Insured, or, if the claim be made and rejected and an action
or suit be not commenced within twelve months after such rejection or (in
case of arbitration place in pursuance of the 18th condition of this Policy)
within twelve months after the arbitrator or arbitrators or umpire shall have
made their award, all benefits under this Policy shall be forfeited. (Emphasis
supplied). (Decision. p. 10, R.O.A.).

On April 6, 1956, the Fulton Fire Insurance Company wrote the plaintiffs that their claim
was denied. This denial of the claim was received by the plaintiffs on April 19, 1956.
On January 13, 1955, plaintiff Paulo Ang and ten others were charged for arson but
they were subsequently acquitted.
The present action was instituted on May 5, 1958.
The action was originally instituted against both the Fulton Fire Insurance Company and
the Paramount Surety and Insurance Company, Inc., but on June 16, 1958, upon motion
of the Paramount Surety, the latter was dropped from the complaint.
On May 26, 1958, the defendant Fulton Fire Insurance Company filed an answer to the
complaint, admitting the existence of the contract of insurance, its renewal and the loss
by fire of the department store and the merchandise contained therein, but denying
that the loss by the fire was accidental, alleging that it was occasioned by the willful act
of the plaintiff Paulo Ang himself.
It claims that under paragraph 13 of the policy, if the loss or damage is occasioned by
the willful act of the insured, or if the claim is made and rejected but no action is
commenced within 12 months after such rejection, all benefits under the policy would
be forfeited, and that since the claim of the plaintiffs was denied and plaintiffs received
notice of denial on April 18, 1956, and they brought the action only on May 5, 1958, all
the benefits under the policy have been forfeited.
2-S, 2013-2014

HELD:
The basic error committed by the trial court is its view that the filing of the action
against the agent of the defendant company was "merely a procedural mistake of no
significance or consequence, which may be overlooked." The condition contained in the
insurance policy that claims must be presented within one year after rejection is not
merely a procedural requirement. The condition is an important matter, essential to a
prompt settlement of claims against insurance companies, as it demands that insurance
suits be brought by the insured while the evidence as to the origin and cause of
destruction have not yet disappeared. It is in the nature of a condition precedent to the
liability of the insurer, or in other terms, a resolutory cause, the purpose of which is to
terminate all liabilities in case the action is not filed by the insured within the period
stipulated.

57

Insurance Case Digests
Atty. Bathan-Basuel

The bringing of the action against the Paramount Surety & Insurance Company, the
agent of the defendant Company cannot have any legal effect except that of notifying
the agent of the claim. Beyond such notification, the filing of the action can serve no
other purpose. There is no law giving any effect to such action upon the principal.
Besides, there is no condition in the policy that the action must be filed against the
agent, and this Court can not by interpretation, extend the clear scope of the
agreement beyond what is agreed upon by the parties.
Eagle Star Ins Co Ltd v Chia Yu 96 Phil 966
EAGLE STAR INSURANCE CO., LTD. VS. CHIA YU
G.R. No. L-5915
March 31, 1955
REYES, A., J.:
FACTS:
On January 15, 1946, Atkins, Kroll & Co., loaded on the S. S. Roeph Silverlight
owned and operated by Leigh Hoegh & Co., A/S, of San Francisco, California, 14 bales
of assorted underwear valued at P8,085.23 consigned to Chia Yu in the City of Manila.
The shipment was insured against all risks by Eagle Star Ins. Co. of San Francisco,
California, under a policy issued to the shipper and by the latter assigned to the
consignee. The vessel arrived in Manila on February 10, 1946, and on March 4 started
discharging its cargo into the custody of the Manila Terminal Co., Inc., which was then
operating the arrastre service for the Bureau of Customs. But of the 14 bales consigned
to Chia Yu only 10 were delivered to him as the remaining 4 could not be found. Three
of those delivered were also found damaged to the extent of 50 per cent.
Chia Yu claimed indemnity for the missing and damaged bales. But the claim
was declined, first, by the carrier and afterward by the insurer, whereupon Chia Yu
brought the present action against both, including their respective agents in the
Philippines. Commenced in the Court of First Instance of Manila on November 16, 1948,
or more than two years after delivery of the damaged bales and the date when the
missing bales should have been delivered, the action was resisted by the defendants
principally on the ground of prescription. But the trial court found for plaintiff and
rendered judgment in his favor for the sum claimed plus legal interest and costs. The
judgment was affirmed by the Court of Appeals.
Under our law the time limit for bringing a civil action upon a written contract
is ten years after the right of action accrues. But counsel for the insurer claim that this
statutory limitation must yield to the following stipulation in the policy:
"No suit or action on this Policy, for the recovery of any claim,
shall be sustainable in any Court of law or equity unless the
insured shall have fully complied with all the terms and conditions
of this Policy nor unless commenced within twelve (12) months
next after the happening of the loss * * *"
ISSUE:
2-S, 2013-2014

Whether or not the action to claim the proceeds from the insurance policy
issued by Eagle Star Insurance Company has already prescribed.
HELD:
The case for the insurer stands on a different footing from that of the
shipping company, for its claim of prescription is founded upon the terms of the policy
and not upon the bill of lading.
In the case of E. Macias & Co. vs. China Fire Insurance & Co., Ltd., et al., 46
Phil. 345, relied upon by the insurer, this Court held that a clause in an insurance policy
providing that an action upon the policy by the insured must be brought within a certain
time is, if reasonable, valid and will prevail over statutory limitations of the action. That
decision, however, was rendered before the passage of Act 4101, which amended the
Insurance Act by inserting the following section in chapter one thereof:
"SEC. 61-A. Any condition, stipulation or agreement in any
policy of insurance, limiting the time for commencing' an
action thereunder to a period of less than one year from
the time when the cause of action accrues, is void."
Examining the policy sued upon in the present case, we find that its
prescriptive clause, if given effect in accordance with the terms of the policy, would
reduce the period allowed the insured for bringing his action to less than one year. This
is so because the said clause makes the prescriptive period begin from the happening of
the loss and at the same time provides that no suit on the policy shall be sustainable in
any court unless the insured shall have first fully complied with all the terms and
conditions of the policy, among them that which requires that, as soon as the loss is
determined, written claim therefor be filed with the carrier and that the letter to the
carrier and the latter's reply should be attached to the claim papers to be sent to the
insurer. It is obvious that compliance with this condition precedent will necessarily
consume time and thus shorten the period for bringing suit to less than one year if the
period is to begin, as stated in the policy, from "the happening of the loss" Being
contrary to the law of the forum, such stipulation cannot be given effect.
In concluding, we may state that contractual limitations contained in insurance
policies are regarded with extreme jealousy by courts and will be strictly construed
against the insurer and should not be permitted to prevent a recovery when their just
and honest application would not produce that result.
Sun Life Office Ltd v CA 195 SCRA 193
SUN INSURANCE OFFICE LTD. V CA (TAN)
195 SCRA 193
PARAS; March 13, 1991
NATURE
Petition for certiorari to review the decision of the CA
58

Insurance Case Digests
Atty. Bathan-Basuel

FACTS
- Private respondent Emilio Tan took from the petitioner a Peso 300,000 property
insurance policy to cover his interest in the electrical insurance store of his brother
housed in a building in Iloilo City on August 15, 1983. Four days after the issuance of
the policy, the building including the insured store burned.
- On August 20, 1983, Tan filed his claim for fire loss. Sun Insurance, on February 29,
1984, wrote the private respondent denying the claim. On April 3, 1984, private
respondent wrote another letter to the insurance company requesting reconsideration
of the denial. Tan’s lawyer wrote another letter to the insurance company inquiring
about the April 3 letter which sought for a reconsideration of the denial. In its reply to
the lawyer’s letter, Sun Insurance reiterated its denial of the claim and enclosed therein
copies of the two previous denials dated February 29, 1984 and May 17, 1985.
- On November 20, 1985, Tan filed a civil case with the RTC. Petition filed a motion to
dismiss on the alleged ground that the action has already prescribed based on Condition
27 of the Insurance Policy which stated that the window to file the appropriate action
with either the Insurance Commission or in any court of competent jurisdiction is twelve
months from the rejection of the claim. RTC denied the motion and the subsequent
motion for reconsideration. The CA likewise denied the petition of Sun Insurance.
ISSUE
1. WON the court the filing of a motion for reconsideration interrupts the 12 months
prescription period to contest the denial of the insurance claim
2. WON the rejection of the claim shall be deemed final only if it contains words to the
effect that the denial is final

HELD
1. NO
- The SC held that Condition 27 of the Insurance policy is very clear and free from any
doubt or ambiguity. It has to be taken in its plain, ordinary, and popular sense. The
rejection letter of February 29, 1984 was clear and plain. The Court noted that the one
year period is likewise in accord with Section 23 of the Insurance Code which states
that any condition which limits the time for commencing an action to a period of less
than one year when the cause of action accrues is void. The right of action, according
to the SC, accrues at the time that the claim is rejected at the first instance. A request
for reconsideration of the denial cannot suspend the running of the prescriptive period.
The Court noted that the rationale for the one year period is to ensure that the
evidence as to the origin and cause of the destruction have not yet disappeared.
2. NO
- The Court clarified its ruling in Eagle Star Insurance Co. vs Chia Yu where it ruled that
“the cause of action in an insurance contract does not accrue until the Insured’s claim is
finally rejected by the Insurer” by stating the use of the word “finally” cannot be
construed to mean the rejection of a petition for reconsideration. What the court
referred to in effect is the rejection in the first instance as claimed by Sun Insurance
Disposition The decision of the CA is reversed and set aside. The case is dismissed
2-S, 2013-2014

Filipino Merchants Insurance Co Inc V CA 179 SCRA 638

G.R. No. 85141 November 28, 1989
FILIPINO
MERCHANTS
INSURANCE
CO.,
vs.
COURT OF APPEALS and CHOA TIEK SENG, respondents.

INC., petitioner,

On the issue of misrepresentation of lack of insurable interest.
FACTS:

§

§

§
§
§
§
§

Choa Tiek Seng, consignee of the shipment of fishmeal loaded, insured in "all risks
policy" 600 metric tons of fishmeal in new gunny bags of 90 kilos each from
Bangkok, Thailand to Manila against all risks under warehouse to warehouse terms
but only 59.940 metric tons was imported
When it was unloaded unto the arrastre contractor E. Razon, Inc. and Filipino
Merchants' surveyor ascertained and certified that in such discharge 105 bags
were in bad order condition which was reflected in the survey report of Bad Order
cargoes
Before delivery to Choa, E. Razon's Bad Order Certificate showed that a total of
227 bags in bad order condition
Choa brought an action against Filipino Merchants Insurance Co. who brought a
third party complaint against Compagnie Maritime Des Chargeurs Reunis and/or
E. Razon, Inc.
RTC: Ordered Filipino Merchants to pay Choa and reimburse from Compagnie
Maritime Des Chargeurs Reunis and third party defendant E. Razon, Inc.
CA: Affirmed but modified by adjudicating the third party complaint
Filipino Merchants contended that Chao has no insurable interest and therefore
the policy should be void and that it was fraud that it did not disclose of
such fact

ISSUE: W/N Choa Tiek Seng as consignee of the shipment has insurable interest

HELD: YES. CA affirmed.
§
§

§

Filipino contends that Chao does not have insurable interest, being only a
consignee of the goods.
Anent the issue of insurable interest, SC upheld the ruling of the CA that Chao, as
consignee of the goods in transit under an invoice containing the terms under "C &
F Manila," has insurable interest in said goods.
Section 13 of the Insurance Code defines insurable interest in property as every
interest in property, whether real or personal, or any relation thereto, or liability in
59

Insurance Case Digests
Atty. Bathan-Basuel

respect thereof, of such nature that a contemplated peril might directly damnify
the insured. In principle, anyone has an insurable interest in property who derives a
benefit from its existence or would suffer loss from its destruction whether he has
or has not any title in, or lien upon or possession of the property.
Insurable
interest in property may consist in (a) an existing interest; (b) an inchoate interest
founded on an existing interest; or (c) an expectancy, coupled with an existing
interest in that out of which the expectancy arises.
§

Chao, as vendee/consignee of the goods in transit has such existing interest
therein as may be the subject of a valid contract of insurance. His interest over
the goods is based on the perfected contract of sale.
The perfected
contract of sale between him and the shipper of the goods operates to vest in him
an equitable title even before delivery or before he performed the
conditions of the sale.
The contract of shipment, whether under F.O.B.,
C.I.F., or C. & F. as in this case, is immaterial in the determination of whether the
vendee has an insurable interest or not in the goods in transit. The perfected
contract of sale even without delivery vests in the vendee an equitable title, an
existing interest over the goods sufficient to be the subject of insurance

§

Article 1523 of the Civil Code provides that where, in pursuance of a contract of
sale, the seller is authorized or required to send the goods to the buyer,
delivery of the goods to a carrier, whether named by the buyer or not, for,
the purpose of transmission to the buyer is deemed to be a delivery of the goods
to the buyer, the exceptions to said rule not obtaining in the present case. The
Court has heretofore ruled that the delivery of the goods on board the carrying
vessels partake of the nature of actual delivery since, from that time, the foreign
buyers assumed the risks of loss of the goods and paid the insurance premium
covering them

§

C & F contracts are shipment contracts. The term means that the price fixed
includes in a lump sum the cost of the goods and freight to the named destination.
It simply means that the seller must pay the costs and freight necessary to bring
the goods to the named destination but the risk of loss or damage to the goods is
transferred from the seller to the buyer when the goods pass the ship's rail in the
port of shipment.

§

Moreover, the issue of lack of insurable interest was not among the defenses
averred in petitioners answer.
Mayer Steel Pipe Corporation V CA 274 SCRA 432

G.R. No. 124050; June 19, 1997
MAYER STEEL PIPE CORPORATION and HONGKONG GOVERNMENT SUPPLIES
DEPARTMENT, petitioners vs. CA, SOUTH SEA SURETY AND INSURANCE CO.,
INC. and the CHARTER INSURANCE CORPORATION, respondents
PUNO, J.:
2-S, 2013-2014

FACTS:
In 1983, Hongkong contracted Mayer to manufacture and supply various steel pipes and
fittings. From August to October, 1983, Mayer shipped the same to Hongkong as
evidenced by Invoices. Prior to the shipping, Mayer insured the pipes and fittings against
all risks with South Sea and Charter.
Mayer and Hongkong jointly appointed Industrial Inspection (International) Inc. as thirdparty inspector, which certified all the pipes and fittings to be in good order condition
before they were loaded in the vessel. Nonetheless, when the goods reached
Hongkong, it was discovered that a substantial portion was damaged.
A claim for indemnity was filed under the insurance contract. Charter paid Hongkong
HK$64,904.75. The latter and Mayer demanded payment of the balance of
HK$299,345.30 but was refused because the insurance surveyor's report allegedly
showed that the damage is a factory defect.
In 1986, an action to recover the sum of HK$299,345.30 was filed. The trial court
ruled in favor of Mayer and Hongkong. CA affirmed that the damage is not due to
factory defect and that it was covered by the "all risks" insurance policies. However,
it set aside the decision of the trial court and dismissed the complaint on the ground of
prescription. It held that the action is barred under Section 3(6) of the Carriage of
Goods by Sea Act since it was filed only on April 17, 1986, more than two years from
the time the goods were unloaded from the vessel. Section 3(6) of the Carriage of
Goods by Sea Act provides that "the carrier and the ship shall be discharged from all
liability in respect of loss or damage unless suit is brought within one year after delivery
of the goods or the date when the goods should have been delivered." Respondent
court ruled that this provision applies not only to the carrier but also to the insurer,
citing Filipino Merchants Insurance Co., Inc. vs. Alejandro. Hence, this petition.
ISSUE:
Whether the CA erred in holding that the cause of action had already prescribed on the
mistaken application of the Carriage of Goods by Sea Act and thus dismissing the
complaint.
HELD:
YES. CA erred in applying Section 3(6) of the Carriage of Goods by Sea Act. Under this
provision, only the carrier's liability is extinguished if no suit is brought within one
year. But the liability of the insurer is not extinguished because the insurer's liability is
based not on the contract of carriage but on the contract of insurance. The Carriage
of Goods by Sea Act governs the relationship between the carrier on the one hand and
the shipper, the consignee and/or the insurer on the other hand. It defines the
obligations of the carrier under the contract of carriage. It does not, however, affect
the relationship between the shipper and the insurer. The latter case is governed by
the Insurance Code.
The Filipino Merchants case is different from the case at bar. In Filipino Merchants, it
was the insurer which filed a claim against the carrier for reimbursement of the amount
60

Insurance Case Digests
Atty. Bathan-Basuel

it paid to the shipper. In the case at bar, it was the shipper which filed a claim against
the insurer. The ruling in Filipino Merchants should apply only to suits against the
carrier filed either by the shipper, the consignee or the insurer. The obligation under
an all-risk policy prescribes in ten years, in accordance with Article 1144 of the New
Civil Code. Petition GRANTED.

Malayan Ins Co v. Cruz Arnaldo 154 SCRA 672
MALAYAN INSURANCE CO., INC. (MICO), petitioner, vs. GREGORIA CRUZ
ARNALDO, in her capacity as the INSURANCE COMMISSIONER, and
CORONACION PINCA, respondents.
FACTS:










On June 7, 1981, the petitioner (hereinafter called (MICO) issued to the
private respondent, Coronacion Pinca, Fire Insurance Policy No. F-001-17212
on her property for the amount of P100,000.00, effective July 22, 1981, until
July 22, 1982.
On October 15, 1981, MICO allegedly cancelled the policy for non-payment, of
the premium and sent the corresponding notice to Pinca.
On December 24, 1981, payment of the premium for Pinca was received by
Domingo Adora, agent of MICO.
On January 15, 1982, Adora remitted this payment to MICO, together with
other payments.
On January 18, 1982, Pinca's property was completely burned
On February 5, 1982, Pinca's payment was returned by MICO to Adora on the
ground that her policy had been cancelled earlier. But Adora refused to accept
it.
In due time, Pinca made the requisite demands for payment, which MICO
rejected. She then went to the Insurance Commission. It is because she was
ultimately sustained by the public respondent that the petitioner has come to
us for relief.

ISSUE: Whether there is an existing insurance policy at the time of the loss sustained
by Pinca
HELD:
Yes. The Court does not agree in MICO's view that there was no existing insurance at
the time of the loss sustained by Pinca because her policy never became effective for
non-payment of premium. Payment was in fact made, rendering the policy operative as
of June 22, 1981, and removing it from the provisions of Article 77. Thereafter, the
policy could be cancelled on any of the supervening grounds enumerated in Article 64
(except "non-payment of premium") provided the cancellation was made in accordance
therewith and with Article 65.
2-S, 2013-2014

Section 64 reads as follows:
"SEC. 64.No policy of insurance other than life shall be cancelled by the insurer except
upon prior notice thereof to the insured, and no notice of cancellation shall be effective
unless it is based on the occurrence, after the effective date of the policy, of one or
more of the following:
"(a)non-payment of premium;
"(b)conviction of a crime arising out of acts increasing the hazard insured against;
"(c)discovery of fraud or material misrepresentation;
"(d)discovery of willful or reckless acts or commissions increasing the hazard insured
against;
"(e)physical changes in the property insured which result in the property becoming
uninsurable; or
"(f)a determination by the Commissioner that the continuation of the policy would
violate or would place the insurer in violation of this Code."
As for the method of cancellation, Section 65 provides as follows:
"SEC. 65.All notices of cancellation mentioned in the preceding
section shall be in writing, mailed or delivered to the named insured
at the address shown in the policy, and shall state (a) which of the
grounds set forth in section sixty-four is relied upon and (b) that,
upon written request of the named insured, the insurer will furnish
the facts on which the cancellation is based."
A valid cancellation must, therefore, require concurrence of the following conditions:
(1)There must be prior notice of cancellation to the insured;
(2)The notice must be based on the occurrence, after the effective date of the policy,
of one or more of the grounds mentioned.
(3)The notice must be (a) in writing, (b) mailed, or delivered to the named insured, (c)
at the address shown in the policy;
(4)It must state (a) which of the grounds mentioned in Section 64 is relied upon and
(b) that upon written request of the insured, the insurer will furnish the facts on which
the cancellation is based.
MICO claims it cancelled the policy in question on October 15, 1981, for nonpayment of premium. On the other hand, there is the flat denial of Pinca, who says she
never received the claimed cancellation and who, of course, did not have to prove such
denial Considering the strict language of Section 64 that no insurance policy shall be
cancelled except upon prior notice, it behooved MICO to make sure that the cancellation
was actually sent to and received by the insured.

61

Insurance Case Digests
Atty. Bathan-Basuel

It stands to reason that if Pinca had really received the said notice, she would
not have made payment on the original policy on December 24, 1981. Instead, she
would have asked for a new insurance, effective on that date and until one year later,
and so taken advantage of the extended period. The Court finds that if she did pay on
that date, it was because she honestly believed that the policy issued on June 7, 1981,
was still in effect and she was willing to make her payment retroact to July 22, 1981,
its stipulated commencement date. As it has not been shown that there was a valid
cancellation of the policy, there was consequently no need to renew it but to pay the
premium thereon. Payment was thus legally made on the original transaction and it
could be, and was, validly received on behalf of the insurer by its agent Adora. Adora,
incidentally, had not been informed of the cancellation either and saw no reason not to
accept the said payment.

HELD:

There was NO violation of the Fire Extinguishing Appliances Warranty in the
policy.
The warranty provides:
“WARRANTED that during the currency of this Policy, Fire Extinguishing
Appliances as mentioned below shall be maintained in efficient working order
on the premises to which insurance applies:
PORTABLE EXTINGUISHERS
INTERNAL HYDRANTS
EXTERNAL HYDRANTS
FIRE PUMP
24-HOUR SECURITY SERVICES

WHEREFORE, the petition is DENIED.
7.

BREACH of this warranty shall render this policy null and void and the Company
shall no longer be liable for any loss which may occur.”

Warranties
American Home Assurance Company v. Tantuco Enterprises, Inc. 366 SCRA 740

American Home Assurance Company v. Tantuco Enterprises, Inc.
G.R. No. 138941. October 8, 2001
FACTS:

Tantuco Enterprises (later to be referred to as Tantuco) is engaged in the
coconut oil milling and refining industry. It owns two oil mills which are located at its
factory compound at Iyam, Lucena City. The two oil mills were separately covered by
fire insurance policies issued by American Home Assurance Co., Philippine Branch. The
first oil mill was insured for three million pesos (P3,000,000.00), while the new oil mill
was insured for six million pesos (P6,000,000.00).
A fire that broke out in the early morning of September 30,1991 and
consumed the new oil mill. Respondent immediately notified the petitioner of the
incident. American Home rejected respondent’s claim for the insurance proceeds on the
ground that no policy was issued by it covering the burned oil mill. It stated that the
description of the insured establishment referred to another building. American Home
also raised the issue that Tantuco violated the express terms of the Fire Extinguishing
Appliances Warranty stated in the policy.
A complaint for specific performance and damages was consequently
instituted by the Tantuco with the RTC Lucena City.
RTC Lucena – in favor of Tantuco
CA – affirmed RTC's decision
ISSUE: Whether there was a violation of the Fire Extinguishing Appliances Warranty
stated in the policy resulting to the avoidance of the policy.
2-S, 2013-2014

Petitioner argues that the warranty clearly obligates the insured to maintain all the
appliances specified therein. The breach occurred when the respondent failed to install
internal fire hydrants inside the burned building as warranted. This fact was admitted
by the oil mill’s expeller operator, Gerardo Zarsuela.

The aforementioned warranty did not require respondent to provide for all the fire
extinguishing appliances enumerated therein. Additionally, we find that neither did it
require that the appliances are restricted to those mentioned in the warranty. In other
words, what the warranty mandates is that respondent should maintain in efficient
working condition within the premises of the insured property, fire fighting equipments
such as, but not limited to, those identified in the list, which will serve as the oil mill’s
first line of defense in case any part of it bursts into flame.
To be sure, respondent was able to comply with the warranty. Within the vicinity of the
new oil mill can be found the following devices: numerous portable fire extinguishers,
two fire hoses, fire hydrant, and an emergency fire engine. All of these equipments
were in efficient working order when the fire occurred.
It ought to be remembered that not only are warranties strictly construed against the
insurer, but they should, likewise, by themselves be reasonably interpreted. That
reasonableness is to be ascertained in light of the factual conditions prevailing in each
case. Here, we find that there is no more need for an internal hydrant considering that
inside the burned building were: (1) numerous portable fire extinguishers, (2) an
emergency fire engine, and (3) a fire hose which has a connection to one of the
external hydrants.
Ang Giok Chip v. Sprinfiled Fire & Maritime Ins. Co., 56 Phil 375
FACTS:
62

Insurance Case Digests
Atty. Bathan-Basuel

Ang Giok Chip doing business under the name and style of Hua Bee Kong Si was
formerly the owner of a warehouse situated at No. 643 Calle Reina Regente, City of
Manila. The contents of the warehouse were insured with the three insurance companies
for the total sum of P60,000. One insurance policy, in the amount of P10,000, was
taken out with the Springfield Fire & Marine Insurance Company. The warehouse was
destroyed by fire on January 11, 1928, while the policy issued by the latter company
was in force.
Predicated on this policy the plaintiff instituted action in the Court of First Instance of
Manila against the defendant to recover a proportional part of the loss coming to
P8,170.59. Four special defenses were interposed on behalf of the insurance company,
one being planted on a violation of warranty F fixing the amount of hazardous goods
which might be stored in the insured building.

Ten thousand pesos Philippine Currency. — On general non-hazardous merchandise,
chiefly consisting of chucherias, also produce, Cacao, Flour, all the property of the
Insured, or held by them in trust, on commission or on joint account with others, or for
which he is responsible, while contained during the currency of this policy in the
godown, situate No. 643 Calle Reina Regent. . . .
This policy is subject to the hereon attached "Ordinary Short Period Rate Scale"

We are given to understand, and there is no indication to the contrary, that we have
here a standard insurance policy. We are further given to understand, and there is no
indication to the contrary, that the issuance of the policy in this case with its attached
rider conforms to well established practice in the Philippines and elsewhere. We are
further given to understand, and there is no indication to the contrary, that there are
no less than sixty- nine insurance companies doing business in the Philippine Islands with
outstanding policies more or less similar to the one involved in this case, and that to
nullify such policies would place an unnecessary hindrance in the transaction of
insurance business in the Philippines. These are matters of public policy. We cannot
believe that it was ever the legislative intention to insert in the Philippine Law on
Insurance an oddity, an incongruity, entirely out of harmony with the law as found in
other jurisdiction, and destructive of good business practice.
We have studied this case carefully and having done so have reached the definite
conclusion that warranty F, a rider attached to the face of the insurance policy, and
referred to in contract of insurance, is valid and sufficient under section 65 of the
Insurance Act. Accordingly, sustaining the first and fourth errors assigned, and it being
unnecessary to discuss the remaining errors, the result will be to reverse the judgment
appealed from and to order the dismissal of the complaint, without special
pronouncement as to costs in either instance.
Young v. Midland Textile Insurance Co., 30 Phil 617

Warranties A & F, Co-insurances Clause "and Three Fourths Loss Clause," which are
forming part of same. Co-insurance declared:

FACTS:

"P20,000. — Sun Insurance Office Ltd. (K & S)." (Emphasis inserted.) Securely pasted
on the left hand margin of the face of the policy are five warranties and special clauses.
One of them is warranty F, specially referred to on the face of the policy, reading in part
as follows:

The undisputed facts upon which said action is based are as follows:

ISSUE: whether a warranty referred to in the policy as forming part of the contract of
insurance and in the form of a rider to the insurance policy, is null and void because not
complying with the Philippine Insurance Act.
HELD:
"Every express warranty, made at or before the execution of a policy, must be
contained in the policy itself, or in another instrument signed by the insured and
referred to in the policy, as making a part of it." As the Philippine law was taken
verbatim from the law of California, in accordance with well settled canons of statutory
construction, the court should follow in fundamental points, at least, the construction
placed by California courts on a California law.
"Any express warranty or condition is always a part of the policy, but, like any other
part of an express contract, may be written in the margin, or contained in proposals or
documents expressly referred to in the policy, and so made a part of it."
2-S, 2013-2014

The purpose of the present action is to recover the sum of P3,000 upon an
insurance policy. The lower court rendered a judgment in favor of the plaintiff.

1. The plaintiff conducted a candy and fruit store on the Escolta and occupied a building
as a residence and bodega (storehouse).
2 The defendant entered into a contract of insurance with the plaintiff.
3. On the conditions of said contract of insurance is found in "warranty B" and is as
follows: "Waranty B. — It is hereby declared and agreed that during the pendency of
this policy no hazardous goods stored or kept for sale, and no hazardous trade or
process be carried on, in the building to which this insurance applies, or in any building
connected therewith."
4. The plaintiff placed in said residence and bodega three boxes which belonged to him
and which were filed with fireworks.
5. Said residence and bodega and the contents thereof were partially destroyed by fire.
6. Said fireworks had been given to the plaintiff by the former owner of the Luneta
Candy Store; that the plaintiff intended to use the same in the celebration of the
63

Insurance Case Digests
Atty. Bathan-Basuel

Chinese new year; that the authorities of the city of Manila had prohibited the use of
fireworks on said occasion, and that the plaintiff then placed the same in said bodega,
where they remained from the 4th or 5th of February, 1913, until after the fire of the
18th of March, 1913.
7. Both of the parties agree that said fireworks come within the phrase "hazardous
goods," mentioned in said "warranty B" of the policy.
8. That said fireworks were found in a part of the building not destroyed by the fire;
that they in no way contributed to the fire, or to the loss occasioned thereby.
ISSUE:
Whether or not the placing of said fireworks in the building insured, under the
conditions above enumerated, they being "hazardous goods," is a violation of the terms
of the contract of insurance and especially of "warranty B."
HELD:

The plaintiff contends that under all the facts and circumstances of the case,
they were not "stored" in said building, and that the placing of them in the building was
not a violation of the terms of the contract.
This leads us to a consideration of the meaning of the accord "stored" as used in said
"warranty B. The author of the Century Dictionary defines the world "store" to be a
deposit in a store or warehouse for preservation or safe keeping; o place in a warehouse
or other place of deposit for safe keeping. Said definitions, of course, do not include a
deposit in a store, in small quantities, for daily use. "Daily use" precludes the idea of a
deposit for preservation or safe keeping, as well as a deposit for future consumption, or
safe keeping. In the present case no claim is made that the "hazardous goods" were
placed in the bodega for present or daily use. It is admitted that they were placed in the
bodega "for future use," or for future consumption, or for safe keeping. It seems clear
to us that the "hazardous goods" in question were "stored" in the bodega, as that word
is generally defined.
If the "warranty" is a term of the contract, will not its violation cause a breach and
justify noncompliance or a repudiation?
Contracts of insurance are contracts of indemnity upon the terms and conditions
specified in the policy. The parties have a right to impose such reasonable conditions at
the time of the making of the contract as they may deem wise and necessary. he terms
of the policy constitute the measure of the insurer's liability, and in order to recover the
insured must show himself within those terms; and if it appears that the contract has
been terminated by a violation, on the part of the insured, of its conditions, then there
can be no right of recovery. The compliance of the insured with the terms of the
contract is a condition precedent to the right of recovery. If the insured has violated or
failed to perform the conditions of the contract, and such a violation or want of
performance has not been waived by the insurer, then the insured cannot recover. The
conditions of contracts of insurance, when plainly expressed in a policy, are binding
2-S, 2013-2014

upon the parties and should be enforced by the courts, if the evidence brings the case
clearly within their meaning and intent.

The appellant argues, however, that in view of the fact that the "storing" of the
fireworks on the premises of the insured did not contribute in any way to the damage
occasioned by the fire, he should be permitted to recover — that the "storing" of the
"hazardous goods" in no way caused injury to the defendant company.
The violation of the terms of the contract, by virtue of the provisions of the policy
itself, terminated, at the election of either party, he contractual relations. Certainly it
cannot be denied that the placing of the firecrackers in the building insured increased
the risk. The plaintiff had not paid a premium based upon the increased risk, neither had
the defendant issued a policy upon the theory of a different risk. The plaintiff was
enjoying, if his contention may be allowed may be allowed, the benefits of an insurance
policy upon one risk, whereas, as a matter of fact, it was issued upon an entirely
different risk. The defendant had neither been paid nor had issues a policy to cover the
increased risk. An increase of risk which is substantial and which is continued for a
considerable period of time, is a direct and certain injury to the insurer, and changes the
basis upon which the contract of insurance rests.
Therefore and for the foregoing reasons, the judgment of the lower court is hereby
revoked and the defendant is hereby relieved from any responsibility under said
complaint, and, without any finding as to costs, it is so ordered.
Qua Chee Gan v. law Unoin & Rock Ins. Co. Ltd, G.R. No. L-4611, December 17,
1955
FACTS: Qua Chee Gan, a merchant of Albay, instituted this action in 1940, in the Court
of First Instance of said province, seeking to recover the proceeds of certain fire
insurance policies totalling P370,000, issued by the Law Union & Rock Insurance Co.,
Ltd., upon certain bodegas and merchandise of the insured that were burned on June
21, 1940. The records of the original case were destroyed during the liberation of the
region, and were reconstituted in 1946. After a trial that lasted several years, the Court
of First Instance rendered a decision in favor of the plaintiff. The record shows that
before the last war, plaintiff-appellee owned four warehouses or bodegas (designated as
Bodegas Nos. 1 to 4) in the municipality of Tabaco, Albay, used for the storage of
stocks of copra and of hemp, baled and loose, in which the appellee dealth extensively.
They had been, with their contents, insured with the defendant Company since 1937,
and the lose made payable to the Philippine National Bank as mortgage of the hemp and
crops, to the extent of its interest. TOTAL 370,000. BODEGA 1, 2, 3, 4 etc.
Fire of undetermined origin that broke out in the early morning of July 21, 1940, and
lasted almost one week, gutted and completely destroyed Bodegas Nos. 1, 2 and 4,
with the merchandise stored theren. Plaintiff-appellee informed the insurer by telegram
on the same date;; and on the next day, the fire adjusters engaged by appellant
insurance company arrived and proceeded to examine and photograph the premises,
pored over the books of the insured and conducted an extensive investigation. The
64

Insurance Case Digests
Atty. Bathan-Basuel

plaintiff having submitted the corresponding fire claims, totalling P398,562.81 (but
reduced to the full amount of the insurance, P370,000), the Insurance Company
resisted payment, claiming violation of warranties and conditions, filing of fraudulent
claims, and that the fire had been deliberately caused by the insured or by other
persons in connivance with him.
With counsel for the insurance company acting as private prosecutor, Que Chee Gan,
with his brother, Qua Chee Pao, and some employees of his, were indicted and tried in
1940 for the crime of arson, it being claimed that they had set fire to the destroyed
warehouses to collect the insurance. They were, however, acquitted by the trial court in
a final decision dated July 9, 1941 (Exhibit WW). Thereafter, the civil suit to collect the
insurance money proceeded to its trial and termination in the Court below, with the
result noted at the start of this opinion. The Philippine National Bank's complaint in
intervention was dismissed because the appellee had managed to pay his indebtedness
to the Bank during the pendecy of the suit, and despite the fire losses.
ISSUE: Whether the policy is avoided on the ground of breach of warranty, specifically
appearing on a rider pasted on the face of the policies
HELD: Appellant is barred by waiver (or rather estoppel) to claim violation of the socalled fire hydrants warranty, for the reason that knowing fully all that the number of
hydrants demanded therein never existed from the very beginning, the appellant
neverthless issued the policies in question subject to such warranty, and received the
corresponding premiums. It would be perilously close to conniving at fraud upon the
insured to allow appellant to claims now as void ab initio the policies that it had issued
to the plaintiff without warning of their fatal defect, of which it was informed, and after
it had misled the defendant into believing that the policies were effective.
The insurance company was aware, even before the policies were issued, that in the
premises insured there were only two fire hydrants installed by Qua Chee Gan and two
others nearby, owned by the municipality of Tabaco, only two fire hydrants installed by
Qua Chee Gan and two others nearby, owned by the municipality of TAbaco,

the contract inconsistent with the facts, and the insurer is stopped thereafter from
asserting the breach of such conditions. The law is charitable enough to assume, in the
absence of any showing to the contrary, that an insurance company intends to
executed a valid contract in return for the premium received;; and when the policy
contains a condition which renders it voidable at its inception, and this result is known
to the insurer, it will be presumed to have intended to waive the conditions and to
execute a binding contract, rather than to have deceived the insured into thinking he is
insured when in fact he is not, and to have taken his money without consideration.

To allow a company to accept one's money for a policy of insurance which it then
knows to be void and of no effect, though it knows as it must, that the assured
believes it to be valid and binding, is so contrary to the dictates of honesty and fair
dealing, and so closely related to positive fraud, as to the abhorent to fairminded men.
We see no reason why the prohibition of keeping gasoline in the premises could not be
expressed clearly and unmistakably, in the language and terms that the general public
can readily understand, without resort to obscure esoteric expression
his rigid application of the rule on ambiguities has become necessary in view of current
business practices. The courts cannot ignore that nowadays monopolies, cartels and
concentrations of capital, endowed with overwhelming economic power, manage to
impose upon parties dealing with them cunningly prepared "agreements" that the
weaker party may not change one whit, his participation in the "agreement" being
reduced to the alternative to take it or leave it" labelled since Raymond Baloilles"
contracts by adherence" (con tracts d'adhesion), in contrast to these entered into by
parties bargaining on an equal footing, such contracts (of which policies of insurance
and international bills of lading are prime examples) obviously call for greater strictness
and vigilance on the part of courts of justice with a view to protecting the weaker party
from abuses and imposition, and prevent their becoming traps for the unwarry
RTC DECISION

contrary to the requirements of the warranty in question. Such fact appears from
positive testimony for the insured that appellant's agents inspected the premises;; and
the simple denials of appellant's representative (Jamiczon) can not overcome that
proof. That such inspection was made is moreover rendered probable by its being a
prerequisite for the fixing of the discount on the premium to which the insured was
entitled, since the discount depended on the number of hydrants, and the fire fighting
equipment available (See "Scale of Allowances" to which the policies were expressly
made subject). The law, supported by a long line of cases, is expressed by American
Jurisprudence (Vol. 29, pp. 611-612) to be as follows:
It is usually held that where the insurer, at the time of the issuance of a policy of
insurance, has knowledge of existing facts which, if insisted on, would invalidate the
contract from its very inception, such knowledge constitutes a waiver of conditions in
2-S, 2013-2014

Bachrach v. British American Assurance Co., 17 Phil 555
FACTS: Fire insurance policy issued by the defendant to the plaintiff. Properties
destroyed by fire.
IN DEFENDANT’S ANSWER:
First. That the plaintiff maintained a paint and varnish shop in the said building where
the goods which were insured were stored.
Second. That the plaintiff transferred his interest in and to the property covered by the
policy to H. W. Peabody & Co. to secure certain indebtedness due and owing to said
65

Insurance Case Digests
Atty. Bathan-Basuel

company, and also that the plaintiff had transferred his interest in certain of the goods
covered by the said policy to one Macke, to secure certain obligations assumed by the
said Macke for and on behalf of the insured. That the sanction of the said defendant
had not been obtained by the plaintiff, as required by the said policy.
Third. That the plaintiff, on the 18th of April, 1908, and immediately preceding the
outbreak of the alleged fire, willfully placed a gasoline can containing 10 gallons of
gasoline in the upper story of said building in close proximity to a portion of said goods,
wares, and merchandise, which can was so placed by the plaintiff as to permit the
gasoline to run on the floor of said second story, and after so placing said gasoline, he,
the plaintiff, placed in close proximity to said escaping gasoline a lighted lamp
containing alcohol, thereby greatly increasing the risk of fire.
Fourth. That the plaintiff made no proof of the loss within the time required by
condition five of said policy, nor did the insured file a statement with he municipal or
any other judge or court of the goods alleged to have been in said building at the time
of the alleged fire, nor of the goods saved, nor the loss suffered.
LOWER COURT’S DECISION:
It is claimed that either gasoline or alcohol was kept in violation of the policy in the
bodega containing the insured property. The testimony on this point is somewhat
conflicting, but conceding all of the defendant's claims, the construction given to this
claim by American courts would not justify the forfeiture of the policy on that ground.
The property insured consisted mainly of household furniture kept for the purpose of
sale. The preservation of the furniture in a salable condition by retouching or otherwise
was incidental to the business. The evidence offered by the plaintiff is to the effect that
alcohol was used in preparing varnish for the purpose of retouching, though he also
says that the alcohol was kept in store and not in the bodega where the furniture was.
It is well settled that the keeping of inflammable oils on the premises, though prohibited
by the policy, does not void it if such keeping is incidental to the business. Thus, where
a furniture factory keeps benzine for the purposes of operation (Davis vs. Pioneer
Furniture Company, 78 N. W. Rep., 596;; Faust vs. American Fire Insurance Company,
91 Wis., 158), or where it is used for the cleaning machinery (Mears vs. Humboldt
Insurance Company, 92 Pa. St., 15;; 37 Am. Rep., 647), the insurer can not on that
ground avoid payment of loss, though the keeping of the benzine on the premises is
expressly prohibited. These authorities also appear sufficient to answer the objection
that the insured automobile contained gasoline and that the plaintiff on one occasion
was seen in the bodega with a lighted lamp. The first was incidental to the use of the
insured article and the second being a single instance falls within the doctrine of the
case last cited.
ISSUE: WHETHER BACHRACH CAN CLAIM PROCEEDS
HELD: It may be added that there was no provision in the policy prohibiting the keeping
of paints and varnishes upon the premises where the insured property was stored. If the
2-S, 2013-2014

company intended to rely upon a condition of that character, it ought to have been
plainly expressed in the policy.
As to the claim that the loss occurred through the voluntary act of the insured, we
consider it unnecessary to review the evidence in detail. That was done by another
branch of this court in disposing of the criminal prosecution brought against the
insured, on the same ground, based mainly on the same evidence. And regardless of
whether or not the judgment in that proceeding is res adjudicata as to anything here,
we are at least of the opinion that the evidence to establish this defense should not be
materially less convincing than that required in order to convict the insured of the crime
of arson. (Turtell vs. Beamount, 25 Rev. Rep., 644.) In order to find that the defense of
incendiarism was established here, we would be obliged, therefore, in effect to set aside
the findings of the judge and assessors in the criminal cause, and this we would be loath
to do even though the evidence now produced were much stronger than it is.
It is claimed that the execution of a chattel mortgage on the insured property violated
what is known as the "alienation clause," which is now found in most policies, and which
is expressed in the policies involved in cases 6496 and 6497 by a purchase imposing
forfeiture if the interest in the property pass from the insured. (Cases 6496 and 6497,
in which are involved other action against other insurance companies for the same loss
as in the present action.)
This clause has been the subject of a vast number of judicial decisions (13 Am. & Eng.
Encyc. of Law, 2d ed., pp. 239 et seq.), and it is held by the great weight of authority
that the interest in property insured does not pass by the mere execution of a chattel
mortgage and that while a chattel mortgage is a conditional sale, there is no alienation
within the meaning of the insurance law until the mortgage acquires a right to take
possession by default under the terms of the mortgage. No such right is claimed to
have accrued in the case at bar, and the alienation clause is therefore inapplicable.
Geagonia V. CA 214 SCRA 152 (1995)
FACTS:
The petitioner is the owner of Norman's Mart located in the public market of San
Francisco, Agusan del Sur. On 22 December 1989, he obtained from the private
respondent fire insurance policy for P100,000. The period of the policy was from 22
December 1989 to 22 December 1990 and covered the following: "Stock-in-trade
consisting principally of dry goods such as RTW's for men and women wear and other
usual to assured's business. The petitioner declared in the policy under the subheading
entitled CO-INSURANCE that Mercantile Insurance Co., Inc. was the co-insurer for
P50,000.00. From 1989 to 1990, the petitioner had in his inventory stocks amounting
to P392,130.50.
The policy contained the following condition:

66

Insurance Case Digests
Atty. Bathan-Basuel

3. The insured shall give notice to the Company of any insurance or insurances already
affected, or which may subsequently be effected, covering any of the property or
properties consisting of stocks in trade, goods in process and/or inventories only
hereby insured, and unless such notice be given and the particulars of such insurance or
insurances be stated therein or endorsed in this policy pursuant to Section 50 of the
Insurance Code, by or on behalf of the Company before the occurrence of any loss or
damage, all benefits under this policy shall be deemed forfeited, provided however, that
this condition shall not apply when the total insurance or insurances in force at the time
of the loss or damage is not more than P200,000.00.
On 27 May 1990, fire of accidental origin broke out at around 7:30 p.m. at the public
market of San Francisco, Agusan del Sur. The petitioner's insured stock-in-trade were
completely destroyed prompting him to file with the private respondent a claim under
the policy. On 28 December 1990, the private respondent denied the claim because it
found that at the time of the loss the petitioner's stocks-in-trade were likewise covered
by fire insurance policies No. GA-28146 and No. GA-28144, for P100,000.00 each,
issued by the Cebu Branch of the Philippines First Insurance Co., Inc. (hereinafter PFIC).
3 These policies indicate that the insured was "Messrs. Discount Mart (Mr. Armando
Geagonia, Prop.)" with a mortgage clause reading:
MORTGAGE: Loss, if any shall be payable to Messrs. Cebu Tesing Textiles, Cebu City as
their interest may appear subject to the terms of this policy. CO-INSURANCE DECLARED:
P100,000. — Phils. First CEB/F 24758. 4
The basis of the private respondent's denial was the petitioner's alleged violation of
Condition 3 of the policy.
PETITIONER’S CONTENTION: at the time he obtained the private respondent's fire
insurance policy he knew that the two policies issued by the PFIC were already in
existence; however, he had no knowledge of the provision in the private respondent's
policy requiring him to inform it of the prior policies; this requirement was not
mentioned to him by the private respondent's agent; and had it been mentioned, he
would not have withheld such information.
RESPONDENT’S CONTENTION: denied the allegations in the complaint and set up as its
principal defense the violation of Condition 3 of the policy.
INSURANCE COMMISSION: petitioner did not violate Condition 3 as he had no knowledge
of the existence of the two fire insurance policies obtained from the PFIC.
CA: reversed the decision of I.C.
ISSUES:
(a) whether the petitioner had prior knowledge of the two insurance policies issued by
the PFIC when he obtained the fire insurance policy from the private respondent,
thereby, for not disclosing such fact, violating Condition 3 of the policy, and [YES]
(b) if he had, whether he is precluded from recovering therefrom. [NO]
2-S, 2013-2014

HELD: We agree with the Court of Appeals that the petitioner knew of the prior policies
issued by the PFIC. His letter of 18 January 1991 to the private respondent
conclusively proves this knowledge. His testimony to the contrary before the Insurance
Commissioner and which the latter relied upon cannot prevail over a written admission
made ante litem motam. It was, indeed, incredible that he did not know about the prior
policies since these policies were not new or original. Policy No. GA-28144 was a
renewal of Policy No. F-24758, while Policy No. GA-28146 had been renewed twice, the
previous policy being F-24792.
Condition 3 of the private respondent's Policy No. F-14622 is a condition which is not
proscribed by law. Its incorporation in the policy is allowed by Section 75 of the
Insurance Code which provides:

"[a] policy may declare that a violation of specified provisions thereof shall avoid it,
otherwise the breach of an immaterial provision does not avoid the policy." Such a
condition is a provision which invariably appears in fire insurance policies and is intended
to prevent an increase in the moral hazard. It is commonly known as the additional or
"other insurance" clause and has been upheld as valid and as a warranty that no other
insurance exists. Its violation would thus avoid the 
policy. However, in order to
constitute a violation, the other insurance must be upon same subject matter, the same
interest therein, and the same risk.”
As to a mortgaged property, the mortgagor and the mortgagee have each an
independent insurable interest therein and both interests may be one policy, or each
may take out a separate policy covering his interest, either at the same or at separate
times. A mortgagor may, however, take out insurance for the benefit of the mortgagee,
which is the usual practice. The mortgagee may be made the beneficial payee in several
ways. He may become the assignee of the policy with the consent of the insurer; or the
mere pledgee without such consent; or the original policy may contain a mortgage
clause; or a rider making the policy payable to the mortgagee "as his interest may
appear" may be attached; or a "standard mortgage clause," containing a collateral
independent contract between the mortgagee and insurer, may be attached; or the
policy, though by its terms payable absolutely to the mortgagor, may have been
procured by a mortgagor under a contract duty to insure for the mortgagee's benefit,
in which case the mortgagee acquires an equitable lien upon the proceeds.
It is a cardinal rule on insurance that a policy or insurance contract is to be interpreted
liberally in favor of the insured and strictly against the company, the reason being,
undoubtedly, to afford the greatest protection which the insured was endeavoring to
secure when he applied for insurance. It is also a cardinal principle of law that forfeitures
are not favored and that any construction which would result in the forfeiture of the
policy benefits for the person claiming thereunder, will be avoided, if it is possible to
construe the policy in a manner which would permit recovery, as, for example, by
finding a waiver for such forfeiture.

67

Insurance Case Digests
Atty. Bathan-Basuel

With these principles in mind, we are of the opinion that Condition 3 of the subject
policy is not totally free from ambiguity and must, perforce, be meticulously analyzed.
Such analysis leads us to conclude that (a) the prohibition applies only to double
insurance, and (b) the nullity of the policy shall only be to the extent exceeding
P200,000.00 of the total policies obtained.
The first conclusion is supported by the portion of the condition referring to other
insurance "covering any of the property or properties consisting of stocks in trade,
goods in process and/or inventories only hereby insured," and the portion regarding the
insured's declaration on the subheading CO-INSURANCE that the co-insurer is Mercantile
Insurance Co., Inc. in the sum of P50,000.00.
A double insurance exists where the same person is insured by several insurers
separately in respect of the same subject and interest. As earlier stated, the insurable
interests of a mortgagor and a mortgagee on the mortgaged property are distinct and
separate. Since the two policies of the PFIC do not cover the same interest as that
covered by the policy of the private respondent, no double insurance exists. The nondisclosure then of the former policies was not fatal to the petitioner's right to recover
on the private respondent's policy.
When a property owner obtains insurance policies from two or more insurers in a total
amount that exceeds the property's value, the insured may have an inducement to
destroy the property for the purpose of collecting the insurance. The public as well as
the insurer is interested in preventing a situation in which a fire would be profitable to
the insured.
PETITION GRANTED. CA DECISION SET ASIDE.
8.

Premium
Arce v. Capital Insurance & Surety Co., Inc 117 SCRA 63

FACTS:The appellee owned a residential house in Tondo Manila which was insured with
the appellant COMPANY since 1961. In November 1965, the COMPANY sent to the
INSURED a Renewal Certificate to cover the period from December 5, 1965 to
December 5,1966, and requested payment of the corresponding premium. Anticipating
that the premium could not be paid on time, the INSURED, thru his wife asked for an
extension which was on January 4, 1996. The request was granted by the COMPANY.
After the lapse of the requested extension, INSURED still failed to pay the premium.
Thereafter, the house of the INSURED was totally destroyed by fire. Upon INSURED's
presentation of claim for indemnity, he was told that no indemnity was due because the
premium was not paid. The INSURED sued the COMPANY for indemnity. The trial court
held the COMPANY liable to indemnify the INSURED on the ground that since the
COMPANY could have demanded payment of the premium, mutuality of obligation
required that it should be liable on the policy. Hence, this appeal by the COMPANY on
question of law.
ISSUE: Whether or not payment of premiums necessary for effectivity of policy?
2-S, 2013-2014

HELD : YES. The Supreme Court reversed the decision of the trial court.It held that
Section 72 of the Insurance Act as amended by R.A.. 3540 states that "no policy
issued by an insurance company is valid and binding unless and until the premium
thereof has been paid." It is obvious from both the Insurance Act, as amended, and the
stipulation of the parties that time is of the essence in respect to the payment of the
insurance premium so that if it is not paid the contract does not take effect unless
there is still another stipulation to the contrary. In the instant case, the INSURED was
given a grace period to pay the premium but the period having expired with no payment
made, he cannot insist that the COMPANY is nonetheless obligated to him.
Acme Shoe Rubber & Plastic Corporation v. CA 134 SCRA 155
Facts:
Since 1946, ACME had been insuring yearly against fire its building, machines and
general merchandise, located at Caloocan City, with respondent Domestic Insurance
Company of the Philippines. On May 14, 1962, ACME continued to insure its properties
with Domestic and was issued a policy in the amount of P200,000.00 for the period
May 15, 1962 up to May 15, 1963. Domestic issued a renewal receipt to cover the
period May 15, 1963 to May 15, 1964. On January 8, 1964, ACME paid P3,331.26 as
premium.Domestic applied the payment as renewal premium for the period May 15,
1963 to May 15, 1964. A receipt was issued for the renewal premium of P3,331.26 for
the period May 15, 1964 to May 15, 1965. Stamped on the receipt was the:
“"Note: Subject to 'Receipt of Payment Clause' and 'Credit Agreement'
attached hereto and forming part hereof."
The clauses mentioned, which were attached as riders to Renewal Receipt No. 30127,
respectively read as follows:
"RECEIPT OF PAYMENT CLAUSE
"IT IS HEREBY DECLARED AND AGREED that notwithstanding anything to the
contrary contained in the within policy, this insurance will be deemed valid and
binding upon the Company only when the premium and documentary stamps
therefor have actually been paid in full and duly acknowledged in an official
receipt signed by an authorized official/representative of the Company"
"CREDIT AGREEMENT
"The premium corresponding to the first ninety days of the term of this
policy or any renewal thereof is hereby considered paid for the purpose only of
making this Policy valid and binding during said portion of the term. Thereafter,
this Policy shall automatically become void and ineffective (without prejudice
to the obligation of the Insured to pay the corresponding short period premium
for the said 90 days) unless prior to the expiration of said period the Insured
shall have actually paid to the Company the total premium and the
documentary stamps stipulated in this Policy." (Exhibit 'E-2')”
ACME thru its president, also signed a promissory note, promising to pay Domestic
within 90 days from the effective date of the policy (May 15, 1964) the premium and
68

Insurance Case Digests
Atty. Bathan-Basuel

the documentary stamps in the sum of P3,331.26. Otherwise, the policy will be
automatically cancelled.
ACME's properties were completely destroyed by fire on October 13, 1964. ACME filed
its insurance claim but the INSURER disclaimed liability on the ground that as of the date
of loss, the properties burned were not covered by insurance.
Issue: WON there is no insurance contract since Domestic accepted a one-year
premium on January 8, 1964
Held:
Yes, there is no insurance contract. By the express terms of the Promissory Note signed
by its President, ACME was fully aware that the policy would be automatically cancelled
on August 13, 1964, the 90th day from March 14, 1964, if it did not pay the premium
before the former date. There is also evidence to the effect that various reminders by
the INSURER for payment remained unheeded (Exhibit "10"). Not having paid the 19641965 premium within the extension granted, and pursuant to R.A. No. 3540, the policy
was automatically cancelled and there was no insurance coverage to speak of as of the
date of the fire on October 13, 1964.
The pertinent provision of Republic Act No. 3540, approved on June 20, 1963, and put
into effect by the Office of the Insurance Commissioner beginning October 1, 1963
(Exhibit "11"), reads:
"Sec. 72.An insurer is entitled to payment of the premium as soon as the thing
insured is exposed to the peril insured against, unless there is clear agreement to
grant the insured credit extension of the premium due. No policy issued by an
insurance company is valid and binding unless and until the premium thereof has
been paid."
Since Republic Act No. 3540 was approved only on June 20, 1963 and was put into
effect only beginning October 1, 1963, it could not retroactively affect the renewal of
the insurance policy on May 15, 1963, or prior to the Act's effective date. ACME's
premium payment of January 8, 1964, therefore, was properly applied to the 19631964 premium.
ACME's claim that the INSURER would unjustly enrich itself if it were to be allowed to
apply the one-year premium it received to a past period when the policy was void and
the INSURER had incurred no risk, is flawed for the reason already stated that Renewal
Receipt No. 22989 for 1963-1964 had been issued on May 14, 1963 before R.A. No.
3540 was approved on June 20, 1963 and implemented on October 1, 1963.What
became automatically cancelled by R.A. No. 3540 was the 1964-1965 policy for
ACME's failure to pay the premium within the 90-day extension granted, and in
accordance with the express terms of the Promissory Note that it had signed.
Valenzuela vs. CA 191 SCRA 1
FACTS:
2-S, 2013-2014

1) Arturo P. Valenzuela as a General Agent of Philippine American General Insurance
Company, Inc. (Philamgen) since 1965, was authorized to solicit and sell in behalf of
Philamgen all kinds of non-life insurance, and in consideration of services rendered was
entitled to receive the full agent's commission of 32.5% from Philamgen under the
scheduled commission rates. From 1973 to 1975, Valenzuela solicited marine insurance
from one of his clients, the Delta Motors, Inc. (Division of Electronics Airconditioning
and Refrigeration) in the amount of P4.4M from which he was entitled to a commission
of 32%. However, Valenzuela did not receive his full commission which
amounted to P1.6M from the P4.4 M insurance coverage of the Delta
Motors. During the period 1976 to 1978, premium payments amounting to
P1,946,886.00 were paid directly to Philamgen and Valenzuela's commission to which
he is entitled amounted to P632,737.00.
2) In 1977, Philamgen started to become interested in and expressed its intent to share
in the commission due Valenzuela on a 50-50 basis. Valenzuela refused.
3) On February 8, 1978 Philamgen and its President, Bienvenido M. Aragon insisted on
the sharing of the commission with Valenzuela. This was followed by another sharing
proposal dated June 1, 1978. On June 16,1978, Valenzuela firmly reiterated his
objection to the proposals. Because of the refusal of Valenzuela, Philamgen and its
officers, namely: Bienvenido Aragon, Carlos Catolico and Robert E. Parnell took drastic
action against Valenzuela. They: (a) reversed the commission due him by not crediting
in his account the commission earned from the Delta Motors, Inc. insurance; (b) placed
agency transactions on a cash and carry basis; (c) threatened the cancellation of
policies issued by his agency; and (d) started to leak out news that Valenzuela has a
substantial account with Philamgen. All of these acts resulted in the decline of
his business as insurance agent. Then on December 27, 1978, Philamgen
terminated the General Agency Agreement of Valenzuela.
TRIAL COURT: Ruled in favor of Valenzuela – “Since defendants are not justified in the
termination of Arturo P. Valenzuela as one of their General Agents, defendants shall be
liable for the resulting damage and loss of business of Valenzuela.”
CA: Ruled in favor of Philamgen, ordered Valenzuela to pay Philamgen the amount
corresponding to the unpaid and uncollected premiums.
ISSUE: Whether or not the Valenzuela is liable to Philamgen for the unpaid and
uncollected premiums?
HELD: NO, Valenzuela is not liable to Philamgen for the unpaid and uncollected
premiums.
Ø Section 77 of the Insurance Code: [N]otwithstanding any agreement to
the contrary, no policy or contract of insurance is valid and binding unless and
until the premiums thereof have been paid except in the case of a life or
industrial life policy whenever the grace period provision applies (P.D. 612, as
amended otherwise known as the Insurance Code of 1974).
Ø

Perforce, since admittedly the premiums have not been paid, the policies
issued have lapsed. The insurance coverage did not go into effect or did not
continue and the obligation of Philamgen as insurer ceased. Hence, for
Philamgen which had no more liability under the lapsed and inexistent policies
69

Insurance Case Digests
Atty. Bathan-Basuel

to demand, much less sue Valenzuela for the unpaid premiums would be the
height of injustice and unfair dealing. In this instance, with the lapsing of the
policies through the nonpayment of premiums by the insured there were no
more insurance contracts to speak of.
Ø

In Philippine Phoenix Surety and Insurance, Inc. v. Woodworks, Inc. ,
SC held that the non-payment of premium does not merely suspend but puts
an end to an insurance contract since the time of the payment is peculiarly of
the essence of the contract. And in Arce v. The Capital Insurance and
Surety Co. Inc. , SC reiterated the rule that unless premium is paid, an
insurance contract does not take effect.

Makati Tuscany Condominium Corp. V. CA 215 SCRA 462
FACTS
Ø

Ø

Ø

Ø
Ø

Ø

Ø
Ø

1982: The American Home Assurance Co. issued in favor of Makati Tuscany
Condominium Corp. an insurance policy on the latter’s building and premises.
The premium was paid on instalments, all of which were accepted by the
insurer.
1983: The insurer issued to the Makati Tuscany another policy which replaced
and renewed the previous polcy. The premium was again paid on instalments,
all of which were accepted by the insurer.
1984: The policy was again renewed and insurer issued to insured another
policy. On this renewed policy, the insured made two instalment payments,
both accepted by the insurer. Thereafter, the insured refused to pay the
balance of the premium.
The insurer thus filed an action to recover the unpaid balance.
Insured explained that it discontinued the payment because the policy did not
contain a credit clause in its favor and the receipts for the instalment
payments covering the policy for 1984-1985 as well as the two previous
policies stated the ff. reservations:
(2) Acceptance of this payment shall not waive any of the company rights to
deny liability on any claim under the policy arising before such payments or
after the expiration of the credit clause of the policy;
(3) Subject to no loss prior to premium payment. If there be any loss such is
not covered.
Insured further claimed that the policy was never binding and no risk attached
to the policy. It pleaded for a counterclaim for the premiums already paid from
1982 to 1985.
Trial court dismissed the complaint and the counterclaim. Both appeals.
CA ordered the insured to pay the balance of the premiums. It explained that
the obligation to pay when due is ordinarily an indivisible obligation to pay the
entire premium. Here, the parties agreed to make the premiums payable in
INSTALLMENTS. It said that while it may be true under Section 77 that the
parties may not agree to make the insurance contract valid and binding w/o
payment of premiums, THERE IS NOTHING IN SAID SECTION WHICH SUGGESTS

2-S, 2013-2014

Ø

THAT THE PARTIES MAY NOT AGREE TO ALLOW PAYMENT OF THE PREMIUMS
IN INSTALLMENTS, OR TO CONSIDER THE CONTRACT AS VALID AND BINDING
UPON PAYMENT OF THE FIRST PREMIUM. Otherwise, we would allow the insurer
to renege on its liability under the contract had a loss occurred before the
completion of payment of the entire premium despite the insurer’s voluntary
acceptance of partial payments. THE INSURANCE CONTRACT BECAME VALID
AND BINDING UPON PAYMENT OF THE FIRST PREMIUM AND THE INSURER
COULD NOT HAVE DENIED LIABILITY ON THE GROUND THAT PAYMENT WAS
NOT MADE IN FULL FOR THE REASON THAT IT AGREED TO ACCEPT
INSTALLMENT PAYMENTS.
The insured now asserts that its payment by instalment of the premiums for
the insurance policies for 1982, 1092 and 1094 invalidates said policies
because of Sec. 77 and by the conditions stipulated by the insurer in the
receipts, disclaiming liability for loss occurring before payment of premiums.
The insured concludes that there could not be a perfected contract of
insurance upon mere partial payment of the premiums.

ISSUE
Whether payment by instalment of the premiums due on an insurance policy invalidates
the contract of insurance in view of Section 77 of PD 612
RULING
The policies are valid even if the premiums were paid on instalments
(1)
Parties intended subject insurance polices to be binding and effective
notwithstanding the staggered payment of the premiums. Basic principles
of equity and fairness would not allow the insurer to continue collecting
and accepting the premiums although paid on instalments and later deny
liability on the lame excuse that the premiums were not prepaid in full.
(2)
While the import of Sec. 77 is that prepayment of premiums is strictly
required as a condition to the validity of the contract, we are not
prepared to rule that the request to make instalment payments duly
approved by insurer would prevent the entire contract from going into
effect despite payment and acceptance of the first instalment/ initial
premium. SECTION 78 allows waiver by the insurer of the condition of
prepayment by making an acknowledgment in the insurance policy of
receipt of premium as conclusive evidence of payment so far as to make
the policy binding despite the fact that premium is actually unpaid.
(3)
Insured not entitled to a refund of the premiums. Parties intended to
make the three insurance contracts valid. Hence, petitioner may not be
allowed to renege on its obligation to pay the balance after the expiration
of the term of the third policy. Where the risk is entire and the contract is
indivisible, the insured is not entitled to a refund of the premiums paid if
the insurer was exposed to the risk insured for any period, however brief
or momentary.
UCPB General Insurance Co., INc. v. Masagana Telmart Inc., G.R. No. 137172 April
4, 2001
70

Insurance Case Digests
Atty. Bathan-Basuel

FACTS: Masagana Telamart, Inc. (Telamart) obtained from UCPB Gen. Insurance Co., Inc.
(UCPB) 5 insurance policies on its properties in Pasay City and Manila which all reflect
the effectivity term: “from 4PM of May 22, 1991 to 4PM of May 22, 1992”.
On June 13, 1992, Telamart’s properties located in Taft Avenue, Pasay City were razed
by fire.
On July 13, 1992, Telamart tendered and UCPB accepted 5 Equitable Bank Manager’s
Checks in the total amount of P225,753.45 as renewal premium payments for which
Official Receipt Direct Premium No. 62926 was issued by UCPB.
On July 14, 1992, Telamart made its formal demand for indemnification for the burned
insured properties. UCPB returned the 5 manager’s checks stating in its letter that it
was rejecting Telamart’s claim on the ff. grounds:
a. The policies expired last May 22, 1992 and were not renewed for another term;
b. UCPB had put Telamart and its alleged broker on notice of non-renewal earlier; and
c. The properties covered by the said policies were burned in a fire that took place
last June 13, 1992, or before tender of premium payment.
Hence, Telamart filed this case. The trial court (a) allowed Telamart to consign
P225,753.95 as full payment of the premiums for the renewal of the 5 insurance
policies on Telamart’s properties; (b) declared the replacement renewal
policies
effective and binding from May 22, 1992 until May 22, 1993; and (c) ordered UCPB to
pay Telamart P18,645,000 as indemnity for the burned properties covered by the
renewal-replacement policies.

On June 15, 1999, the SC ruled that the fire insurance policies issued by UCPB to
Telamart covering the period from May 22, 1991 to May 22, 1992 had NOT been
extended nor renewed by an implied credit arrangement though actual payment of
premium was tendered on a later date and after the occurrence of the (fire) risk insured
against, pursuant to Sec. 77 of the Insurance Code. Telemart filed an MR.
No notice of non-renewal was made within 45 days before 22 May 1992, or before the
expiration date of the fire insurance policies. Thus, the policies in question were
renewed by operation of law and were effective and valid on 30 June 1992 when the
fire occurred, since the premiums were paid within the 60- to 90-day credit term.
The trial court and the CA established the ff. facts:
1. For years, UCPB had been issuing fire policies to Telamart, and these policies were
annually renewed.
2. UCPB had been granting Telamart a 60- to 90-day credit term within which to pay
the premiums on the renewed policies.
3. There was no valid notice of non-renewal of the policies in question, as there is no
proof at all that the notice sent by ordinary mail was received by Telamart and the
copy thereof allegedly sent to Zuellig was ever transmitted to Telamart.
4. The premiums for the policies in question in the aggregate amount of P225,753.95
were paid by Telamart within the 60- to 90-day credit term and were duly
accepted and received by UCPB’s cashier.
ISSUE: Whether Sec. 77 of the Insurance Code of 1978 (PD 1460) must be strictly
applied to UCPB’s advantage despite its practice of granting a 60- to 90-day credit
term for the payment of premiums.

CA reversed and set aside the decision (1) deleting the declaration that 3 of the
policies were in force from August 1991 to August 1992; and (2) reducing the award
of the attorney’s fees from 25% to 10% of the total amount due to Telamart; because
the tender of payment of the premiums on July 13, 1992 did not result in the renewal
of the policies, having been made beyond the effective date of renewal as provided
under Policy Condition No. 26, which states:

HELD: NO. Section 77 of the Insurance Code of 1978 provides:
SEC. 77. An insurer is entitled to payment of the premium as soon as the thing insured
is exposed to the peril insured
against. Notwithstanding any agreement to the contrary,
no policy or contract of insurance issued by an insurance company is valid and binding
unless and until the premium thereof has been paid, except in the case of a life or an
industrial life policy whenever the grace period provision applies.

26. Renewal Clause. -- Unless the company at least forty five days in advance of the
end of the policy period mails or delivers to the assured at the address shown in the
policy notice of its intention not to renew the policy or to condition its renewal upon
reduction of limits or elimination of coverages, the assured shall be entitled to renew
the policy upon payment of the premium due on the effective date of renewal.

There are, however, exceptions to Sec. 77:
1. In case of a life or industrial life policy whenever the grace period provision applies
(Sec. 77)
2. Any acknowledgement in a policy or contract of insurance of the receipt of
premium is conclusive evidence of its payment so far as to make the policy binding,
notwithstanding any stipulation therein that it shall not be binding until premium is
actually paid. (Sec. 78)
3. If the parties agreed to the payment in installments of the premium and partial
payment has been made at the time of loss.
4. The insurer may grant credit extension for the payment of the premium.
5. Estoppel.

Both the trial court and the CA found that sufficient proof exists that Telamart which
had procured insurance coverage from UCPB for a number of years had been granted a
60- to 90-day credit for the renewal of the policies. Such a practice had existed up to
the time the claims were filed. Also, the CA ruled that that no timely notice of nonrenewal was made by UCPB.

2-S, 2013-2014

71

Insurance Case Digests
Atty. Bathan-Basuel

While the import of Section 77 is that prepayment of premiums is strictly required as a
condition to the validity of the contract, the request to make installment payments duly
approved by the insurer would NOT prevent the entire contract of insurance from going
into effect despite payment and acceptance of the initial premium or first installment.
Section 78 of the Insurance Code in effect allows waiver by the insurer of the
condition of prepayment by making an acknowledgment in the insurance policy of
receipt of premium as conclusive evidence of payment so far as to make the policy
binding despite the fact that premium is actually unpaid. Section 77 merely precludes
the parties from stipulating that the policy is valid even if premiums are not paid, but
does not expressly prohibit an agreement granting credit extension, and such an
agreement is not contrary to morals, good customs, public order or public policy. So is
an understanding to allow insured to pay premiums in installments not so prescribed. At
the very least, both parties should be deemed in estoppel to question the
arrangement they have voluntarily accepted.
Hence, if the insurer has granted the insured a credit term for the payment of the
premium and loss occurs before the expiration of the term, recovery on the policy
should be allowed even though the premium is paid after the loss but within the credit
term.
There is nothing in Section 77 which prohibits the parties in an insurance contract to
provide a credit term within which to pay the premiums. That agreement is not against
the law, morals, good customs, public order or public policy. The agreement binds the
parties. Article 1306 of the Civil Code provides:

ART. 1306. The contracting parties may establish such stipulations clauses, terms and
conditions as they may deem convenient, provided they are not contrary to law, morals,
good customs, public order, or public policy.
It would be unjust and inequitable if recovery on the policy would not be permitted
against UCPB, which had consistently granted a 60- to 90-day credit term for the
payment of premiums despite its full awareness of Section 77. Estoppel bars it from
taking refuge under said Section, since Respondent relied in good faith on such practice.
WHEREFORE, the Decision in this case of 15 June 1999 is RECONSIDERED and SET
ASIDE, and a new one is hereby entered DENYING the instant petition for failure of
Petitioner to sufficiently show that a reversible error was committed by the Court of
Appeals in its challenged decision, which is hereby AFFIRMED in toto.

Phil. Pryce Assurance Corp. v. CA 230 SCRA 164
FACTS:
Respondent Gegroco, Inc. filed a complaint for collection of sum of money against
petitioner, Interworld Assurance Corporation (the company now carries the corporate
2-S, 2013-2014

name Philippine Pryce Assurance Corporation). The complaint alleged that Pryce Corp.
issued two surety bonds in behalf of its principal Sagum General Merchandise for
(P500,000.00) PESOS and (1,000,000.00) PESOS, respectively. Pryce admitted having
executed the said bonds, but denied liability because allegedly 1) the checks which were
to pay for the premiums bounced and were dishonored hence there is no contract to
speak of between petitioner and its supposed principal; and 2) that the bonds were
merely to guarantee payment of its principal's obligation, thus, excussion is necessary.
Petitioner Pryce failed to attend the pre-trial on several occasions and was therefor held
in default and respondent was allowed to present evidence ex-parte. Both RTC and CA
ruled in favor of respondent Gegroco Inc.
ISSUE: Whether Pryce should be held liable for the surety bond which it issued
HELD:
Yes. The Insurance Code states that:
"SECTION 177.The surety is entitled to payment of the premium as soon as the
contract of suretyship or bond is perfected and delivered to the obligor. No contract of
suretyship or bonding shall be valid and binding unless and until the premium therefor
has been paid, except where the obligee has accepted the bond, in which case the bond
becomes valid and enforceable irrespective of whether or not the premium has been
paid by the obligor to the surety. . . ." (emphasis added)
The above provision outrightly negates petitioner's first defense. In a desperate
attempt to escape liability, petitioner further asserts that the above provision is not
applicable because the respondent allegedly had not accepted the surety bond, hence
could not have delivered the goods to Sagum Enterprises.
The above provision outrightly negates petitioner's first defense. In a desperate
attempt to escape liability, petitioner further asserts that the above provision is not
applicable because the respondent allegedly had not accepted the surety bond, hence
could not have delivered the goods to Sagum Enterprises. This statement clearly
intends to muddle the facts as found by the trial court and which are on record. cdrep
In the first place, petitioner, in its answer, admitted to have issued the bonds subject
matter of the original action. 19 Secondly, the testimony of Mr. Leonardo T. Guzman,
witness for the respondent, reveals the following:

"Q.What are the conditions and terms of sales you extended to Sagum General
Merchandise?
A.First, we required him to submit to us Surety Bond to guaranty payment of the spare
parts to be purchased. Then we sell to them on 90 days credit. Also, we required them
to issue post-dated checks.
72

Insurance Case Digests
Atty. Bathan-Basuel

Q.Did Sagum General Merchandise comply with your surety bond requirement?
A.Yes. They submitted to us and which we have accepted two surety bonds.
On the other hand, petitioner's defense that it did not have authority to issue a Surety
Bond when it did is an admission of fraud committed against respondent. No person can
claim benefit from the wrong he himself committed. A representation made is rendered
conclusive upon the person making it and cannot be denied or disproved as against the
person relying thereon.

whether at the time the insurance policy was delivered to Plastic, the latter was able to
pay the premium. It appears on record that Plastic did not, but instead executed an
acknowledgment receipt of the policy. In said receipt, Plastic promised to pay the
premium w/in 30 days from the effectivity date of the policy and Capital Insurance
accepted it.
What then is the effect of accepting such acknowledgment receipt? Did capital
insurance mean to agree to make good its undertaking under the policy if the premium
could not be paid on or before Jan 16? What would be the effect of the delivery to
Capital of the postdated check? Could not this have been considered a valid payment?
#damingtanong

Capital Ins. & Surety Cor., Inc v. Plastic Era Co., Inc. 65 SCRA 134
RULING
FACTS
Ø

Ø

Ø
Ø
Ø

Ø

Ø

Ø

Capital Insurance & Surety Inc. delivered to Plastic Era Manufacturing Co. Inc.
an open fire policy wherein the former undertook to insure the latter’s
building, equipment, raw materials, products, and accessories located at
Sheridan Street, Mandaluyong, Rizal.
The policy expressly provides that if the property insured would be destroyed
or damaged by fire after the payment of the premiums, the insurance
company shall make good all such loss/damage in an amount not exceeding
100K.
When the policy was delivered, Plastic Era failed to pay the corresponding
insurance premium.
HOWEVER,
its
duly
authorized
representative
executed
the
an
acknowledgment receipt.
Subsequently, in partial payment of the premium, Plastic Era delivered to
Capital Insurance a check postdated January 16, 1961 (date when premium is
due) payable to the order of the latter. However, Capital tried to deposit the
check only on Feb. 20, 1961 and the same was dishonoured for lack of funds.
On January 18, 1961, 2 days after the insurance premium became due, the
property insured was destroyed by fire. Plastic Era notified the insurer of the
loss and filed its claim for indemnity. The loss was estimated to be P283,875.
However, the records of the same property show that it has been insured by
Plastic Era with the PhilamGen Insurance Company for 200K.
In less than a month, Plastic era demanded from Capital the payment of 100K
as indemnity for the loss but the latter refused because Plastic failed to pay
the insurance premium.
Plastic filed its complaint against Capital for the recovery of 100K. Trial court
ruled in favor of Plastic. Capital appealed to CA which affirmed the trial court
decision. Hence, this petition.

ISSUE
Whether or not a contract of insurance has been duly perfected.
The insurance policy provides it is only upon payment of the premiums by Plastic that
Capital agrees to insure the properties of the former. The crux of the problem then is
2-S, 2013-2014

(1) Under Article 1249, the mere delivery of a BOE in payment of a debt does not
immediately effect payment. Tender of draft or check to effect payment that
would extinguish the liability should actually be cashed. If the delivery of the
check were to be viewed in light of the foregoing, NO PAYMENT OF THE
PREMIUM HAD BEEN EFFECTED, for it is only when the check is cashed that it is
said to effect payment.
(2) By accepting the promise of Plastic to pay the premium w/in 30 days from the
effective date of the policy, Capital implicitly agreed to MODIFY THE TENOR OF
THE POLICY and in effect WAIVED THE PROVISION THEREIN THAT IT WOULD
ONLY PAY FOR THE LOSS OR DAMAGE IN CASE THE SAME OCCURS AFTER THE
PAYMENT OF THE PREMIUM. Considering that the policy is silent as to the
mode of payment, Capital is deemed to have accepted the promissory note in
payment of the premium. This rendered the policy IMMEDIATELY OPERATIVE
ON THE DATE IT WAS DELIVERED.
In US cases: Although one of the conditions of an insurance policy is that it
shall not be valid until the payment of the first premium, if the policy is silent
as to the mode of payment, promissory notes received by the company must
be deemed to have been accepted in payment of the premium. In other words,
a requirement for the payment of the 1st or initial premium in advance or
actual cash may be waived by acceptance of a promissory note. This is what
happened in this case when Capital accepted the receipt promising to pay w/in
30 days from Dec. 17. Hence, when the damage or loss of the insured
property occurred, the insurance policy was in full force and effect. THE FACT
THAT THE CHECK WAS LATER ON DISHONORED DID NOT OPERATE AS A
FOREFEITURE OF ITS RIGHTS UNDER THE POLICY, there being no express
stipulation therein to that effect. If the check is accepted as payment of the
premium even though it turns out be worthless, there is payment which will
prevent forfeiture.
(3) By accepting the promise of Plastic, Capital had in effect extended credit to
Plastic. The payment of the premium therefore became an independent
obligation the non fulfilment of which would entitle Capital to recover. It could
just deduct the premium due and unpaid upon the satisfaction of the loss
73

Insurance Case Digests
Atty. Bathan-Basuel

under the policy. It did not have the right to cancel the policy for non-payment
except by putting Plastic Era in default and giving it personal notice to that
effect. This Capital failed to do. Where credit is given by the insurer for the
payment of the premium, it has no right to cancel the policy for non-payment
except by putting the insured in default and giving him personal notice. On the
other hand, Capital had accepted a check from Plastic in partial payment of
the premium. Having held the check for 35 days before presenting it for
payment (an unreasonable period of time), Capital was stopped from claiming
a forfeiture even if the check had been dishonoured later. Where the check is
held for an unreasonable time before presenting it for payment, the insurer
may be held estopped from claiming a forfeiture if the check is dishonoured.

2-S, 2013-2014

74

Sponsor Documents

Or use your account on DocShare.tips

Hide

Forgot your password?

Or register your new account on DocShare.tips

Hide

Lost your password? Please enter your email address. You will receive a link to create a new password.

Back to log-in

Close